Sunteți pe pagina 1din 78

CHAPTER 8: AFFIRMATIVE DEFENSES PART 3.

ASSUMPTION OF RISK When the plaintiff voluntarily assumed a known risk, courts refused him a right to recover. There has been intense debate as to whether implied assumption of risk has been swallowed by comparative fault. For EXPRESS assumption of risk, the plaintiff assumes the risk when: 1. HE KNOWS OF THE RISK; AND 2. VOLUNTARILY 3. ASSUMES IT
A.

EXPRESS ASSUMPTION OF RISK (remains a viable defense in all jurisdictions) When the defendant agreed to allow plaintiff to be exposed to her conduct only if the plaintiff agreed to exculpate the defendant from liability from negligence. There are two factors in favor of giving effect to such contracts: i. Because the defendant acts in reliance on the plaintiffs agreement not to hold her liable this is similar to the consent defense in intentional torts. ii. Agreements made in advance of entering a relationship can specify the scope of the conduct covered by the contractual exculpation these are more defined and can set out the kind of negligent conduct to be exculpated from liability. EVANS V. LIMA LIMA FLIGHT TEAM Although exculpatory agreements are not favored and are strictly construed against the party they benefit, parties may allocate the risk of negligence as they see fit, and exculpatory agreements do not violate public policy as a matter of law. An exculpatory agreement will be enforced if: (1) it clearly spells out the intention of the parties; (2) there is nothing in the social relationship between the parties militating against enforcement; and (3) it is not against public policy. An exculpatory agreement must contain clear, explicit, and unequivocal language referencing the type of activity, circumstance, or situation that it encompasses and for which the plaintiff agrees to relieve the defendant from a duty of care. However, the parties need not have contemplated the precise occurrence which results in injury. The injury must only fall within the scope of possible dangers ordinarily accompanying the activity and, therefore, reasonably contemplated by the parties. Whether an injury accompanies a certain activity is ordinarily a question of fact, precluding summary The court looks at three factors to determine if the release clause is enforceable. First, whether the release is clear, unambiguous and explicit in expressing the intent of the parties. Second, the act of negligence, which results in injury to the releaser, must be reasonably related to the object or purpose for which the release is given. Third, whether the release is contrary to public policy. Broad releases will be held against the drafter. PUBLIC POLICY EXCEPTION Courts retain the right to deny enforcement of exculpation clauses when they believe that society would be ill served if they were given effect. The following factors should be taken into account from a public policy standpoint: i. whether the business was of a type generally thought suitable for public regulation ii. whether the party seeking exculpation was engaged in performing a service of great importance to the public, which is often a matter of practical necessity for some members of the public iii. whether the party holds itself out as willing to perform its service for any member of the public who seeks it

iv. v.

whether the party invoking the exculpation clause possesses superior bargaining power whether the party confronts the public with a standard adhesion contract of exculpation and makes no provision whereby a purchaser may pay an additional fee and obtain protection against negligence.

TUNKL V. REGENTS OF THE UNIVERSITY OF CALIFORNIA The court found that the agreement exhibited all of the characteristics set forth by the courts of the type of transaction in which exculpatory provisions would be held invalid, including the following: (1) the agreement involved an institution subject to public regulation; (2) the hospital's services to those who needed the particular skill of its staff and facilities constituted a practical and crucial necessity; and (3) the hospital held itself out as willing to perform its services for those qualified members of the public. PARENTAL WAVING OF ASSUMPTION OF RISK Some courts have refused to enforce the waiver signed by parents on behalf of their children because it creates an unacceptable conflict of interest.
B.

IMPLIED ASSUMPTION OF RISK When there is no express agreement, either oral or written, between the parties, but actions of the plaintiff act as an implied agreement to accept the risk this is different from contributory negligence because the plaintiff VOLUNTARILY accepts the risk - before you jump to this type of analysis, you must go through the plaintiffs prima facie case against the defendant for duty, breach, causation and harm because this is a DEFENSE. BLACKBURN V. DORTA Implied assumption of risk is merged into the defense of contributory negligence and the principles of comparative negligence. PRIMARY IMPLIED ASSUMPTION OF RISK do not get to defenses unless the defendant is negligent so the defendant in these cases has no duty SECONDARY IMPLIED ASSUMPTION OF RISK - reasonable save child from fire (was a complete bar of recovery) courts did away with this under the duty element UNREASONABLE save hat from fire (allowed recovery minus the contributory negligence) This is unfair and the reason the courts have done away with this doctrine TURCOTTE V. FELL Assumption of risk is not a complete defense but is a measure of the defendants duty of care. Tripp hates this case because it is a duty case and they bring in a defense instead of analyzing it based on the prima facie case of duty. Some states have completely done away with implied assumption of risk as an independent defense. Most states treat implied assumption of risk as a form of fault to be compared with that of the defendant Courts still struggle between the no-duty/assumption of risk rule and when to compare the plaintiffs and defendants faults according to comparative fault. Bottom line on Assumption of Risk EXPRESS AOR IS STILL VALID IN EVERY JURISDICTION AND IS A TOTAL BAR TO RECOVERY IMPLIED AOR SOME JURISDICTION, SMALL NUMBER, STILL A TOTAL BAR TO RECOVERY OTHER COURTS MAINTAIN IMPLIED AOR BUT TREAT IT JUST LIKE COMPARATIVE FAULT IF YOU REASONABLY ASSUME THE RISK AND YOU ARE INJURED BUT HAVE ACTED REASONABLY (SAVING A CHILD FROM A BURNING HOUSE), THEN YOUR DAMAGES ARE NOT REDUCED .

PART 4. AVOIDABLE CONSEQUENCES When the plaintiff, after the injury, fails to take reasonable steps to avoid aggravation of the injury and mitigating the damages, the plaintiff will be unable to recover for any damages that she herself either caused to happen after the initial injury or did nothing about which made the initial injury worse than it should have been. NOVKO V. STATE OF NEW YORK A dairy farmer suffered serious injuries, making it difficult to continue farming, after he was rear-ended by a New York state trooper's cruiser. The appeals court held that although the farmer properly received no award for lost earning capacity, because he failed to prove it, new trials were required to determine damages for pain and suffering and the wife's derivative claim, because the trial court's assertion that they should have mitigated their damages by giving up farming was erroneous and based on pure speculation that lucrative employment as an equipment salesman was available. Mitigation of damages is usually thought of in the context of steps that a reasonable plaintiff could have taken subsequent to the incident giving rise to injuries, in order to reduce the amount of damages. As a general rule, a party who claims to have suffered damage by the tort of another is bound to use reasonable and proper efforts to make the damage as small as practicable, and if an injured party allows the damages to be unnecessarily enhanced, the incurred loss justly falls upon him. PRE-INJURY AVOIDABLE CONSEQUENCES Example of seat belts some jurisdictions do not allow evidence regarding whether the plaintiff was wearing a seat belt. Other jurisdictions allow evidence but cap liability to the plaintiff at a certain percentage. Some jurisdictions also allow a helmet defense if a motorcyclist is not wearing a helmet. B. NON-CONDUCT BASED DEFENSES 1. IMMUNITIES
A. INTRAFAMILY IMMUNITIES

the law used to recognize complete immunity for lawsuits between family members. However, the law has evolved to allow children to sue parents for auto accidents due to insurance claims. There are two main concerns the court has for allowing family members to sue one another: 1) it would bring about family discord and 2) that it would result in collusive actions between family members to cook up a story to defraud insurance companies. The first concern is not really all that necessary because in run of the mill negligence in driving cases, the fear of discord is remote. There is still concern about collusion, but if these types of suits were excluded, the court would have to bar suits between close friends and other relatives. (1) INTERSPOUSAL IMMUNITY: suits are also now usually allowed because of the Married Womens Property Acts. In most states negligence actions proceed no differently than those involving strangers. Some courts allow claims for battery and IIED, but other courts will not allow cases that involve infidelity and other forms of marital distress within the marriage as sufficiently outrageous. (2) PARENTAL IMMUNITY: now allows children to sue parents because of the widespread availability of insurance. There are two situations that are still covered under parental immunity, 1) where the negligence involved parental discipline and 2) where the negligence involved exercise of normal parental discretion over such matters as providing food, housing and medical services. However, most states have abandoned parental immunity and allow children to sue their parents for negligence, especially involving automobile accidents.

B.

CHARITABLE IMMUNITY Charitable organizations were immune from actions based on the charities negligence. The theory behind immunity was that if people were allowed to sue charities that all of the charities would cease to exist because the money they collected would be used to pay out lawsuits. While some states have done away with charitable immunity, most states retain the doctrine.
C.

GOVERNMENTAL IMMUNITY The government used to enjoy complete immunity and to this day there is very little in which the government can be sued for. (1) Federal Government: The Federal Tort Claims Act (a) Discretionary Immunity the FTCA abolished tort immunity but carved out exceptions where complete immunity is still binding. It prohibits tort actions based upon the exercise or performance or the failure to exercise or perform a discretionary function of duty on the part of a federal agency or an employee of the Government, whether or not the discretion involved abuse. TIPPETT V. UNITED STATES Plaintiff couple were members of a guided snowmobile tour traveling in a national park. Plaintiff husband was charged by a moose while attempting to pass the mouse and seriously injured. The Federal Tort Claims Act (FTCA) contains an exception to this broad waiver of immunity, however, for claims based upon the exercise or performance or the failure to exercise or perform a discretionary function or duty on the part of a federal agency or an employee of the Government, whether or not the discretion involved be abused. In order to determine whether the discretionary function exception applies the appellate court utilizes a two-prong analysis. The appellate court determines: (1) whether the action at issue is one of choice for the government employee; and (2) if the conduct involves such an element of judgment, whether that judgment is of the kind that the discretionary function exception is designed to shield. MUNICIPAL IMMUNITY RISS V. CITY OF NEW YORK A rejected suitor terrorized appellant by threatening to have appellant killed or maimed if she did not date him. In fear for her life, appellant went to respondent city's police for protection, which was refused. After appellant became engaged to another man, the rejected suitor hired someone to throw lye in appellant's face. No Liability! There is no warrant in judicial tradition or in the proper allocation of the powers of government for the courts, in the absence of legislation, to carve out an area of tort liability for police protection to members of the public. Quite distinguishable, of course, is the situation where the police authorities undertake responsibilities to particular members of the public and expose them, without adequate protection, to the risks which then materialize into actual losses. In a public entity liability context, establishing a special relationship based on a municipality's assumption of a duty requires (1) an assumption by a municipality, through promises or actions, of an affirmative duty to act on behalf of the injured party; (2) knowledge on the part of a municipality's agents that inaction could lead to harm; (3) some form of direct contact between the municipality's agents and the injured party; and (4) that party's justifiable reliance on the municipality's affirmative undertaking. 2. STATUTES OF LIMITATION Most jurisdictions cut off liability for negligence at 2-3 years. A majority of states do not trigger the statute of limitations until the injured party knows are should know of the injury. 3. STATUTES OF REPOSE Several legislatures have enacted statutes to cut off liability after several years for building designers, engineers, etc.

CHAPTER 9: JOINT TORTFEASORS A. CONCERTED ACTION One defendant is held liable for the acts of others, not because he actually caused the harm, but because he actually or tacitly agreed to engage in tortuous activity with others. HERMAN V. WESGATE A guest at a bachelor's party was injured when he was thrown unwillingly overboard. The injured guest filed an action against the boat owners, the host, and other guests and alleged concerted action by all of them. Concerted action liability rests upon the principle that all those who, in pursuance of a common plan or design to commit a tortuous act, actively take part in it, or further it by cooperation or request, or who lend aid or encouragement to the wrongdoer, or ratify and adopt his acts done for their benefit, are equally liable with him. An injured plaintiff may pursue any one joint tortfeasor on a concerted action theory. Such tortfeasor may, in turn, seek contribution from others who acted in concert with him. RESTATEMENT 876: PERSONS ACTING IN CONCERT: For harm resulting to a third person from the tortious conduct of another, one is subject to liability if he (a) does a tortious act in concert with the other or pursuant to a common design with him, or (b) knows that the other's conduct constitutes a breach of duty and gives substantial assistance or encouragement to the other so to conduct himself, or (c) gives substantial assistance to the other in accomplishing a tortious result and his own conduct, separately considered, constitutes a breach of duty to the third person. COMMENTS & ILLUSTRATIONS: Comment on Clause (a): a. Parties are acting in concert when they act in accordance with an agreement to cooperate in a particular line of conduct or to accomplish a particular result. The agreement need not be expressed in words and may be implied and understood to exist from the conduct itself. Whenever two or more persons commit tortious acts in concert, each becomes subject to liability for the acts of the others, as well as for his own acts. The theory of the early common law was that there was a mutual agency of each to act for the others, which made all liable for the tortious acts of any one. Illustrations: 1. A, B, C and D come together to E's house at night to rob. A breaks in E's front door, B ties E up, C beats E and D steals and carries away E's jewelry. A, B, C and D are all subject to liability to E for all damages caused by the trespass to land, the false imprisonment, the battery and the conversion. 2. A and B are driving automobiles on the public highway. A attempts to pass B. B speeds up his car to prevent A from passing. A continues in his attempt and the result is a race for a mile down the highway, with the two cars abreast and both travelling at dangerous speed. At the end of the mile, A's car collides with a car driven by C and C suffers harm. Both A and B are subject to liability to C. b. The same rule is applicable, in general, to tortious acts done pursuant to a common design or plan for cooperation in a tortious line of conduct or to accomplish a tortious end. It is in connection with these common designs or plans that the word "conspiracy" is often used. The mere common plan, design or even express agreement is not enough for liability in itself, and there must be acts of a tortious character in carrying it into

execution. When both parties engage in the acts, each becomes subject to liability for the cars of the other. c. In order for the rule stated in Clause (a) to be applicable, it is essential that the conduct of the actor be in itself tortious. One who innocently, rightfully and carefully does an act that has the effect of furthering the tortious conduct or cooperating in the tortious design of another is not for that reason subject to liability. Illustration: 3. A is drunk and disorderly on the public street. B, C and D, who are all police officers, attempt to arrest A for the misdemeanor committed in their presence. A resists arrest. B and C take hold of A, using no more force than is reasonable under the circumstances. A breaks away and attempts to escape. D draws a pistol and shoots A in the back. B and C are not liable to A for the shooting. Comment on Clause (b): d. Advice or encouragement to act operates as a moral support to a tortfeasor and if the act encouraged is known to be tortious it has the same effect upon the liability of the adviser as participation or physical assistance. If the encouragement or assistance is a substantial factor in causing the resulting tort, the one giving it is himself a tortfeasor and is responsible for the consequences of the other's act. This is true both when the act done is an intended trespass (see Illustrations 4 and 5) and when it is merely a negligent act. (See Illustration 6). The rule applies whether or not the other knows his act is tortious. (See Illustrations 7 and 8). It likewise applies to a person who knowingly gives substantial aid to another who, as he knows, intends to do a tortious act. The assistance of or participation by the defendant may be so slight that he is not liable for the act of the other. In determining this, the nature of the act encouraged, the amount of assistance given by the defendant, his presence or absence at the time of the tort, his relation to the other and his state of mind are all considered. (See Illustration 9). Likewise, although a person who encourages another to commit a tortious act may be responsible for other acts by the other (see Illustration 10), ordinarily he is not liable for other acts that, although done in connection with the intended tortious act, were not foreseeable by him. (See Illustration 11). In determining liability, the factors are the same as those used in determining the existence of legal causation when there has been negligence (see 442) or recklessness. (See 501). Illustrations: 4. A and B participate in a riot in which B, although throwing no rocks himself, encourages A to throw rocks. One of the rocks strikes C, a bystander. B is subject to liability to C. 5. A, a policeman, advises other policemen to use illegal methods of coercion upon B. A is subject to liability to B for batteries committed in accordance with the advice. 6. A and B are members of a hunting party. Each of them in the presence of the other shoots across a public road at an animal, which is negligent toward persons on the road. A hits the animal. B's bullet strikes C, a traveler on the road. A is subject to liability to C. 7. A persuades B, who is not an officer, to arrest C for a crime which A tells B was committed by C but which he knows has not been committed by anyone. A is subject to liability to C. 8. A sells to B for resale a gun known by him to be dangerously defective. B negligently fails to examine the gun before selling it to C, who is hurt while attempting to discharge it. A is subject to liability to C.

9. A is employed by B to carry messages to B's workmen. B directs A to tell B's workmen to tear down a fence that B believes to be on his own land but that in fact, as A knows, is on the land of C. A delivers the message and the workmen tear down the fence. Since A was a servant used merely as a means of communication, his assistance is so slight that he is not liable to C. 10. A and B conspire to burglarize C's safe. B, who is the active burglar, after entering the house and without A's knowledge of his intention to do so, burns the house in order to conceal the burglary. A is subject to liability to C, not only for the conversion of the contents of the safe but also for the destruction of the house. 11. A supplies B with wire cutters to enable B to enter the land of C to recapture chattels belonging to B, who, as A knows, is not privileged to do this. In the course of the trespass upon C's land, B intentionally sets fire to C's house. A is not liable for the destruction of the house. Comment on Clause (c): e. When one personally participates in causing a particular result in accordance with an agreement with another, he is responsible for the result of the united effort if his act, considered by itself, constitutes a breach of duty and is a substantial factor in causing the result, irrespective of his knowledge that his act or the act of the other is tortious. Thus each of a number of trespassers who are jointly excavating a short ditch is liable for the entire harm done by the ditch, although each reasonably believes that he is not trespassing. In a large undertaking to which the services of many persons contribute, the contribution to the enterprise of one individual may be so small as not to constitute substantial assistance within the meaning of the rule stated in this Section. Thus a workman who tortiously excavates for the foundation of one of a series of buildings to be used by a manufacturing plant is not necessarily a co-tortfeasor with other workmen simultaneously tortiously excavating for other buildings upon the same premises. It is to be noted that a person may be privileged, and hence be committing no breach of duty, in assisting another who is committing or who later commits a tort. Thus one who reasonably believes that he is defending another against an aggressor may not be liable although the other is in fact the aggressor. (See 76). Further, one who assists in doing an act that from his standpoint does not involve elements of undue risk is not liable merely because another with whom he co-operates is negligent. (See Illustration 12). Likewise one who supplies another with the means of committing a tort is not liable if he has no reason to suppose that a tort will be committed. (See Illustrations 13 and 14). In none of these cases is the defendant committing a breach of duty to the injured person. Illustrations: 12. A and B hunt together but not in the prosecution of a joint enterprise. It is not negligent to hunt where they are, and neither of them has reason to believe that the other will be negligent. Under the unreasonable belief that it is an animal, A shoots at a moving object that proves to be a man. B is not liable for A's negligent act. 13. A sells to B a second-hand gun, knowing that it is defective but reasonably believing that B, who also knows of the defect, will repair it before it is used. B, however, uses it without repairs and C is harmed by the resulting explosion. A is not liable to C. 14. A supplies B with wrecking tools, knowing that B is going to use them on a specific tract of land but having no reason to know that B is planning to burglarize a building on the land. A is not liable to C, the owner of the building burglarized by B through the use of the wrecking tools. Comment on Caveat:

f. On the liability for the escape of animals and for abnormally dangerous conduct for which there is strict liability, see 504-524. Liability in these cases is imposed, not on the ground that the conduct upon which it is based is wrongful, but on the ground that the conduct, although lawful because of the importance of the enterprise to the community, creates such great risk of harm to third persons that it is fair that the one conducting the enterprise should be required to compensate for the harm caused by it. Illustrations: 15. In a state in which there is strict liability for harm resulting from the intentional explosion of dynamite, A and B are employees of C and make preparations for exploding dynamite at a place where it is not negligent to do so. Debris from the explosion strikes D. The Institute takes no position on the liability of A and B. 16. A is B's servant, employed to feed and care for the animals in a menagerie. Without fault on A's part, one of the wild animals escapes and harms C. Assuming that A was not a possessor of the animal and hence not liable as such, the Institute takes no position on A's liability to C. PODIAS V. MAIRS Defendants were passengers in a car driven by their friend when their vehicle struck a motorcyclist. Defendants and the driver all exited their vehicle and looked around and made numerous cell phone calls, but none called for emergency assistance. Defendants continually told the driver not to get them involved. Eventually, all of them left the scene and the motorcyclist was struck by another vehicle and killed. Vicarious liability does not have to be based on acts of assistance but may rest on inaction, or on words of encouragement. Advice or encouragement to act operates as a moral support to a tortfeasor and if the act encouraged is known to be tortious it has the same effect upon the liability of the adviser as participation or physical assistance. Suggestive words that plant the seeds of or fuel negligent action may be enough to create joint liability. B. LIABILITY BY OPERATION OF LAW The court imposes liability on a party who is not negligent and is not directly responsible for the plaintiffs injury. For example, Employer/employee under Respondeat Superior. DONT limit analysis to this make sure party doesnt have separate COA C. INDIVISIBLE INJURY Joint and several liability occurs when the tortfeasors are jointly responsible for the indivisible injury and that the defendants must then work it out amongst themselves who has to pay what share. The plaintiff is free to collect the damages from any one or all of the tortfeasors, up to the entire amount of the damages. If the plaintiff can only collect from one of the defendants, then that defendant must try to collect from any other defendants. The injury is indivisible but each tortfeasor is responsible for the entire harm. AMERICAN MOTORCYCLE ASSN. V. SUPERIOR COURT OF LOS ANGELES COUNTY Defendant race sponsor filed a motion for leave to file a cross-complaint, seeking indemnity from the parents of plaintiff race participant and seeking a declaration of the "allocable negligence" of plaintiff's parents, who were not named as parties in an action brought by plaintiff to recover damages for injuries incurred in a motorcycle race. California's adoption of comparative negligence to ameliorate the inequitable consequences of the contributory negligence rule does not warrant the abolition or contraction of the established "joint and several liability" doctrine; each tortfeasor whose negligence is a proximate cause of an indivisible injury remains individually liable

for all compensable damages attributable to that injury. Joint and several liability does not logically conflict with a comparative negligence regime. A C are the 3 situations where a tortfeasor can be held responsible to pay all of s damages even though other tortfeasors are also responsible for the harm RESTATEMENT 14: APPORTIONMENT OF LIABILITY: A person who is liable to another based on a failure to protect the other from the specific risk of an intentional tort is jointly and severally liable for the share of comparative responsibility assigned to the intentional tortfeasor in addition to the share of comparative responsibility assigned to the person. COMMENTS & ILLUSTRATIONS: Comment: a. Scope. The rule in this Section applies only when a person is negligent because of the failure to take reasonable precautions to protect against the specific risk created by an intentional tortfeasor. Negligence (or strict liability) is determined in accordance with governing rules about tortious conduct, but this Section only applies if the risk that makes the tortfeasor negligent or strictly liable is the failure to take precautions against an intentional tort. When a person's unrelated tortious conduct and an intentional tortfeasor's acts concur to cause harm to another, the rules of joint and several liability provided in the applicable Track (A-E) govern. For reasons explained in Comment b, this Section is limited to instances in which the person is liable because of the risk of an intentional tort and does not extend to duties to protect against another's negligence. This Section does not determine when a party who fails to protect against the risk of an intentional tort is liable for failing to do so. Rather, the rule in this Section applies only when the governing law provides for such liability. Illustration: 1. A is a guest at a hotel operated by B. B neglects to provide adequate door locks on A's room, as a result of which C, an intruder, gains access to A's room, assaults A, and steals A's property. B is liable for the shares of comparative responsibility assigned both to B and to C, because the risk that made B's conduct negligent was specifically the risk that someone would assault A and steal A's property. C is jointly and severally liable for all of A's damages. See 12 (intentional tortfeasors). B's right to contribution from C is governed by 23, Comment e. C's right to contribution from B is governed by 23 and Comment l. The rule stated in this Section includes tortious acts of commission as well as omission. Thus, if in Illustration 1, B had provided adequate door locks, but negligently repaired the lock on A's room, thereby enabling C to gain access to A's room, B would also be liable for C's share of comparative responsibility. Illustrations: 2. A, a police officer, is injured when shot by B, a criminal that A was pursuing in a car chase. A is injured both by the bullet and by a sharp shard of defective glass in the windshield of A's police cruiser, manufactured by C. A was wearing a bullet-proof vest manufactured by D that, had it not been defective, would have prevented A's injuries. Assuming that the applicable rules of legal cause make B and D liable for the injury caused by the bullet and B, C, and D liable for the injury caused by the glass, D is liable for its share of comparative responsibility and B's share of comparative responsibility for the bullet damages and the glass damages, because D is liable for failing to protect against both the shooting and the risk of being assaulted with a broken bottle or other glass. This Section does not impose liability on D for C's share of comparative responsibility for the glass damages; D's joint and several or several liability with regard to C's comparative share of responsibility for the glass damages is determined by the applicable law on joint and several liability for independent tortfeasors in the jurisdiction (Tracks A-E). B is jointly and severally liable for the entirety of A's damages. See 12 (intentional tortfeasors). D's right to contribution from B is governed by 23, Comment e. B's right to contribution from D is governed by 23 and Comment l.

3. While going around a traffic rotary, A, driving a large, late-model automobile manufactured by C, refuses to yield so that B, driving another car, can enter the flow of traffic in the rotary. B, enraged at A's boorish (but nonnegligent) driving, follows A until B has the opportunity to ram her car into A's car. A's airbag, due to a defect, fails to inflate, causing A serious injury, which would not have occurred if the airbag had operated properly. C is not liable for B's share of comparative responsibility pursuant to this Section, because the risk that made C negligent was the general risk of an accident in which A might be injured without an airbag, not the specific risk of an intentional tort such as the one committed by B. B is jointly and severally liable for whatever damages B's tortious conduct legally caused. See 12 (intentional tortfeasors). Whether C is jointly and severally liable, severally liable, or some hybrid of the two depends on the law in the applicable jurisdiction. See A18E18 (liability of multiple tortfeasors for indivisible harm). 4. A, engaged in salvage operations, locates and raises to the ocean surface a ship that had sunk many years before. A contracts with B to tow the shipwreck into port later that day. The contract between A and B provides that any damage to the ship that occurs while B is performing its obligations will be borne by the parties according to applicable tort law. B neglects to tow the ship into dock, and, because it is sitting on the water overnight, C, a thief who pirates unoccupied ships, strips it of much of its valuable contents. If B is negligent for the delay in towing the ship only because of the risk that the ship would be harmed or resunk by adverse weather conditions, B is not liable for the share of comparative responsibility assigned to C based on the rule stated in this Section. B's joint and several liability is determined by the applicable law on joint and several liability for independent tortfeasors in the jurisdiction (Tracks A-E). If B is negligent for the delay in towing the ship because of the risk of thieves ransacking the ship (e.g., B was on notice that pirates were known to be lurking in the area looking for looting opportunities), B is liable for the share of comparative responsibility assigned to C based on the rule stated in this Section. B is liable for C's share regardless of whether B would be negligent in addition because of the risks posed by adverse weather conditions. B's right to contribution from C is governed by 23, Comment e. C's right to contribution from B is governed by 23 and Comment l. 5. A, while walking down a corridor in a shopping mall, slips on a banana peel that had not been cleaned up due to the negligence of a maintenance worker employed by the mall. The banana peel had been placed there by B, an adolescent, for the purpose of tripping a shopper. The shopping mall is not liable for the share of comparative responsibility of B under this Section. While there is some very small risk that a person will intentionally place an obstacle to cause another harm, the specific risk that made the shopping mall negligent was not protecting against such a risk but rather protecting against the more likely occurrence that a slippery condition would occur other than by the conduct of an intentional tortfeasor. The duty to protect against an intentional tort may derive from common-law tort principles as in the preceding Illustrations or from violations of statutes that impose obligations that courts then conclude are appropriate to be employed in a tort case. Illustration: 6. A, the estranged spouse of B, was severely injured when B assaulted A. A had previously obtained a restraining order against B because of prior acts of harassment by B and had complained to law-enforcement officials about continuing harassment by B that occurred after the restraining order was issued. A state statute provides that a law-enforcement official must arrest a person who violates a restraining order such as the one B violated, and courts have interpreted the statute as providing a standard of conduct appropriate for liability in a tort suit. Despite the statute, C, the law-enforcement official to whom A complained, took no action to arrest B before B's assault on A. In a suit by A against B and C, C is liable for the shares of comparative responsibility assigned to both B and C, because C is negligent based on the specific risk of an assault by B. B is jointly and severally liable for all of A's damages. See 12 (intentional tortfeasors). C's right to contribution from B is governed by 23, Comment e. B's right to contribution from C is governed by 23, Comment l.

b. Rationale. The modification of joint and several liability and the application of comparative responsibility to intentional tortfeasors create a difficult problem. When a person is injured by an intentional tort and another person negligently failed to protect against the risk of an intentional tort, the great culpability of the intentional tortfeasor may lead a factfinder to assign the bulk of responsibility for the harm to the intentional tortfeasor, who often will be insolvent. This would leave the person who negligently failed to protect the plaintiff with little liability and the injured plaintiff with little or no compensation for the harm. Yet when the risk of an intentional tort is the specific risk that required the negligent tortfeasor to protect the injured person, that result significantly diminishes the purpose for requiring a person to take precautions against this risk. A number of courts therefore have concluded that persons who negligently fail to protect against the specific risk of an intentional tort should bear the risk that the intentional tortfeasor is insolvent. The rule stated in this Section similarly makes such persons liable for the intentional tortfeasor's share of comparative responsibility. The negligent person may assert a contribution claim against the intentional tortfeasor, as provided in 23, Comment e. The method that most courts have employed to impose the risk of insolvency of an intentional tortfeasor on a person who negligently fails to protect the plaintiff is to refuse to permit assignment of a share of comparative responsibility to the intentional tortfeasor in a suit against the negligent party. While this method accomplishes the same goal, it is a less desirable means than the rule provided in this Section, which makes the negligent party liable for both that party's share as well as the intentional tortfeasor's share of comparative responsibility. Including intentional tortfeasors in the assignment of comparative responsibility provides a system for resolving all claims in a single proceeding, including any contribution or indemnity claims that may exist among negligent, strictly liable, and intentional tortfeasors. See 1, Comment c. Including intentional tortfeasors also permits appropriate crediting of the judgment in the case of partial settlements. See 16. In addition, coupling a rule permitting assignment of comparative responsibility to intentional tortfeasors with the rule provided in this Section avoids the unfairness of holding a party whose negligence is wholly unrelated to the intentional tortfeasor liable for the share of comparative responsibility assigned to the intentional tortfeasor. See Illustration 2. Because 12 (intentional tortfeasors) already provides that an intentional tortfeasor is jointly and severally liable for all harm legally caused by the intentional tort, this Section is unnecessary when the person with a duty to protect commits an intentional tort. Thus, if the law-enforcement officer in Illustration 6 took no action because of a grudge against A and a desire to see A harmed by B, 12 (intentional tortfeasors) would make the lawenforcement officer jointly and severally liable for the injury suffered by A, regardless of this Section. More significantly, in jurisdictions that retain full joint and several liability (Track A), the rule provided in this Section is unnecessary because joint and several liability applies to every tortfeasor who is a legal cause of the plaintiff's injury. However, in jurisdictions that have adopted some modification of joint and several liability (Tracks B-E), this Section makes the person who tortiously fails to protect against an intentional tort liable not only for that person's own share of comparative responsibility but also for the intentional tortfeasor's share of comparative responsibility. Arguably, the rule provided in this Section might logically apply even when a person negligently fails to protect another from risks created by a negligent tortfeasor. Thus, a person who negligently entrusts a handgun to a child, who then negligently shoots a third party, might be held liable for any share of comparative responsibility assigned to the child. However, the breadth of circumstances in which a person might be found negligent for failing to act based on the risk created by the foreseeable negligence of a third person is so great that a categorical extension of the rule stated in this Section could undermine the policies embodied in a jurisdiction's decision to abrogate joint and several liability.

When the third party's actions are merely negligent, rather than intentional, there are less likely to be great disparities between the share of comparative responsibility assigned to each party. When a jurisdiction has made a judgment to modify joint and several liability for unintentional tortfeasors who act independently, extending the rule provided in this Section to all those who negligently fail to protect against another's negligent conduct appears inconsistent with the basic judgment made by the jurisdiction that adopted several liability for the purpose of limiting each tortfeasor's liability to his or her comparative share of responsibility. Thus, while the rule stated in this Section is limited to a specific duty to protect against intentional torts, whether there are other limited circumstances in which the rule should be extended to those who fail to protect against nonintentional tortfeasors is a matter on which this Restatement takes no position. D. SATISFACTION
OF A

JUDGMENT

AND THE

AFTERMATH : CONTRIBUTION

AND

INDEMNITY

CONTRIBUTION: If 2 s are joint and severally liable, and one pays more than his pro rate share, he may usually obtain partial reimbursement from the other Generally, Defs must pay an equal share In comparative negligence states, duty of contribution is usually proportional to fault LIMITS No contribution when intentional tort Contribution must in fact be liable to the original RESTATEMENT 22: APPORTIONMENT OF LIABILITY: INDEMNITY: (a) When two or more persons are or may be liable for the same harm and one of them discharges the liability of another in whole or in part by settlement or discharge of judgment, the person discharging the liability is entitled to recover indemnity in the amount paid to the plaintiff, plus reasonable legal expenses, if: (1) the indemnitor has agreed by contract to indemnify the indemnitee, or (2) the indemnitee (i) was not liable except vicariously for the tort of the indemnitor, or (ii) was not liable except as a seller of a product supplied to the indemnitee by the indemnitor and the indemnitee was not independently culpable. (b) A person who is otherwise entitled to recover indemnity pursuant to contract may do so even if the party against whom indemnity is sought would not be liable to the plaintiff. COMMENTS & ILLUSTRATIONS: Comment: b. Extinguishing liability of the indemnitor. Except when a contract for indemnity provides otherwise, see Comment f, an indemnitee must extinguish the liability of the indemnitor to collect indemnity. The indemnitee may do so either by a settlement with the plaintiff that by its terms or by application of law discharges the indemnitor from liability or by satisfaction of judgment that by operation of law discharges the indemnitor from liability. See 16, Comment d (discharge of another tortfeasor's liability when both tortfeasors are treated as a single party and one settles); 24 (discharge of another tortfeasor's liability by settlement); 25 (discharge of another tortfeasor's liability by discharge of judgment). An indemnitee may, however, assert a claim for indemnity and obtain a contingent judgment in an action where the indemnitee is sued by the plaintiff as permitted by procedural rules, even though liability of the indemnitor has not yet been discharged. See Comment i. c. Indemnitor and indemnitee's liability to the plaintiff, including settlors seeking indemnity. Except as provided in Comment d and with contractual indemnity, an indemnitee must prove that the indemnitor would have been liable to the plaintiff in an amount equal to or greater than the amount the indemnitee seeks. An indemnitee may recover reasonable legal expenses defending the claim against the plaintiff, but not reasonable legal fees collecting

indemnity. An indemnitor is not protected from indemnity, however, by settlement or other voluntary discharge of liability by the plaintiff. A plaintiff's settlement with either party to a relationship supporting indemnity releases the other party to that relationship from liability to the plaintiff. In determining whether the indemnitor would have been liable to the plaintiff, the indemnitee has the burden to prove the indemnitor's liability to the plaintiff. The indemnitor has the burden to prove any defenses it would have had against the plaintiff. The effect of factfinding in earlier litigation between the plaintiff and indemnitee is determined by the applicable law of judgments. See Restatement Second, Judgments 50; see also Uniform Commercial Code 2-607(5). Illustrations: 1. A manufactures a product and sells it to B, who is a retailer. B then sells the product to C, who is injured. C settles with B for $ 100,000. B sues A for indemnity. B is unable to prove that the product was defective when it left A's control and, therefore, is unable to prove that A would have been liable to C. Absent contractual indemnity, B cannot recover indemnity from A. 2. A, who is B's employee acting within the scope of the employment, negligently injures C. C settles with B for $ 100,000. B sues A for indemnity. B proves that C suffered $ 80,000 damages and that A would have been liable to C. Absent contractual indemnity, B can recover $ 80,000 in indemnity from A, plus reasonable legal expenses defending C's claim. 3. A, who is B's employee acting within the scope of the employment, negligently injures C. B settles with C for $ 100,000. B sues A for indemnity. B proves that A would have been liable to C and that C suffered $ 100,000 in damages. A proves that C's own negligence was a legal cause of C's injuries. The factfinder assigns 60 percent responsibility to C. Under pure comparative responsibility, B is entitled to recover $ 40,000 in indemnity from A, plus reasonable legal expenses defending C's claim. Under modified comparative responsibility, B cannot recover any indemnity. See 7. A settlor seeking indemnity must prove that the indemnitor would have been liable to the plaintiff, but the settlor need not prove that the settlor would have been liable to the plaintiff. It is enough that the settlor made a reasonable compromise of liability. d. Statutes of limitation. A potential indemnitor is not protected from indemnity when the indemnitor would not have been liable to the plaintiff solely because of a statute of limitation. The limitation period on a claim for indemnity and its beginning are governed by appropriate law governing statutes of limitation. e. Culpability of the indemnitee. Except for contractual indemnity, a retailer may obtain indemnity from the manufacturer only if the retailer is not independently liable. See Restatement Third, Torts: Products Liability 1, 2; Uniform Commercial Code 2-314. Except for contractual indemnity, a vicariously liable person can obtain indemnity from the person whose negligence was imputed only if the vicariously liable person is not independently liable. An independently liable retailer or vicariously liable person may be entitled to contribution. See 23. Illustrations: 4. A manufactures a product and sells it to B, a retailer. B then sells it to C, a consumer who is injured. C sues B and recovers $ 100,000. B then sues A for indemnity. A proves that B was negligent for failing to notice the defect and refrain from selling the product. B is not entitled to indemnity. B may be entitled to contribution. See 23. 5. Same facts as Illustration 4, except that, instead of proving that B was negligent, A proves that B gave C an express warranty under which C could have recovered from B. B is not entitled to indemnity. B may be entitled to

contribution. See 23. 6. A, B's employee acting within the scope of the employment, negligently injures C. C sues B and recovers $ 100,000. B then sues A for indemnity. A proves that B was negligent for hiring A and that C could have recovered from B on that basis. B is not entitled to indemnity. B may be entitled to contribution. See 23. 7. A drives a bus on which B and C are passengers. B becomes rowdy and negligently injures C. C sues A for negligently failing to supervise his passengers and recovers $ 100,000. A sues B for indemnity, claiming that A's negligence was only failing to keep B from injuring C. A is not entitled to indemnity. A may be entitled to contribution. See 23. A more difficult case occurs when the negligence of a person seeking indemnity consists only in failing to prevent an intentional tortfeasor from injuring the plaintiff. In that case the negligent tortfeasor serves as a source of recovery even when the intentional tortfeasor is insolvent. See 14. That is similar to a vicariously liable employer serving as a source of liability when a negligent employee is insolvent. Unlike pure vicarious liability, however, a person whose negligence consists only in failing to prevent an intentional tortfeasor from injuring the plaintiff is still negligent. The policy of allocating a loss according to each person's share of responsibility supports having the negligent tortfeasor and the intentional tortfeasor, as between themselves, each bear their own comparative shares. That is accomplished by contribution, not indemnity. f. Contractual indemnity. Contractual indemnity is similar to exculpatory contracts and is governed by similar rules. See 2. Notwithstanding Comments b, c, and d, contractual indemnity is determined by the terms of the contract. An indemnitee can recover contractual indemnity for his or her own legally culpable conduct only if the contract is clear on that point. If the contract is otherwise clear, it need not contain specific words, such as "negligence" or "fault." Illustrations: 8. A builds a building and leases space to B. B agrees to indemnify A for liability for any injury on the premises "even if the injury is caused by defects in construction." C, a customer of B, is injured on the premises when he trips on a poorly constructed stair. C sues A and B for negligence and recovers $ 100,000. The factfinder assigns 40 percent responsibility to A and 60 percent responsibility to B. A can recover indemnity even though the contract does not use the word "negligence." 9. Same facts as Illustration 8, except that the contract provides for indemnity only for any injury caused "as a result of B's occupying the premises." A is not entitled to indemnity. The contract is not clear that A can recover indemnity even when A is negligent. 10. Same facts as Illustration 8, except that the agreement provides for indemnity "for any judgment or reasonable settlement for injury on the premises, including injury caused by defects in construction." A settles with C for $ 100,000. A then sues B for indemnity. A cannot prove that B would have been liable to C. A can prove that the settlement was reasonable but cannot prove that C's injuries were more than $ 90,000. A is entitled to indemnity for $ 100,000. The contract, not Comments b, c, and d, governs contractual indemnity. g. Other applicable law: insurance, guaranty, suretyship, and restitution. Contracts for indemnity come in many forms, such as contracts of insurance (with or without rights of subrogation), guaranty, or surety. Contracts of insurance, guaranty, or surety are governed by the applicable law of those areas, not by this Section. See, e.g., Restatement Third, Suretyship and Guaranty. Whether a person who provides a benefit to another can obtain restitution is determined by the applicable law of restitution, not by this Section. h. Economic loss. A person otherwise entitled to indemnity is not precluded from indemnity by the fact that he

suffered only economic loss. See Restatement Second, Torts 886B. i. Procedure. Indemnity may normally be recovered in a third-party claim in the suit where the indemnitee is sued by the plaintiff or in a separate lawsuit. That issue is governed by the appropriate procedural rules of each jurisdiction. E. SETTLEMENT AND RELEASE At common law settlement with one tortfeasor released all joint tortfeasors from liability. More recently legislatures have provided that a release of one tortfeasor does not release other joint tortfeasors unless the release specifically so provides. - Restatement 2nd bars contribution against the settling party who obtains release from the plaintiff. - The Uniform Comparative Fault Act does not discharge any other persons liable upon the same claim unless it so provides. However, the claim of the releasing person against other persons is reduced by the amount of the released persons equitable share of the obligation. - The Uniform Contribution Act of 1939 makes settling defendants must make contributions to non-settling defendants. - 1955 settling defendants do not have to make contributions to non-settling defendants1. If settles, he may generally obtain contribution from other potential s if he proves they are also liable to 2. Where 1 settles, and 2 against whom later gets a judgment - sues 1 for contribution, courts are split: TRADITIONAL RULE: The settling is liable for contribution Reduces 's incentive to settle because they may have extra liability in form of contribution to non-settling Reduction of 's claim rule 1979 Uniform Comparative Fault Act Courts deny contribution to non-settling s and Reduce the amount of 's claim against non-settlers to reflect earlier settlement Pro tanto reduction - Some courts reduce 's claim by the dollar amount Proportional reduction some courts reduce it by the proportion of the settling 's responsibility (ie reduce by 30% if settling 30% liable) F. WHERE THE ISSUE OF DIVISIBILITY OF DAMAGES IS UNCLEAR If damages can be reasonably apportioned between two tortfeasors, then the issue of apportionment should be left to the trier of fact. If the defendants can prove that the harm is divisible then it will be treated as divisible and the damages will be apportioned according to the division. If not it will be joint and several liability. MICHIE V. GREAT LAKES STEEL Defendants were several corporations that operated manufacturing plants in the United States, near the Canadian border. Plaintiffs were Canadian residents who filed a complaint in district court Although it is not always definitely so stated the rule seems to have become generally established that, although there is no concert of action between tortfeasors, if the cumulative effects of their acts is a single indivisible injury which it cannot certainly be said would have resulted but for the concurrence of such acts, the actors are to be held liable as joint tortfeasors. DILLON V. TWIN STATE GAS & ELECTRIC COMPANY What are the damages? If he died from the fall then he would only be damages for the shock. Explain the difference between the 1979 Act and the 1955 Act, an example.

1. Fact issue so jury determines indivisible v. divisible 2. If s caused divisible injuries then they are NOT jointly and severally liable 3. If s caused indivisible injuries, then they are jointly and severally liable because injury cannot be apportioned CHAPTER 7: OWNERS & OCCUPIERS OF LAND Focus on: Status, Duty, Facts supporting breach of duty When a trespasser or licensee is discovered in a position of peril, a landowner is required to use ordinary care to avoid injuring him. The duty to exercise ordinary care arises after the landowner knows, or from facts within his knowledge should know or believe, that a trespasser or licensee is on the land. SUTTON V. WHEELING & LAKE ERIE R.R. CO. Minor and his mother brought action against railroad for injuries minor received when, while walking along railroad tracks, he was pulled under passing railroad car. (1) minor was trespasser on railroad property; (2) duty owed by railroad to minor was duty to avoid willful, wanton or reckless conduct; (3) railroad's failure to prevent trespassers from routinely using railroad right-of-way as pathway was not willful, wanton, or reckless conduct; (4) moving railroad train was not attractive nuisance as would give minor walking on railroad right-of-way status of discovered trespasser; and (5) railroad did not breach duty of ordinary care it would owe to discovered trespasser. LICENSEE: A social guest, permission for short cuts, distribution of leaflets, etc. on land with permission, but there for own purpose. Duty to conduct activities on land in a reasonable manner and warn of dangers. INVITEES: 2 Categories: 1) persons who are invited on the land for a purpose connected with the business dealings of the possessor 2) persons who come on the land as a member of the public for a purpose for which the land is held open to the public Duty is one of full reasonable care Possessor of land is liable to invitees for harm caused by obvious dangers when possessor should anticipate that harm may befall them despite the obviousness of the danger. GLADON V. GREATER CLEVELAND REGIONAL TRANSIT AUTHORITY Gladon purchased a passenger ticket and boarded an RTA rapid transit train at Terminal Tower after attending a Cleveland Indians' night game with friends. During the baseball game, Gladon consumed about five 16-ounce beers. He left his friends at the stadium in search of a restroom, and ended up traveling alone on the RTA trains. The speed limit was set at the speed the train was going. This speed for going past a platform is meant to prevent such injuries from occurring. People fall from platforms. We see it on the news and the RTA is aware that it is possible. The status of an invitee is not absolute but is limited by the landowner's invitation. The visitor has the status of an invitee only while he is on part of the land to which his invitation extends, or in other words, the part of the land upon which the possessor gives him reason to believe that his presence is desired for the purpose for which he has come. If the invitee goes outside of the area of his invitation, he becomes a trespasser or a licensee, depending upon whether he goes there without the consent of the possessor, or with such consent. Willful conduct involves an intent, purpose, or design to injure. Wanton conduct involves the failure to exercise any care whatsoever toward those to whom he owes a duty of care, and his failure occurs under the circumstances in which there is great probability that harm will result. Speeding may be evidence of wanton conduct.

ROWLAND V. CHRISTIAN Plaintiff social guest appealed from summary judgment for defendant host in a personal injury action. Plaintiff alleged that defendant failed to warn him that her bathroom fixtures were cracked and dangerous. The fixtures severed tendons and nerves in plaintiff's hand. An exception to the general rule limiting liability has been recognized for cases where the occupier is aware of the dangerous condition, the condition amounts to a concealed trap, and the guest is unaware of the trap. A trap has been defined as a "concealed" danger, a danger with a deceptive appearance of safety. It has also been defined as something akin to a spring gun or steel trap. CARTER V. KINNEY The visitor signed up at church for a home Bible study. When he arrived for the study at the homeowners' residence, he slipped on a patch of ice and broke his leg. The possessor's intention in offering the invitation determines the status of the visitor and establishes the duty of care the possessor owes the visitor. Generally, the possessor owes a trespasser no duty of care, the possessor owes a licensee the duty to make safe dangers of which the possessor is aware, and the possessor owes invitees the duty to exercise reasonable care to protect them against both known dangers and those that would be revealed by inspection. TAYLOR V. OLSEN Plaintiff was injured when her car struck a tree that had fallen in the road during a storm. The tree had been standing on defendant's land before it fell. It is a question for the jury to decide whether defendant had taken reasonable care to inform himself of the condition of the tree, provided there was evidence that an inspection would have disclosed its hazardous condition. SARGENT V. ROSS The tenant daughter of the administratrix fell to her death from a steep stairway at a building owned by the landlord. The administratrix brought suit against the landlord for negligent construction and maintenance of the stairway. The jury returned a verdict in favor of the administratrix against the landlord. Landlords, as other persons, must exercise reasonable care not to subject others to an unreasonable risk of harm. A landlord must act as a reasonable person under all of the circumstances including the likelihood of injury to others, the probable seriousness of such injuries, and the burden of reducing or avoiding the risk. POSECAI V. WAL-MART STORES, INC. Plaintiff was robbed in defendant's parking lot. Plaintiff sued, contending that defendant was negligent in failing to provide adequate security in the parking lot considering the high level of crime in the surrounding area. The foreseeability of the crime risk on the defendant's property and the gravity of the risk determine the existence and the extent of the defendant's duty. The greater the foreseeability and gravity of the harm, the greater the duty of care that will be imposed on the business. A very high degree of foreseeability is required to give rise to a duty to post security guards, but a lower degree of foreseeability may support a duty to implement lesser security measures such as using surveillance cameras, installing improved lighting or fencing, or trimming shrubbery. The plaintiff has the burden of establishing the duty the defendant owed under the circumstances. DELGADO V. TRAX BAR & GRILL

Heightened foreseeability is satisfied by a showing of prior similar criminal incidents, or other indications of a reasonably foreseeable risk of violent criminal assaults in that location, and does not require a showing of prior nearly identical criminal incidents. CHAPTER 10: STRICT LIABILITY Liability that does not depend on actual negligence or intent to harm, but that is based on the breach of an absolute duty to make something safe. (for those who engage in certain activities do so at their own peril and must pay for any damage that foreseeably results even if the activity has been carried out in the most careful possible manner) A. POSSESSION OF ANIMALS In general courts impose strict liability on possessors of livestock and wild animals, but they hold possessors of domestic animals liable only if plaintiff proves that the defendant pet owner knew ahead of time that the animal was prone to violence. Majority Rule whether the owner knew or should have known (actual or constructive knowledge) the animal was dangerous or if the animal has abnormally dangerous propensities. Know Bard for what you should know for SL for domesticated animals, but negligence can be brought like in the dissent. You must either have ownership or control over the animal to be held strictly liable Common law is categorical: Must first determine what category the animal belongs in: 1. Livestock horses, sheep, goats 2. Wild Animals (except for zoo keepers, there must be negligence) possibly ferrets, or rabbits (depends if domesticated) 3. Domestic Animals cattle, dogs, cats 2 step process owner knew or should have known of the animals dangerous propensity. BARD V. JAHNKE The carpenter brought the injured party, who was a self-employed carpenter, to work in the farmer's barn. The bull was used for breeding purposes and was unrestrained inside the barn. It had never hurt anyone or showed signs of hostility before it attacked the injured party. The court affirmed the order, holding that defendants were not liable because the bull had never before shown an inclination to behave in a way that put others at risk of harm. The court refused to hold defendants subject to an enhanced duty because of any increased risk posed by bulls or breeding animals as a class. B. ABNORMALLY DANGEROUS ACTIVITIES When there is a natural activity, there must be negligence, but if the use is not natural, then the actor is strictly liable for damages example, if building a reservoir is not considered a natural activity due to arid conditions, then an actor may be strictly liable for any damage done by the flow of water that escapes. FLETCHER V. RYLANDS An actor is strictly liable for any and all damages when a person who for his own purposes brings on his lands and collects and keeps there anything likely to do mischief if it escapes. RYLANDS V. FLETCHER If the use is natural, then no strict liability - only if the use is unnatural, then strict liability natural is activity that most engage in TURNER V. BIG LAKE OIL

The oil company constructed large, artificial earthen ponds to store polluted water that was generated by its oil wells. On one occasion, water escaped from the ponds. It flowed onto the landowners' property and into the water sources the landowners used to water their livestock. Where water is collected in reservoirs, behind dams, in canals or in ditches, in the ordinary manner for the purpose of being used as a motive power, in navigation, in irrigation, in mining, or for any other convenient and lawful end, the person so collecting it can only be held liable on the ground of something unlawful in the manner in which he has built or maintained his structure, -- that is, on the principle of negligence. It follows, therefore, that if a dam breaks away, to the injury of property below, the owner will not be liable unless the person injured can show negligence; and if it appears in proof that the dam was well and properly built, upon a proper model, he will not be liable merely from the fact that it gave way; but otherwise, if it broke away in consequence of having been improperly constructed, or maintained in unsafe condition. ATLAS CHEMICAL INDUSTRIES V. ANDERSON Strict liability attaches whenever pollutants are intentionally discharged if you intentionally dump toxic waste, you are strictly liable for damages INDIANA HARBOR V. AMERICAN CYANAMID Defendant manufacturer of chemicals was sued by plaintiff switching line for cost of decontamination measures that resulted from railroad tank car leak. Plaintiff based its counts on theories of negligence and strict liability arising from an abnormally dangerous activity. The court below dismissed the negligence claim with prejudice and granted summary judgment on plaintiff's strict liability count. On appeal the court found that the strict liability regime did not apply as the leak was not caused by the inherent properties of the chemical; rather, the leak was found to arise from carelessness in transportation. Also, plaintiff failed to establish hazardous nature of activity. The court reasoned that accidents that are due to a lack of care can be prevented by taking care, and when a lack of care can be shown in court, such accidents are adequately deterred by the threat of liability for negligence. RESTATEMENT 520: ABNORMALLY DANGEROUS ACTIVITIES: In determining whether an activity is abnormally dangerous, the following factors are to be considered: (a) existence of a high degree of risk of some harm to the person, land or chattels of others; (b) likelihood that the harm that results from it will be great; (c) inability to eliminate the risk by the exercise of reasonable care; (d) extent to which the activity is not a matter of common usage; (e) inappropriateness of the activity to the place where it is carried on; and (f) extent to which its value to the community is outweighed by its dangerous attributes. COMMENTS & ILLUSTRATIONS: Comment: a. This Section deals only with the factors which determine whether an activity is abnormally dangerous. The general principle of strict liability for abnormally dangerous activities is stated in 519. The limitations upon strict liability for abnormally dangerous activities are stated in 521-524A. b. Distinguished from negligence. The rule stated in 519 is applicable to an activity that is carried on with all reasonable care, and that is of such utility that the risk which is involved in it cannot be regarded as so great or so unreasonable as to make it negligence merely to carry on the activity at all. (See 282). If the utility of the activity does not justify the risk it creates, it may be negligence merely to carry it on, and the rule stated in this Section is not then necessary to subject the defendant to liability for harm resulting from it. c. Relation to nuisance. If the abnormally dangerous activity involves a risk of harm to others that substantially

impairs the use and enjoyment of neighboring lands or interferes with rights common to all members of the public the impairment or interference may be actionable on the basis of a public or a private nuisance. (See 822, and Comment a under that Section). The rule of strict liability stated in 519 frequently is applied by many courts in these cases under the name of "absolute nuisance," even when the harm that results is physical harm to person, land or chattels. d. Purpose of activity. In the great majority of the cases that involve abnormally dangerous activities the activity is carried on by the actor for purposes in which he has a financial interest, such as a business conducted for profit. This, however, is not necessary for the existence of such an activity. The rule here stated is equally applicable when there is no pecuniary benefit to the actor. Thus a private owner of an abnormally dangerous body of water who keeps it only for his own use and pleasure as a swimming pool is subject to the same liability as one who operates a reservoir of water for profit. e. Not limited to the defendant's land. In most of the cases to which the rule of strict liability is applicable the abnormally dangerous activity is conducted on land in the possession of the defendant. This, again, is not necessary to the existence of such an activity. It may be carried on in a public highway or other public place or upon the land of another. f. "Abnormally dangerous." For an activity to be abnormally dangerous, not only must it create a danger of physical harm to others but the danger must be an abnormal one. In general, abnormal dangers arise from activities that are in themselves unusual, or from unusual risks created by more usual activities under particular circumstances. In determining whether the danger is abnormal, the factors listed in Clauses (a) to (f) of this Section are all to be considered, and are all of importance. Any one of them is not necessarily sufficient of itself in a particular case, and ordinarily several of them will be required for strict liability. On the other hand, it is not necessary that each of them be present, especially if others weigh heavily. Because of the interplay of these various factors, it is not possible to reduce abnormally dangerous activities to any definition. The essential question is whether the risk created is so unusual, either because of its magnitude or because of the circumstances surrounding it, as to justify the imposition of strict liability for the harm that results from it, even though it is carried on with all reasonable care. In other words, are its dangers and inappropriateness for the locality so great that, despite any usefulness it may have for the community, it should be required as a matter of law to pay for any harm it causes, without the need of a finding of negligence. Comment on Clauses (a) and (b): g. Risk of harm. An activity that is abnormally dangerous ordinarily involves a high degree of risk of serious harm to the person, land or chattels of others. The harm threatened must be major in degree, and sufficiently serious in its possible consequences to justify holding the defendant strictly responsible for subjecting others to an unusual risk. It is not enough that there is a recognizable risk of some relatively slight harm, even though that risk might be sufficient to make the actor's conduct negligent if the utility of his conduct did not outweigh it, or if he did not exercise reasonable care in conducting it. If the potential harm is sufficiently great, however, as in the case of a nuclear explosion, the likelihood that it will take place may be comparatively slight and yet the activity be regarded as abnormally dangerous. Some activities, such as the use of atomic energy, necessarily and inevitably involve major risks of harm to others, no matter how or where they are carried on. Others, such as the storage of explosives, necessarily involve major risks unless they are conducted in a remote place or to a very limited extent. Still others, such as the operation of a ten-ton traction engine on the public highway, which crushes conduits beneath it, involve such a risk only because of the place where they are carried on. In determining whether there is such a major risk, it may therefore be necessary to take into account the place where the activity is conducted, as to which see Comment j.

Comment on Clause (c): h. Risk not eliminated by reasonable care. Another important factor to be taken into account in determining whether the activity is abnormally dangerous is the impossibility of eliminating the risk by the exercise of reasonable care. Most ordinary activities can be made entirely safe by the taking of all reasonable precautions; and when safety cannot be attained by the exercise of due care there is reason to regard the danger as an abnormal one. There is probably no activity, unless it is perhaps the use of atomic energy, from which all risks of harm could not be eliminated by the taking of all conceivable precautions, and the exercise of the utmost care, particularly as to the place where it is carried on. Thus almost any other activity, no matter how dangerous, in the center of the Antarctic continent, might be expected to involve no possible risk to any one except those who engage in it. It is not necessary, for the factor stated in Clause (c) to apply, that the risk be one that no conceivable precautions or care could eliminate. What is referred to here is the unavoidable risk remaining in the activity, even though the actor has taken all reasonable precautions in advance and has exercised all reasonable care in his operation, so that he is not negligent. The utility of his conduct may be such that he is socially justified in proceeding with his activity, but the unavoidable risk of harm that is inherent in it requires that it be carried on at his peril, rather than at the expense of the innocent person who suffers harm as a result of it. Thus the manufacture in a city of certain explosives may involve a risk of detonation in spite of everything that the manufacturer may reasonably be expected to do; and although he may not be negligent in manufacturing them at all, he is subject to strict liability for an abnormally dangerous activity. A combination of the factors stated in Clauses (a), (b) and (c), or sometimes any one of them alone, is commonly expressed by saying that the activity is "ultrahazardous," or "extra-hazardous." Liability for abnormally dangerous activities is not, however, a matter of these three factors alone, and those stated in Clauses (d), (e), and (f) must still be taken into account. As to strict liability for ground damage resulting from aviation, see 520A. Comment on Clause (d): i. Common usage. An activity is a matter ofcommon usage if it is customarily carried on by the great mass of mankind or by many people in the community. It does not cease to be so because it is carried on for a purpose peculiar to the individual who engages in it. Certain activities, notwithstanding their recognizable danger, are so generally carried on as to be regarded as customary. Thus automobiles have come into such general use that their operation is a matter of common usage. This, notwithstanding the residue of unavoidable risk of serious harm that may result even from their careful operation, is sufficient to prevent their use from being regarded as an abnormally dangerous activity. On the other hand, the operation of a tank or any other motor vehicle of such size and weight as to be unusually difficult to control safely, or to be likely to damage the ground over which it is driven, is not yet a usual activity for many people, and therefore the operation of such a vehicle may be abnormally dangerous. Although blasting is recognized as a proper means of excavation for building purposes or of clearing woodland for cultivation, it is not carried on by any large percentage of the population, and therefore it is not a matter of common usage. Likewise the manufacture, storage, transportation and use of high explosives, although necessary to the construction of many public and private works, are carried on by only a comparatively small number of persons and therefore are not matters of common usage. So likewise, the very nature of oil lands and the essential interest of the public in the production of oil require that oil wells be drilled, but the dangers incident to the operation are characteristic of oil lands and not of lands in general, and relatively few persons are engaged in the activity.

The usual dangers resulting from an activity that is one of common usage are not regarded as abnormal, even though a serious risk of harm cannot be eliminated by all reasonable care. The difference is sometimes not so much one of the activity itself as of the manner in which it is carried on. Water collected in large quantity in a hillside reservoir in the midst of a city or in coal mining country is not the activity of any considerable portion of the population, and may therefore be regarded as abnormally dangerous; while water in a cistern or in household pipes or in a barnyard tank supplying cattle, although it may involve much the same danger of escape, differing only in degree if at all, still is a matter of common usage and therefore not abnormal. The same is true of gas and electricity in household pipes and wires, as contrasted with large gas storage tanks or high tension power lines. Fire in a fireplace or in an ordinary railway engine is a matter of common usage, while a traction engine shooting out sparks in its passage along the public highway is an abnormal danger. Comment on Clause (e): j. Locality. Another factor to be taken into account in determining whether an activity is abnormally dangerous is the place where it is carried on. If the place is one inappropriate to the particular activity, and other factors are present, the danger created may be regarded as an abnormal one. Even a magazine of high explosives, capable of destroying everything within a distance of half a mile, does not necessarily create an abnormal danger if it is located in the midst of a desert area, far from human habitation and all property of any considerable value. The same is true of a large storage tank filled with some highly inflammable liquid such as gasoline. Blasting, even with powerful high explosives, is not abnormally dangerous if it is done on an uninhabited mountainside, so far from anything of considerable value likely to be harmed that the risk if it does exist is not a serious one. On the other hand, the same magazine of explosives, the hugh storage tank full of gasoline or the blasting operations all become abnormally dangerous if they are carried on in the midst of a city. So likewise, the collection of large quantities of water in irrigation ditches or in a reservoir in open country usually is not a matter of any abnormal danger. On the other hand, if the reservoir is constructed in a coal mining area that is honeycombed with mine passages, or on a bluff overhanging a large city or if water is collected in an enormous standing tank above the same city, there is abnormal danger and strict liability when, without any negligence, the water escapes and does harm. In other words, the fact that the activity is inappropriate to the place where it is carried on is a factor of importance in determining whether the danger is an abnormal one. This is sometimes expressed, particularly in the English cases, by saying there is strict liability for a "non-natural" use of the defendant's land. There are some highly dangerous activities, that necessarily involve a risk of serious harm in spite of all possible care, that can be carried on only in a particular place. Coal mining must be done where there is coal; oil wells can be located only where there is oil; and a dam impounding water in a stream can be situated only in the bed of the stream. If these activities are of sufficient value to the community (see Comment k), they may not be regarded as abnormally dangerous when they are so located, since the only place where the activity can be carried on must necessarily be regarded as an appropriate one. Comment on Clause (f): k. Value to the community. Even though the activity involves a serious risk of harm that cannot be eliminated with reasonable care and it is not a matter of common usage, its value to the community may be such that the danger will not be regarded as an abnormal one. This is true particularly when the community is largely devoted to the dangerous enterprise and its prosperity largely depends upon it. Thus the interests of a particular town whose livelihood depends upon such an activity as manufacturing cement may be such that cement plants will be

regarded as a normal activity for that community notwithstanding the risk of serious harm from the emission of cement dust. There is an analogy here to the consideration of the same elements in determining the existence of a nuisance, under the rule stated in 831; and the Comments under that Section are applicable here, so far as they are pertinent. Thus in Texas and Oklahoma, a properly conducted oil or gas well, at least in a rural area, is not regarded as abnormally dangerous, while a different conclusion has been reached in Kansas and Indiana. California, whose oil industry is far from insignificant, has concluded that an oil well drilled in a thickly settled residential area in the city of Los Angeles is a matter of strict liability. In England, "a pluvial country, where constant streams and abundant rains make the storage of water unnecessary for ordinary or general purposes," a large reservoir in an inappropriate place has been found to be abnormally dangerous. In west Texas, a dry land whose livestock must have water, such a reservoir is regarded as "a natural and common use of the land." The same conclusion has been reached by many of the western states as to irrigation ditches. Comment: l. Function of court. Whether the activity is an abnormally dangerous one is to be determined by the court, upon consideration of all the factors listed in this Section, and the weight given to each that it merits upon the facts in evidence. In this it differs from questions of negligence. Whether the conduct of the defendant has been that of a reasonable man of ordinary prudence or in the alternative has been negligent is ordinarily an issue to be left to the jury. The standard of the hypothetical reasonable man is essentially a jury standard, in which the court interferes only in the clearest cases. A jury is fully competent to decide whether the defendant has properly driven his horse or operated his train or guarded his machinery or repaired his premises, or dug a hole. The imposition of strict liability, on the other hand, involves a characterization of the defendant's activity or enterprise itself, and a decision as to whether he is free to conduct it at all without becoming subject to liability for the harm that ensues even though he has used all reasonable care. This calls for a decision of the court; and it is no part of the province of the jury to decide whether an industrial enterprise upon which the community's prosperity might depend is located in the wrong place or whether such an activity as blasting is to be permitted without liability in the center of a large city. FOSTER V. PRESTON MILL Blasting operations conducted by defendant frightened mother mink owned by plaintiff and caused the mink to kill their kittens. Plaintiff brought an action to recover damages on the theory of absolute liability, and, in the alternative, on the theory of nuisance. The thing which makes blasting ultrahazardous is the risk that property or persons may be damaged or injured by coming into direct contact with flying debris, or by being directly affected by vibrations of the earth or concussions of the air, not that minks will kill their young.

CHAPTER 11. PRODUCTS LIABILITY Considered strict liability - legal theories underlying products liability are negligence and contract bases legal theories: 1) Negligence, 2) Strict Liability, 3) Express Warranty, 4) Misrepresentation, 5) Implied Warranty of Merchantability, 6) Implied Warranty of Fitness for Particular Purpose MANUFACTURING DEFECT

Something happened during the manufacturing process that caused a defect in the product must be defective at the time of the sale DESIGN DEFECT The design of the product is inherently defective ie. lack of shut off valve for water heaters when water gets too hot FAILURE TO WARN No or inadequate warnings about dangers of the product - strict liability must be a defect - absolute liability no defect CONSUMER EXPECTATION TEST RESTATEMENT 402A: SPECIAL LIABILITY OF SELLER OF PRODUCT FOR PHYSICAL HARM TO USER OR CONSUMER: (1) One who sells any product in a defective condition unreasonably dangerous to the user or consumer or to his property is subject to liability for physical harm thereby caused to the ultimate user or consumer, or to his property, if (a) the seller is engaged in the business of selling such a product, and (b) it is expected to and does reach the user or consumer without substantial change in the condition in which it is sold. (2) The rule stated in Subsection (1) applies although (a) the seller has exercised all possible care in the preparation and sale of his product, and (b) the user or consumer has not bought the product from or entered into any contractual relation with the seller. CAVEAT: Caveat: The Institute expresses no opinion as to whether the rules stated in this Section may not apply (1) to harm to persons other than users or consumers; (2) to the seller of a product expected to be processed or otherwise substantially changed before it reaches the user or consumer; or (3) to the seller of a component part of a product to be assembled. COMMENTS & ILLUSTRATIONS: Comment: a. This Section states a special rule applicable to sellers of products. The rule is one of strict liability, making the seller subject to liability to the user or consumer even though he has exercised all possible care in the preparation and sale of the product. The Section is inserted in the Chapter dealing with the negligence liability of suppliers of chattels, for convenience of reference and comparison with other Sections dealing with negligence. The rule stated here is not exclusive, and does not preclude liability based upon the alternative ground of negligence of the seller, where such negligence can be proved. b. History. Since the early days of the common law those engaged in the business of selling food intended for human consumption have been held to a high degree of responsibility for their products. As long ago as 1266 there were enacted special criminal statutes imposing penalties upon victualers, vintners, brewers, butchers, cooks, and other persons who supplied "corrupt" food and drink. In the earlier part of this century this ancient attitude was reflected in a series of decisions in which the courts of a number of states sought to find some method of holding the seller of food liable to the ultimate consumer even though there was no showing of negligence on the part of the seller. These decisions represented a departure from, and an exception to, the general rule that a supplier of chattels was not liable to third persons in the absence of negligence or privity of contract. In the

beginning, these decisions displayed considerable ingenuity in evolving more or less fictitious theories of liability to fit the case. The various devices included an agency of the intermediate dealer or another to purchase for the consumer, or to sell for the seller; a theoretical assignment of the seller's warranty to the intermediate dealer; a third party beneficiary contract; and an implied representation that the food was fit for consumption because it was placed on the market, as well as numerous others. In later years the courts have become more or less agreed upon the theory of a "warranty" from the seller to the consumer, either "running with the goods" by analogy to a covenant running with the land, or made directly to the consumer. Other decisions have indicated that the basis is merely one of strict liability in tort, which is not dependent upon either contract or negligence. Recent decisions, since 1950, have extended this special rule of strict liability beyond the seller of food for human consumption. The first extension was into the closely analogous cases of other products intended for intimate bodily use, where, for example, as in the case of cosmetics, the application to the body of the consumer is external rather than internal. Beginning in 1958 with a Michigan case involving cinder building blocks, a number of recent decisions have discarded any limitation to intimate association with the body, and have extended the rule of strict liability to cover the sale of any product which, if it should prove to be defective, may be expected to cause physical harm to the consumer or his property. c. On whatever theory, the justification for the strict liability has been said to be that the seller, by marketing his product for use and consumption, has undertaken and assumed a special responsibility toward any member of the consuming public who may be injured by it; that the public has the right to and does expect, in the case of products which it needs and for which it is forced to rely upon the seller, that reputable sellers will stand behind their goods; that public policy demands that the burden of accidental injuries caused by products intended for consumption be placed upon those who market them, and be treated as a cost of production against which liability insurance can be obtained; and that the consumer of such products is entitled to the maximum of protection at the hands of someone, and the proper persons to afford it are those who market the products. d. The rule stated in this Section is not limited to the sale of food for human consumption, or other products for intimate bodily use, although it will obviously include them. It extends to any product sold in the condition, or substantially the same condition, in which it is expected to reach the ultimate user or consumer. Thus the rule stated applies to an automobile, a tire, an airplane, a grinding wheel, a water heater, a gas stove, a power tool, a riveting machine, a chair, and an insecticide. It applies also to products which, if they are defective, may be expected to and do cause only "physical harm" in the form of damage to the user's land or chattels, as in the case of animal food or a herbicide. e. Normally the rule stated in this Section will be applied to articles which already have undergone some processing before sale, since there is today little in the way of consumer products which will reach the consumer without such processing. The rule is not, however, so limited, and the supplier of poisonous mushrooms which are neither cooked, canned, packaged, nor otherwise treated is subject to the liability here stated. f. Business of selling. The rule stated in this Section applies to any person engaged in the business of selling products for use or consumption. It therefore applies to any manufacturer of such a product, to any wholesale or retail dealer or distributor, and to the operator of a restaurant. It is not necessary that the seller be engaged solely in the business of selling such products. Thus the rule applies to the owner of a motion picture theatre who sells popcorn or ice cream, either for consumption on the premises or in packages to be taken home. The rule does not, however, apply to the occasional seller of food or other such products who is not engaged in that activity as a part of his business. Thus it does not apply to the housewife who, on one occasion, sells to her neighbor a jar of jam or a pound of sugar. Nor does it apply to the owner of an automobile who, on one occasion, sells it to his neighbor, or even sells it to a dealer in used cars, and this even though he is fully aware that the dealer plans to resell it. The basis for the rule is the ancient one of the special responsibility for the safety of the

public undertaken by one who enters into the business of supplying human beings with products which may endanger the safety of their persons and property, and the forced reliance upon that undertaking on the part of those who purchase such goods. This basis is lacking in the case of the ordinary individual who makes the isolated sale, and he is not liable to a third person, or even to his buyer, in the absence of his negligence. An analogy may be found in the provision of the Uniform Sales Act, 15, which limits the implied warranty of merchantable quality to sellers who deal in such goods; and in the similar limitation of the Uniform Commercial Code, 2-314, to a seller who is a merchant. This Section is also not intended to apply to sales of the stock of merchants out of the usual course of business, such as execution sales, bankruptcy sales, bulk sales, and the like. g. Defective condition. The rule stated in this Section applies only where the product is, at the time it leaves the seller's hands, in a condition not contemplated by the ultimate consumer, which will be unreasonably dangerous to him. The seller is not liable when he delivers the product in a safe condition, and subsequent mishandling or other causes make it harmful by the time it is consumed. The burden of proof that the product was in a defective condition at the time that it left the hands of the particular seller is upon the injured plaintiff; and unless evidence can be produced which will support the conclusion that it was then defective, the burden is not sustained. Safe condition at the time of delivery by the seller will, however, include proper packaging, necessary sterilization, and other precautions required to permit the product to remain safe for a normal length of time when handled in a normal manner. h. A product is not in a defective condition when it is safe for normal handling and consumption. If the injury results from abnormal handling, as where a bottled beverage is knocked against a radiator to remove the cap, or from abnormal preparation for use, as where too much salt is added to food, or from abnormal consumption, as where a child eats too much candy and is made ill, the seller is not liable. Where, however, he has reason to anticipate that danger may result from a particular use, as where a drug is sold which is safe only in limited doses, he may be required to give adequate warning of the danger (see Comment j), and a product sold without such warning is in a defective condition. The defective condition may arise not only from harmful ingredients, not characteristic of the product itself either as to presence or quantity, but also from foreign objects contained in the product, from decay or deterioration before sale, or from the way in which the product is prepared or packed. No reason is apparent for distinguishing between the product itself and the container in which it is supplied; and the two are purchased by the user or consumer as an integrated whole. Where the container is itself dangerous, the product is sold in a defective condition. Thus a carbonated beverage in a bottle which is so weak, or cracked, or jagged at the edges, or bottled under such excessive pressure that it may explode or otherwise cause harm to the person who handles it, is in a defective and dangerous condition. The container cannot logically be separated from the contents when the two are sold as a unit, and the liability stated in this Section arises not only when the consumer drinks the beverage and is poisoned by it, but also when he is injured by the bottle while he is handling it preparatory to consumption. i. Unreasonably dangerous. The rule stated in this Section applies only where the defective condition of the product makes it unreasonably dangerous to the user or consumer. Many products cannot possibly be made entirely safe for all consumption, and any food or drug necessarily involves some risk of harm, if only from overconsumption. Ordinary sugar is a deadly poison to diabetics, and castor oil found use under Mussolini as an instrument of torture. That is not what is meant by "unreasonably dangerous" in this Section. The article sold must be dangerous to an extent beyond that which would be contemplated by the ordinary consumer who purchases it, with the ordinary knowledge common to the community as to its characteristics. Good whiskey is not unreasonably dangerous merely because it will make some people drunk, and is especially dangerous to alcoholics; but bad whiskey, containing a dangerous amount of fusel oil, is unreasonably dangerous. Good tobacco is not unreasonably dangerous merely because the effects of smoking may be harmful; but tobacco containing something like marijuana may be unreasonably dangerous. Good butter is not unreasonably dangerous

merely because, if such be the case, it deposits cholesterol in the arteries and leads to heart attacks; but bad butter, contaminated with poisonous fish oil, is unreasonably dangerous. j. Directions or warning. In order to prevent the product from being unreasonably dangerous, the seller may be required to give directions or warning, on the container, as to its use. The seller may reasonably assume that those with common allergies, as for example to eggs or strawberries, will be aware of them, and he is not required to warn against them. Where, however, the product contains an ingredient to which a substantial number of the population are allergic, and the ingredient is one whose danger is not generally known, or if known is one which the consumer would reasonably not expect to find in the product, the seller is required to give warning against it, if he has knowledge, or by the application of reasonable, developed human skill and foresight should have knowledge, of the presence of the ingredient and the danger. Likewise in the case of poisonous drugs, or those unduly dangerous for other reasons, warning as to use may be required. But a seller is not required to warn with respect to products, or ingredients in them, which are only dangerous, or potentially so, when consumed in excessive quantity, or over a long period of time, when the danger, or potentiality of danger, is generally known and recognized. Again the dangers of alcoholic beverages are an example, as are also those of foods containing such substances as saturated fats, which may over a period of time have a deleterious effect upon the human heart. Where warning is given, the seller may reasonably assume that it will be read and heeded; and a product bearing such a warning, which is safe for use if it is followed, is not in defective condition, nor is it unreasonably dangerous. k. Unavoidably unsafe products. There are some products which, in the present state of human knowledge, are quite incapable of being made safe for their intended and ordinary use. These are especially common in the field of drugs. An outstanding example is the vaccine for the Pasteur treatment of rabies, which not uncommonly leads to very serious and damaging consequences when it is injected. Since the disease itself invariably leads to a dreadful death, both the marketing and the use of the vaccine are fully justified, notwithstanding the unavoidable high degree of risk which they involve. Such a product, properly prepared, and accompanied by proper directions and warning, is not defective, nor is it unreasonably dangerous. The same is true of many other drugs, vaccines, and the like, many of which for this very reason cannot legally be sold except to physicians, or under the prescription of a physician. It is also true in particular of many new or experimental drugs as to which, because of lack of time and opportunity for sufficient medical experience, there can be no assurance of safety, or perhaps even of purity of ingredients, but such experience as there is justifies the marketing and use of the drug notwithstanding a medically recognizable risk. The seller of such products, again with the qualification that they are properly prepared and marketed, and proper warning is given, where the situation calls for it, is not to be held to strict liability for unfortunate consequences attending their use, merely because he has undertaken to supply the public with an apparently useful and desirable product, attended with a known but apparently reasonable risk. l. User or consumer. In order for the rule stated in this Section to apply, it is not necessary that the ultimate user or consumer have acquired the product directly from the seller, although the rule applies equally if he does so. He may have acquired it through one or more intermediate dealers. It is not even necessary that the consumer have purchased the product at all. He may be a member of the family of the final purchaser, or his employee, or a guest at his table, or a mere donee from the purchaser. The liability stated is one in tort, and does not require any contractual relation, or privity of contract, between the plaintiff and the defendant. "Consumers" include not only those who in fact consume the product, but also those who prepare it for consumption; and the housewife who contracts tularemia while cooking rabbits for her husband is included within the rule stated in this Section, as is also the husband who is opening a bottle of beer for his wife to drink. Consumption includes all ultimate uses for which the product is intended, and the customer in a beauty shop to

whose hair a permanent wave solution is applied by the shop is a consumer. "User" includes those who are passively enjoying the benefit of the product, as in the case of passengers in automobiles or airplanes, as well as those who are utilizing it for the purpose of doing work upon it, as in the case of an employee of the ultimate buyer who is making repairs upon the automobile which he has purchased. Illustration: 1. A manufactures and packs a can of beans, which he sells to B, a wholesaler. B sells the beans to C, a jobber, who resells it to D, a retail grocer. E buys the can of beans from D, and gives it to F. F serves the beans at lunch to G, his guest. While eating the beans, G breaks a tooth, on a pebble of the size, shape, and color of a bean, which no reasonable inspection could possibly have discovered. There is satisfactory evidence that the pebble was in the can of beans when it was opened. Although there is no negligence on the part of A, B, C, or D, each of them is subject to liability to G. On the other hand E and F, who have not sold the beans, are not liable to G in the absence of some negligence on their part. m. "Warranty." The liability stated in this Section does not rest upon negligence. It is strict liability, similar in its nature to that covered by Chapters 20 and 21. The basis of liability is purely one of tort. A number of courts, seeking a theoretical basis for the liability, have resorted to a "warranty," either running with the goods sold, by analogy to covenants running with the land, or made directly to the consumer without contract. In some instances this theory has proved to be an unfortunate one. Although warranty was in its origin a matter of tort liability, and it is generally agreed that a tort action will still lie for its breach, it has become so identified in practice with a contract of sale between the plaintiff and the defendant that the warranty theory has become something of an obstacle to the recognition of the strict liability where there is no such contract. There is nothing in this Section which would prevent any court from treating the rule stated as a matter of "warranty" to the user or consumer. But if this is done, it should be recognized and understood that the "warranty" is a very different kind of warranty from those usually found in the sale of goods, and that it is not subject to the various contract rules which have grown up to surround such sales. The rule stated in this Section does not require any reliance on the part of the consumer upon the reputation, skill, or judgment of the seller who is to be held liable, nor any representation or undertaking on the part of that seller. The seller is strictly liable although, as is frequently the case, the consumer does not even know who he is at the time of consumption. The rule stated in this Section is not governed by the provisions of the Uniform Sales Act, or those of the Uniform Commercial Code, as to warranties; and it is not affected by limitations on the scope and content of warranties, or by limitation to "buyer" and "seller" in those statutes. Nor is the consumer required to give notice to the seller of his injury within a reasonable time after it occurs, as is provided by the Uniform Act. The consumer's cause of action does not depend upon the validity of his contract with the person from whom he acquires the product, and it is not affected by any disclaimer or other agreement, whether it be between the seller and his immediate buyer, or attached to and accompanying the product into the consumer's hands. In short, "warranty" must be given a new and different meaning if it is used in connection with this Section. It is much simpler to regard the liability here stated as merely one of strict liability in tort. n. Contributory negligence. Since the liability with which this Section deals is not based upon negligence of the seller, but is strict liability, the rule applied to strict liability cases (see 524) applies. Contributory negligence of the plaintiff is not a defense when such negligence consists merely in a failure to discover the defect in the product, or to guard against the possibility of its existence. On the other hand the form of contributory negligence which consists in voluntarily and unreasonably proceeding to encounter a known danger, and commonly passes under the name of assumption of risk, is a defense under this Section as in other cases of strict liability. If the user or consumer discovers the defect and is aware of the danger, and nevertheless proceeds unreasonably to make use of the product and is injured by it, he is barred from recovery.

Comment on Caveat: o. Injuries to non-users and non-consumers. Thus far the courts, in applying the rule stated in this Section, have not gone beyond allowing recovery to users and consumers, as those terms are defined in Comment l. Casual bystanders, and others who may come in contact with the product, as in the case of employees of the retailer, or a passer-by injured by an exploding bottle, or a pedestrian hit by an automobile, have been denied recovery. There may be no essential reason why such plaintiffs should not be brought within the scope of the protection afforded, other than that they do not have the same reasons for expecting such protection as the consumer who buys a marketed product; but the social pressure which has been largely responsible for the development of the rule stated has been a consumers' pressure, and there is not the same demand for the protection of casual strangers. The Institute expresses neither approval nor disapproval of expansion of the rule to permit recovery by such persons. p. Further processing or substantial change. Thus far the decisions applying the rule stated have not gone beyond products which are sold in the condition, or in substantially the same condition, in which they are expected to reach the hands of the ultimate user or consumer. In the absence of decisions providing a clue to the rules which are likely to develop, the Institute has refrained from taking any position as to the possible liability of the seller where the product is expected to, and does, undergo further processing or other substantial change after it leaves his hands and before it reaches those of the ultimate user or consumer. It seems reasonably clear that themere fact that the product is to undergo processing, or other substantial change, will not in all cases relieve the seller of liability under the rule stated in this Section. If, for example, raw coffee beans are sold to a buyer who roasts and packs them for sale to the ultimate consumer, it cannot be supposed that the seller will be relieved of all liability when the raw beans are contaminated with arsenic, or some other poison. Likewise the seller of an automobile with a defective steering gear which breaks and injures the driver, can scarcely expect to be relieved of the responsibility by reason of the fact that the car is sold to a dealer who is expected to "service" it, adjust the brakes, mount and inflate the tires, and the like, before it is ready for use. On the other hand, the manufacturer of pigiron, which is capable of a wide variety of uses, is not so likely to be held to strict liability when it turns out to be unsuitable for the child's tricycle into which it is finally made by a remote buyer. The question is essentially one of whether the responsibility for discovery and prevention of the dangerous defect is shifted to the intermediate party who is to make the changes. No doubt there will be some situations, and some defects, as to which the responsibility will be shifted, and others in which it will not. The existing decisions as yet throw no light upon the questions, and the Institute therefore expresses neither approval nor disapproval of the seller's strict liability in such a case. q. Component parts. The same problem arises in cases of the sale of a component part of a product to be assembled by another, as for example a tire to be placed on a new automobile, a brake cylinder for the same purpose, or an instrument for the panel of an airplane. Again the question arises, whether the responsibility is not shifted to the assembler. It is no doubt to be expected that where there is no change in the component part itself, but it is merely incorporated into something larger, the strict liability will be found to carry through to the ultimate user or consumer. But in the absence of a sufficient number of decisions on the matter to justify a conclusion, the Institute expresses no opinion on the matter. D. HARD WORK : DEFINING AND PROVING DEFECT Claims have now moved from cases about manufacturing defects and into cases involving products that are defectively designed or marketed with inadequate instructions or warnings. RESTATMENT 3D OF TORTS: PRODUCTS LIABILITY, 1: LIABILITY OF COMMERCIAL SELLER OR DISTRIBUTOR FOR HARM CAUSED BY DEFECTIVE PRODUCTS:

One engaged in the business of selling or otherwise distributing products who sells or distributes a defective product is subject to liability for harm to persons or property caused by the defect. COMMENTS & ILLUSTRATIONS: Comment: a. History. This Section states a general rule of tort liability applicable to commercial sellers and other distributors of products generally. Rules of liability applicable to special products such as prescription drugs and used products are set forth in separate Sections in Topic 2 of this Chapter. The liability established in this Section draws on both warranty law and tort law. Historically, the focus of products liability law was on manufacturing defects. A manufacturing defect is a physical departure from a product's intended design. See 2(a). Typically, manufacturing defects occur in only a small percentage of units in a product line. Courts early began imposing liability without fault on product sellers for harm caused by such defects, holding a seller liable for harm caused by manufacturing defects even though all possible care had been exercised by the seller in the preparation and distribution of the product. In doing so, courts relied on the concept of warranty, in connection with which fault has never been a prerequisite to liability. The imposition of liability for manufacturing defects has a long history in the common law. As early as 1266, criminal statutes imposed liability upon victualers, vintners, brewers, butchers, cooks, and other persons who supplied contaminated food and drink. In the late 1800s, courts in many states began imposing negligence and strict warranty liability on commercial sellers of defective goods. In the early 1960s, American courts began to recognize that a commercial seller of any product having a manufacturing defect should be liable in tort for harm caused by the defect regardless of the plaintiff's ability to maintain a traditional negligence or warranty action. Liability attached even if the manufacturer's quality control in producing the defective product was reasonable. A plaintiff was not required to be in direct privity with the defendant seller to bring an action. Strict liability in tort for defectively manufactured products merges the concept of implied warranty, in which negligence is not required, with the tort concept of negligence, in which contractual privity is not required. See 2(a). Questions of design defects and defects based on inadequate instructions or warnings arise when the specific product unit conforms to the intended design but the intended design itself, or its sale without adequate instructions or warnings, renders the product not reasonably safe. If these forms of defect are found to exist, then every unit in the same product line is potentially defective. See 2, Comments d, f, and i. Imposition of liability for design defects and for defects based on inadequate instructions or warnings was relatively infrequent until the late 1960s and early 1970s. A number of restrictive rules made recovery for such defects, especially design defects, difficult to obtain. As these rules eroded, courts sought to impose liability without fault for design defects and defects due to inadequate instructions or warnings under the general principles of 402A of the Restatement, Second, of Torts. However, it soon became evident that 402A, created to deal with liability for manufacturing defects, could not appropriately be applied to cases of design defects or defects based on inadequate instructions or warnings. A product unit that fails to meet the manufacturer's design specifications thereby fails to perform its intended function and is, almost by definition, defective. However, when the product unit meets the manufacturer's own design specifications, it is necessary to go outside those specifications to determine whether the product is defective. Sections 2(b) and 2(c) recognize that the rule developed for manufacturing defects is inappropriate for the resolution of claims of defective design and defects based on inadequate instructions or warnings. These latter categories of cases require determinations that the product could have reasonably been made safer by a better design or instruction or warning. Sections 2(b) and 2(c) rely on a reasonableness test traditionally used in determining whether an actor has been negligent. See Restatement, Second, Torts 291-293. Nevertheless, many courts insist on speaking of liability based on the standards described in 2(b) and 2(c) as being "strict." Several factors help to explain this rhetorical preference. First, in many design defect cases, if the product causes injury while being put to a reasonably foreseeable use, the seller is held to have known of the risks that foreseeably attend such use. See 2, Comment m. Second, some courts have sought to limit the defense of comparative fault in certain products liability contexts. In furtherance of this objective, they have avoided characterizing the liability test as based in negligence, thereby limiting the effect of comparative or contributory fault. See 17, Comment d. Third, some courts are concerned that a negligence standard might be too forgiving of

a small manufacturer who might be excused for its ignorance of risk or for failing to take adequate precautions to avoid risk. Negligence, which focuses on the conduct of the defendant-manufacturer, might allow a finding that a defendant with meager resources was not negligent because it was too burdensome for such a defendant to discover risks or to design or warn against them. The concept of strict liability, which focuses on the product rather than the conduct of the manufacturer, may help make the point that a defendant is held to the expert standard of knowledge available to the relevant manufacturing community at the time the product was manufactured. Finally, the liability of nonmanufacturing sellers in the distributive chain is strict. It is no defense that they acted reasonably and did not discover a defect in the product, be it from manufacturing, design, or failure to warn. See Comment e. Thus, "strict products liability" is a term of art that reflects the judgment that products liability is a discrete area of tort law which borrows from both negligence and warranty. It is not fully congruent with classical tort or contract law. Rather than perpetuating confusion spawned by existing doctrinal categories, 1 and 2 define the liability for each form of defect in terms directly addressing the various kinds of defects. As long as these functional criteria are met, courts may utilize the terminology of negligence, strict liability, or the implied warranty of merchantability, or simply define liability in the terms set forth in the black letter. See 2, Comment n. b. Sale or other distribution. The rule stated in this Section applies not only to sales transactions but also to other forms of commercial product distribution that are the functional equivalent of product sales. See 20. c. One engaged in the business of selling or otherwise distributing. The rule stated in this Section applies only to manufacturers and other commercial sellers and distributors who are engaged in the business of selling or otherwise distributing the type of product that harmed the plaintiff. The rule does not apply to a noncommercial seller or distributor of such products. Thus, it does not apply to one who sells foodstuffs to a neighbor, nor does it apply to the private owner of an automobile who sells it to another. It is not necessary that a commercial seller or distributor be engaged exclusively or even primarily in selling or otherwise distributing the type of product that injured the plaintiff, so long as the sale of the product is other than occasional or casual. Thus, the rule applies to a motion-picture theater's routine sales of popcorn or ice cream, either for consumption on the premises or in packages to be taken home. Similarly, a service station that does mechanical repair work on cars may also sell tires and automobile equipment as part of its regular business. Such sales are subject to the rule in this Section. However, the rule does not cover occasional sales (frequently referred to as "casual sales") outside the regular course of the seller's business. Thus, an occasional sale of surplus equipment by a business does not fall within the ambit of this rule. Whether a defendant is a commercial seller or distributor within the meaning of this Section is usually a question of law to be determined by the court. d. Harm to persons or property. The rule stated in this Section applies only to harm to persons or property, commonly referred to as personal injury and property damage. For rules governing economic loss, see 21. e. Nonmanufacturing sellers or other distributors of products. The rule stated in this Section provides that all commercial sellers and distributors of products, including nonmanufacturing sellers and distributors such as wholesalers and retailers, are subject to liability for selling products that are defective. Liability attaches even when such nonmanufacturing sellers or distributors do not themselves render the products defective and regardless of whether they are in a position to prevent defects from occurring. See 2, Comment o. Legislation has been enacted in many jurisdictions that, to some extent, immunizes nonmanufacturing sellers or distributors from strict liability. The legislation is premised on the belief that bringing nonmanufacturing sellers or distributors into products liability litigation generates wasteful legal costs. Although liability in most cases is ultimately passed on to the manufacturer who is responsible for creating the product defect, nonmanufacturing sellers or distributors must devote resources to protect their interests. In most situations, therefore, immunizing nonmanufacturers from strict liability saves those resources without jeopardizing the plaintiff's interests. To assure plaintiffs access to a responsible and solvent product seller or distributor, the statutes generally provide that the nonmanufacturing seller or distributor is immunized from strict liability only if: (1) the manufacturer is subject to the jurisdiction of the court of plaintiff's domicile; and (2) the manufacturer is not, nor is likely to become, insolvent. In connection with these statutes, two problems may need to be resolved to assure fairness to plaintiffs. First, as currently structured, the statutes typically impose upon the plaintiff the risk of insolvency of the manufacturer

between the time an action is brought and the time a judgment can be enforced. If a nonmanufacturing seller or distributor is dismissed from an action at the outset when it appears that the manufacturer will be able to pay a judgment, and the manufacturer subsequently becomes insolvent and is unable to pay the judgment, the plaintiff may be left to suffer the loss uncompensated. One possible solution could be to toll the statute of limitations against nonmanufacturers so that they may be brought in if necessary. Second, a nonmanufacturing seller or distributor occasionally will be responsible for the introduction of a defect in a product even though it exercised reasonable care in handling or supervising the product in its control. In such instances, liability for a 2(a) defect should be imposed on the nonmanufacturing seller or distributor. See 2, Illustration 2. RESTATEMENT 3D OF TORTS: PRODUCTS LIABILITY, 2: CATEGORIES OF PRODUCT DEFECT: A product is defective when, at the time of sale or distribution, it contains a manufacturing defect, is defective in design, or is defective because of inadequate instructions or warnings. A product: (a) contains a manufacturing defect when the product departs from its intended design even though all possible care was exercised in the preparation and marketing of the product; (b) is defective in design when the foreseeable risks of harm posed by the product could have been reduced or avoided by the adoption of a reasonable alternative design by the seller or other distributor, or a predecessor in the commercial chain of distribution, and the omission of the alternative design renders the product not reasonably safe; (c) is defective because of inadequate instructions or warnings when the foreseeable risks of harm posed by the product could have been reduced or avoided by the provision of reasonable instructions or warnings by the seller or other distributor, or a predecessor in the commercial chain of distribution, and the omission of the instructions or warnings renders the product not reasonably safe. COMMENTS & ILLUSTRATIONS: Comment: a. Rationale. The rules set forth in this Section establish separate standards of liability for manufacturing defects, design defects, and defects based on inadequate instructions or warnings. They are generally applicable to most products. Standards of liability applicable to special product categories such as prescription drugs and used products are set forth in separate sections in Topic 2 of this Chapter. The rule for manufacturing defects stated in Subsection (a) imposes liability whether or not the manufacturer's quality control efforts satisfy standards of reasonableness. Strict liability without fault in this context is generally believed to foster several objectives. On the premise that tort law serves the instrumental function of creating safety incentives, imposing strict liability on manufacturers for harm caused by manufacturing defects encourages greater investment in product safety than does a regime of fault-based liability under which, as a practical matter, sellers may escape their appropriate share of responsibility. Some courts and commentators also have said that strict liability discourages the consumption of defective products by causing the purchase price of products to reflect, more than would a rule of negligence, the costs of defects. And by eliminating the issue of manufacturer fault from plaintiff's case, strict liability reduces the transaction costs involved in litigating that issue. Several important fairness concerns are also believed to support manufacturers' liability for manufacturing defects even if the plaintiff is unable to show that the manufacturer's quality control fails to meet risk-utility norms. In many cases manufacturing defects are in fact caused by manufacturer negligence but plaintiffs have difficulty proving it. Strict liability therefore performs a function similar to the concept of res ipsa loquitur, allowing deserving plaintiffs to succeed notwithstanding what would otherwise be difficult or insuperable problems of proof. Products that malfunction due to manufacturing defects disappoint reasonable expectations of product performance. Because manufacturers invest in quality control at consciously chosen levels, their knowledge that apredictable number of flawed products will enter the marketplace entails an element of deliberation about the amount of injury that will result from their activity. Finally, many believe that consumers who benefit from products without suffering harm should share, through increases in the prices charged for those products, the burden of unavoidable injury costs that result from manufacturing defects. An often-cited rationale for holding wholesalers and retailers strictly liable for harm caused by manufacturing defects is that, as between them and innocent victims who suffer harm because of defective products, the product

sellers as business entities are in a better position than are individual users and consumers to insure against such losses. In most instances, wholesalers and retailers will be able to pass liability costs up the chain of product distribution to the manufacturer. When joining the manufacturer in the tort action presents the plaintiff with procedural difficulties, local retailers can pay damages to the victims and then seek indemnity from manufacturers. Finally, holding retailers and wholesalers strictly liable creates incentives for them to deal only with reputable, financially responsible manufacturers and distributors, thereby helping to protect the interests of users and consumers. For considerations relevant to reducing nonmanufacturers' liability, see 1, Comment e. In contrast to manufacturing defects, design defects and defects based on inadequate instructions or warnings are predicated on a different concept of responsibility. In the first place, such defects cannot be determined by reference to the manufacturer's own design or marketing standards because those standards are the very ones that plaintiffs attack as unreasonable. Some sort of independent assessment of advantages and disadvantages, to which some attach the label "risk-utility balancing," is necessary. Products are not generically defective merely because they are dangerous. Many product-related accident costs can be eliminated only by excessively sacrificing product features that make products useful and desirable. Thus, the various trade-offs need to be considered in determining whether accident costs are more fairly and efficiently borne by accident victims, on the one hand, or, on the other hand, by consumers generally through the mechanism of higher product prices attributable to liability costs imposed by courts on product sellers. Subsections (b) and (c), which impose liability for products that are defectively designed or sold without adequate warnings or instructions and are thus not reasonably safe, achieve the same general objectives as does liability predicated on negligence. The emphasis is on creating incentives for manufacturers to achieve optimal levels of safety in designing and marketing products. Society does not benefit from products that are excessively safe -- for example, automobiles designed with maximum speeds of 20 miles per hour -- any more than it benefits from products that are too risky. Society benefits most when the right, or optimal, amount of product safety is achieved. From a fairness perspective, requiring individual users and consumers to bear appropriate responsibility for proper product use prevents careless users and consumers from being subsidized by more careful users and consumers, when the former are paid damages out of funds to which the latter are forced to contribute through higher product prices. In general, the rationale for imposing strict liability on manufacturers for harm caused by manufacturing defects does not apply in the context of imposing liability for defective design and defects based on inadequate instruction or warning. Consumer expectations as to proper product design or warning are typically more difficult to discern than in the case of a manufacturing defect. Moreover, the element of deliberation in setting appropriate levels of design safety is not directly analogous to the setting of levels of quality control by the manufacturer. When a manufacturer sets its quality control at a certain level, it is aware that a given number of products may leave the assembly line in a defective condition and cause injury to innocent victims who can generally do nothing to avoid injury. The implications of deliberately drawing lines with respect to product design safety are different. A reasonably designed product still carries with it elements of risk that must be protected against by the user or consumer since some risks cannot be designed out of the product at reasonable cost. Most courts agree that, for the liability system to be fair and efficient, the balancing of risks and benefits in judging product design and marketing must be done in light of the knowledge of risks and risk-avoidance techniques reasonably attainable at the time of distribution. To hold a manufacturer liable for a risk that was not foreseeable when the product was marketed might foster increased manufacturer investment in safety. But such investment by definition would be a matter of guesswork. Furthermore, manufacturers may persuasively ask to be judged by a normative behavior standard to which it is reasonably possible for manufacturers to conform. For these reasons, Subsections (b) and (c) speak of products being defective only when risks are reasonably foreseeable. b. The nonexclusiveness of the definitions of defect in this Section. When a plaintiff seeks recovery under the general rule of liability in 1, in most instances the plaintiff must establish a prima facie case of product defect by satisfying the requirements of 2. Section 2 is not, however, the exclusive means by which the plaintiff may establish liability in a products case based on the general rule in 1. Some courts, for example, while recognizing that in most cases involving defective design the plaintiff must prove the availability of a reasonable alternative

design, also observe that such proof is not necessary in every case involving design defects. Sections 3 and 4 and Comment e to 2 provide approaches to the establishment of defective design other than that provided in 2(b). Section 3 provides that when circumstantial evidence supports the conclusion that a defect was a contributing cause of the harm and that the defect existed at the time of sale, it is unnecessary to identify the specific nature of the defect and meet the requisites of 2. Section 3 frees the plaintiff from the strictures of 2 in circumstances in which common experience teaches that an inference of defect may be warranted under the specific facts, including the failure of the product to perform its manifestly intended function. When the defect established under 3 may involve product design, some courts recognize consumer expectations as an adequate test for defect, in apparent conflict with the reasonable alternative design requirement in 2(b). But when the claims involve a product's failure to perform its manifestly intended function and the other requisites of 3 are met, the apparent conflict disappears. Section 4, dealing with violations of statutory and regulatory norms, also provides an alternate method of establishing defect. A plaintiff is not required to establish the standard for design or warning under 2, but merely to identify a government-imposed standard. Comment e provides a further qualification of the rule in 2(b). This Restatement recognizes the possibility that product sellers may be subject to liability even absent a reasonable alternative design when the product design is manifestly unreasonable. When 2(b) is read in conjunction with these other provisions that allow for other avenues for determining defective design, it reflects the substantial body of case law suggesting that reasonable alternative design is the predominant, yet not exclusive, method forestablishing defective design. c. Manufacturing defects. As stated in Subsection (a), a manufacturing defect is a departure from a product unit's design specifications. More distinctly than any other type of defect, manufacturing defects disappoint consumer expectations. Common examples of manufacturing defects are products that are physically flawed, damaged, or incorrectly assembled. In actions against the manufacturer, under prevailing rules concerning allocation of burdens of proof the plaintiff ordinarily bears the burden of establishing that such a defect existed in the product when it left the hands of the manufacturer. Occasionally a defect may arise after manufacture, for example, during shipment or while in storage. Since the product, as sold to the consumer, has a defect that is a departure from the product unit's design specifications, a commercial seller or distributor down the chain of distribution is liable as if the product were defectively manufactured. As long as the plaintiff establishes that the product was defective when it left the hands of a given seller in the distributive chain, liability will attach to that seller. Such defects are referred to in this Restatement as "manufacturing defects" even when they occur after manufacture. When the manufacturer delegates some aspect of manufacture, such as final assembly or inspection, to a subsequent seller, the manufacturer may be subject to liability under rules of vicarious liability for a defect that was introduced into the product after it left the hands of the manufacturer. Although Subsection (a) calls for liability without fault, a plaintiff may seek to recover based upon allegations and proof of negligent manufacture. See Comment n. For the rule governing food products that contain impurities or foreign matter, see 7. For the rule governing commercial used-product sellers' liability for harm caused by manufacturing defects, see 8. Illustrations: 1. Jack purchased a bottle of champagne from the BBB Liquor Mart. The champagne was bottled by AAA Inc., utilizing bottles manufactured by CCC Glass Co. While Jack was opening the bottle it suddenly exploded, causing disfiguring cuts to his face. The trier of fact determines that, originating with CCC, the bottle contained a manufacturing defect and that the defect caused the bottle to explode. AAA, BBB, and CCC are subject to liability even though they exercised reasonable care in the preparation and distribution of the defective bottle of AAA Champagne. The weakness in the glass structure in the bottle that caused Jack's harm was a departure from the product's intended design, subjecting each of the sellers in the distributive chain to strict liability for selling a defective product. 2. The same facts as Illustration 1, except that the trier of fact determines that the manufacturing defect in the bottle resulted from customer abuse of the bottle while it was on display at the BBB Liquor Mart prior to sale to Jack. BBB is subject to liability for selling a defective bottle of champagne to Jack even though it exercised

reasonable care in its marketing of the champagne. Neither CCC nor AAA is subject to liability if the trier of fact determines that the bottle was not defective when it left CCC's or AAA's control. d. Design defects: general considerations. Whereas a manufacturing defect consists of a product unit's failure to meet the manufacturer's design specifications, a product asserted to have a defective design meets the manufacturer's design specifications but raises the question whether the specifications themselves create unreasonable risks. Answering that question requires reference to a standard outside the specifications. Subsection (b) adopts a reasonableness ("riskutility balancing") test as the standard for judging the defectiveness of product designs. More specifically, the test is whether a reasonable alternative design would, at reasonable cost, have reduced the foreseeable risks of harm posed by the product and, if so, whether the omission of the alternative design by the seller or a predecessor in the distributive chain rendered the product not reasonably safe. (This is the primary, but not the exclusive, test for defective design. See Comment b.) Under prevailing rules concerning allocation of burden of proof, the plaintiff must prove that such a reasonable alternative was, or reasonably could have been, available at time of sale or distribution. See Comment f. Assessment of a product design in most instances requires a comparison between an alternative design and the product design that caused the injury, undertaken from the viewpoint of a reasonable person. That approach is also used in administering the traditional reasonableness standard in negligence. See Restatement, Second, Torts 283, Comment c. The policy reasons that support use of a reasonable-person perspective in connection with the general negligence standard also support its use in the products liability context. How the defendant's design compares with other, competing designs in actual use is relevant to the issue of whether the defendant's design is defective. Defendants often seek to defend their product designs on the ground that the designs conform to the "state of the art." The term "state of the art" has been variously defined to mean that the product design conforms to industry custom, that it reflects the safest and most advanced technology developed and in commercial use, or that it reflects technology at the cutting edge of scientific knowledge. The confusion brought about by these various definitions is unfortunate. This Section states that a design is defective if the product could have been made safer by the adoption of a reasonable alternative design. If such a design could have been practically adopted at time of sale and if the omission of such a design rendered the product not reasonably safe, the plaintiff establishes defect under Subsection (b). When a defendant demonstrates that its product design was the safest in use at the time of sale, it may be difficult for the plaintiff to prove that an alternative design could have been practically adopted. The defendant is thus allowed to introduce evidence with regard to industry practice that bears on whether an alternative design was practicable. Industry practice may also be relevant to whether the omission of an alternative design rendered the product not reasonably safe. While such evidence is admissible, it is not necessarily dispositive. If the plaintiff introduces expert testimony to establish that a reasonable alternative design could practically have been adopted, a trier of fact may conclude that the product was defective notwithstanding that such a design was not adopted by any manufacturer, or even considered for commercial use, at the time of sale. Early in the development of products liability law, courts held that a claim based on design defect could not be sustained if the dangers presented by the product were open and obvious. Subsection (b) does not recognize the obviousness of a design-related risk as precluding a finding of defectiveness. The fact that a danger is open and obvious is relevant to the issue of defectiveness, but does not necessarily preclude a plaintiff from establishing that a reasonable alternative design should have been adopted that would have reduced or prevented injury to the plaintiff. The requirement in Subsection (b) that the plaintiff show a reasonable alternative design applies in most instances even though the plaintiff alleges that the category of product sold by the defendant is so dangerous that it should not have been marketed at all. See Comment e. Common and widely distributed products such as alcoholic beverages, firearms, and above-ground swimming pools may be found to be defective only upon proof of the requisite conditions in Subsection (a), (b), or (c). If such products are defectively manufactured or sold without reasonable warnings as to their danger when such warnings are appropriate, or if reasonable alternative designs could have been adopted, then liability under 1 and 2 may attach. Absent proof of defect under those Sections, however, courts have not imposed liability for categories of products that are generally available and widely used and consumed, even if they pose substantial risks of harm. Instead, courts generally have concluded that

legislatures and administrative agencies can, more appropriately than courts, consider the desirability of commercial distribution of some categories of widely used and consumed, but nevertheless dangerous, products. Illustrations: 3. ABC Co. manufactured and sold a high-speed printing press to XYZ Printers, by whom Robert is employed. The press includes a circular plate cylinder that spins at a very high speed. On occasion, a foreign object, known in the trade as a "hickie," finds its way onto the plate of the unit, causing a blemish or imperfection on the printed page. To remove a hickie, it is customary practice for an employee to apply a piece of plastic to the printing plate while it is spinning. Robert performed this practice, known as "chasing the hickie," and while doing so suffered serious injuries to his hand. All employees, including Robert, knew that chasing the hickie was a dangerous procedure. Plaintiff's expert testifies that a safety-guard at the point of operation, which could have prevented Robert's injury, was both technologically and economically feasible and is utilized in similar machinery without causing difficulty. The fact that the danger is open and obvious does not bar the design claim against ABC. 4. XYZ Co. manufactures above-ground swimming pools that are four feet deep. Warnings are embossed on the outside of the pools in large letters stating "DANGER -- DO NOT DIVE -- SHALLOW WATER." In disregard of the warnings, Mary, age 21, dove head first into an XYZ pool and suffered serious injury. Expert testimony establishes that when Mary's outstretched hands hit the pool's slippery vinyl bottom her hands slid apart, causing her to strike her head against the bottom of the pool. For the purposes of this Illustration it is assumed that the warnings were adequate and that the only issue is whether the above-ground pool was defectively designed because the bottom was too slippery. All the expert witnesses agree that the vinyl pool liner that XYZ utilized was the best and safest liner available and that no alternative, less slippery liner was feasible. Mary has failed to establish defective design under Subsection (b). e. Design defects: possibility of manifestly unreasonable design. Several courts have suggested that the designs of some products are so manifestly unreasonable, inthat they have low social utility and high degree of danger, that liability should attach even absent proof of a reasonable alternative design. In large part the problem is one of how the range of relevant alternative designs is described. For example, a toy gun that shoots hard rubber pellets with sufficient velocity to cause injury to children could be found to be defectively designed within the rule of Subsection (b). Toy guns unlikely to cause injury would constitute reasonable alternatives to the dangerous toy. Thus, toy guns that project ping-pong balls, soft gelatin pellets, or water might be found to be reasonable alternative designs to a toy gun that shoots hard pellets. However, if the realism of the hard-pellet gun, and thus its capacity to cause injury, is sufficiently important to those who purchase and use such products to justify the court's limiting consideration to toy guns that achieve realism by shooting hard pellets, then no reasonable alternative will, by hypothesis, be available. In that instance, the design feature that defines which alternatives are relevant -- the realism of the hard-pellet gun and thus its capacity to injure -- is precisely the feature on which the user places value and of which the plaintiff complains. If a court were to adopt this characterization of the product, and deem the capacity to cause injury an egregiously unacceptable quality in a toy for use by children, it could conclude that liability should attach without proof of a reasonable alternative design. The court would declare the product design to be defective and not reasonably safe because the extremely high degree of danger posed by its use or consumption so substantially outweighs its negligible social utility that no rational, reasonable person, fully aware of the relevant facts, would choose to use, or to allow children to use, the product. Illustration: 5. ABC Co. manufactures novelty items. One item, an exploding cigar, is made to explode with a loud bang and the emission of smoke. Robert purchased the exploding cigar and presented it to his boss, Jack, at a birthday party arranged for him at the office. Jack lit the cigar. When it exploded, the heat from the explosion lit Jack's beard on fire causing serious burns to his face. If a court were to recognize the rule identified in this Comment, the finder of fact might find ABC liable for the defective design of the exploding cigar even if no reasonable alternative design was available that would provide similar prank characteristics. The utility of the exploding cigar is so low and the risk of injury is so high as to warrant a conclusion that the cigar is defective and should not have been marketed at all.

f. Design defects: factors relevant in determining whether the omission of a reasonable alternative design rendersa product not reasonably safe. Subsection (b) states that a product is defective in design if the omission of a reasonable alternative design renders the product not reasonably safe. A broad range of factors may be considered in determining whether an alternative design is reasonable and whether its omission renders a product not reasonably safe. The factors include, among others, the magnitude and probability of the foreseeable risks of harm, the instructions and warnings accompanying the product, and the nature and strength of consumer expectations regarding the product, including expectations arising from product portrayal and marketing. See Comment g. The relative advantages and disadvantages of the product as designed and as it alternatively could have been designed may also be considered. Thus, the likely effects of the alternative design on production costs; the effects of the alternative design on product longevity, maintenance, repair, and esthetics; and the range of consumer choice among products are factors that may be taken into account. A plaintiff is not necessarily required to introduce proof on all of these factors; their relevance, and the relevance of other factors, will vary from case to case. Moreover, the factors interact with one another. For example, evidence of the magnitude and probability of foreseeable harm may be offset by evidence that the proposed alternative design would reduce the efficiency and the utility of the product. On the other hand, evidence that a proposed alternative design would increase production costs may be offset by evidence that product portrayal and marketing created substantial expectations of performance or safety, thus increasing the probability of foreseeable harm. Depending on the mix of these factors, a number of variations in the design of a given product may meet the test in Subsection (b). On the other hand, it is not a factor under Subsection (b) that the imposition of liability would have a negative effect on corporate earnings or would reduce employment in a given industry. When evaluating the reasonableness of a design alternative, the overall safety of the product must be considered. It is not sufficient that the alternative design would have reduced or prevented the harm suffered by the plaintiff if it would also have introduced into the product other dangers of equal or greater magnitude. While a plaintiff must prove that a reasonable alternative design would have reduced the foreseeable risks of harm, Subsection (b) does not require the plaintiff to produce expert testimony in every case. Cases arise in which the feasibility of a reasonable alternative design is obvious and understandable to laypersons and therefore expert testimony is unnecessary to support a finding that the product should have been designed differently and more safely. For example, when a manufacturer sells a soft stuffed toy with hard plastic buttons that are easily removable and likely to choke and suffocate a small child who foreseeably attempts to swallow them, the plaintiff should be able to reach the trier of fact with a claim that buttons on such a toy should be an integral part of the toy's fabric itself (or otherwise be unremovable by an infant) without hiring an expert to demonstrate the feasibility of an alternative safer design. Furthermore, other products already available on the market may serve the same or very similar function at lower risk and at comparable cost. Such products may serve as reasonable alternatives to the product in question. In many cases, the plaintiff must rely on expert testimony. Subsection (b) does not, however, require the plaintiff to produce a prototype in order to make out a prima facie case. Thus, qualified expert testimony on the issue suffices, even though the expert has produced no prototype, if it reasonably supports the conclusion that a reasonable alternative design could have been practically adopted at the time of sale. The requirements in Subsection (b) relate to what the plaintiff must prove in order to prevail at trial. This Restatement takes no position regarding the requirements of local law concerning the adequacy of pleadings or pretrial demonstrations of genuine issues of fact. It does, however, assume that the plaintiff will have the opportunity to conduct reasonable discovery so as to ascertain whether an alternative design is practical. A test that considers such a broad range of factors in deciding whether the omission of an alternative design renders a product not reasonably safe requires a fair allocation of proof between the parties. To establish a prima facie case of defect, the plaintiff must prove the availability of a technologically feasible and practical alternative design that would have reduced or prevented the plaintiff's harm. Given inherent limitations on access to relevant data, the plaintiff is not required to establish with particularity the costs and benefits associated with adoption of the suggested alternative design. In sum, the requirement of Subsection (b) that a product is defective in design if the foreseeable risks of harm could have been reduced by a reasonable alternative design is based on the commonsense notion that liability for

harm caused by product designs should attach only when harm is reasonably preventable. For justice to be achieved, Subsection (b) should not be construed to create artificial and unreasonable barriers to recovery. The necessity of proving a reasonable alternative design as a predicate for establishing design defect is, like any factual element in a case, addressed initially to the courts. Sufficient evidence must be presented so that reasonable persons could conclude that a reasonable alternative could have been practically adopted. Assuming that a court concludes that sufficient evidence on this issue has been presented, the issue is then for the trier of fact. This Restatement takes no position regarding the specifics of how a jury should be instructed. So long as jury instructions are generally consistent with the rule of law set forth in Subsection (b), their specific form and content are matters of local law. Illustrations: 6. Andrea, age four, suffered serious burns when she got out of bed one night to go to the bathroom and tripped on the electric cord connected to a hot-water vaporizer. When Andrea tripped on the cord, the vaporizer separated into its three component parts -- a large, wide-mouthed glass jar, a metal pan, and a plastic top-heating unit. The plastic top-heating unit was not secured to the jar. When it came off, the hot water in the jar poured out, causing Andrea's burns. In an action alleging defective design of the vaporizer for not securing the top unit to the jar, the following factors are relevant: (1) the foreseeability that the vaporizer might be accidentally tipped over; (2) the overall safety provided by an alternative design that secures the heating unit to the receptacle holding the water; (3) consumer knowledge or lack thereof that the water in the glass jar is scalding hot; (4) the added cost of the safer alternative design; and (5) the relative convenience of a vaporizer with a lift-off cap. If Andrea offers expert testimony based on a reasonable analysis of these factors and such testimony is sufficient to allow reasonable persons to conclude that the omission of the alternative design renders the product not reasonably safe, the question of defectiveness is for the trier of fact. Whether instructions to the trier of fact should include specific reference to these factors is beyond the scope of this Restatement and should be determined under local law. 7. The same facts as Illustration 6, except that the safety feature that will secure the top-heating unit to the jar and prevent the water from spilling is a component that will add $ 5 to the cost of the vaporizer. Although the increase in cost to consumers is a relevant consideration, the impact of a finding of defectiveness on the general economy or on the profitability of the vaporizer manufacturer is not a factor to be considered in deciding whether the alternative safer design is reasonable. 8. XYZ Corp. designs and markets a new television antenna for home use that utilizes the electrical system in the buyer's home as the main signal receptor. The new antenna improves reception compared with traditional television antennas, but also introduces significant risks of electrical shock and electrocution. XYZ markets the new antenna with warnings about these risks, and emphasizes the benefits of improved reception to be derived from using the electrical wiring system. Fred buys one of the new antennas and installs it in his home. Two months later Fred suffers serious electrical burns while changing channels on his television set. Fred brings an action against XYZ, claiming that the design of the new television antenna is defective. At trial, Fred relies on the availability of traditional television antennas that are not connected with electrical systems to show that reasonable alternative designs could have been adopted by the manufacturer. In connection with proving that the defendant's design is defective under Subsection (b), Fred is not confined to offering variations of television antennas that rely on electrical wiring systems. The feature of XYZ's design that presents the risk of electrocution -- incorporating pre-existing electrical wiring systems -- is merely a means of achieving the objective of improved television reception. That feature does not, in itself, provide consumers with intrinsic benefits other than its effects on reception. Merely because the defendant chooses to emphasize that feature in its marketing does not prevent the plaintiff from proving that the increases in risk generated by that feature more than outweigh any increases in reception when compared with traditional antenna systems. 9. John was driving a compact automobile manufactured by the ABC Auto Company when he lost control and collided with a tree. John suffered serious injuries. John brings a products-liability claim against ABC, arguing that the design of his car is defective in that it does not offer the same level of crashworthiness as does a full-size automobile. John's experts admit that reducing the size of an automobile unavoidably increases the risk of injuries to occupants in collisions. John can identify no specific feature of the ABC automobile that could have been designed differently so as to be safer without increasing its size and substantially reducing its desirable

characteristics of lower cost and lower fuel economy. John has not established a defect within the meaning of Subsection (b). Although ABC's design is less safe than larger vehicles, the only way to make it safer, on John's own proof, is to make it larger, and the costs of doing that are unacceptably great. Moreover, eliminating smaller automobiles from the market would unduly restrict the range of consumer choice among automobile designs. Thus, the ABC design is not, by reason of its being smaller than other automobiles, "not reasonably safe." Given that the risks and benefits associated with relative automobile size are generally known, decisions regarding which sizes to purchase and use should be left to purchasers and users in the market. 10. ABC Armour Co. manufactures bullet-proof vests for use by police and security personnel. ABC offers several models, some providing front and back protection only, and others providing wrap-around protection. State A highway patrol officials chose to purchase the model that provides front and back protection only. They did so because that model is less expensive, allows greater flexibility of movement, and is more comfortable. Robert, a state trooper, was shot and killed while making a routine traffic arrest. The bullet entered the side of his body, where the vest did not provide protection. Robert's legal representative argues that the design of the vest is defective because it does not provide wrap-around coverage. Robert has not established a defect within the meaning of Subsection (b). The mere fact of the availability of a wrap-around vest does not render defective the vest that provides only front-and-back protection. Although the wrap-around design is somewhat safer, it is also more costly to buy and use. Moreover, the differences in advantages and disadvantages are sufficiently understood by consumers that omission of the wrap-around feature does not render the front-and-back design not reasonably safe. To subject sellers to liability based on that design would unduly restrict the range of consumer choice among products. g. Consumer expectations: general considerations. Under Subsection (b), consumer expectations do not constitute an independent standard for judging the defectiveness of product designs. Courts frequently rely, in part, on consumer expectations when discussing liability based on other theories of liability. Some courts, for example, use the term "reasonable consumer expectations" as an equivalent of "proof of a reasonable, safer design alternative," since reasonable consumers have a right to expect product designs that conform to the reasonableness standard in Subsection (b). Other courts, allowing an inference of defect to be drawn when the incident is of a kind that ordinarily would occur as a result of product defect, observe that products that fail when put to their manifestly intended use disappoint reasonable consumer expectations. See 3. However, consumer expectations do not play a determinative role in determining defectiveness. See Comment h. Consumer expectations, standing alone, do not take into account whether the proposed alternative design could be implemented at reasonable cost, or whether an alternative design would provide greater overall safety. Nevertheless, consumer expectations about product performance and the dangers attendant to product use affect how risks are perceived and relate to foreseeability and frequency of the risks of harm, both of which are relevant under Subsection (b). See Comment f. Such expectations are often influenced by how products are portrayed and marketed and can have a significant impact on consumer behavior. Thus, although consumer expectations do not constitute an independent standard for judging the defectiveness of product designs, they may substantially influence or even be ultimately determinative on risk-utility balancing in judging whether the omission of a proposed alternative design renders the product not reasonably safe. Subsection (b) likewise rejects conformance to consumer expectations as a defense. The mere fact that a risk presented by a product design is open and obvious, or generally known, and that the product thus satisfies expectations, does not prevent a finding that the design is defective. But the fact that a product design meets consumer expectations may substantially influence or even be ultimately determinative on risk-utility balancing in judging whether the omission of a proposed alternative design renders the product not reasonably safe. It follows that, while disappointment of consumer expectations may not serve as an independent basis for allowing recovery under Subsection (b), neither may conformance with consumer expectations serve as an independent basis for denying recovery. Such expectations may be relevant in both contexts, but in neither are they controlling. h. Consumer expectations: food products and used products. With regard to two special product categories consumer expectations play a special role in determining product defect. See 7 (food products) and 8 (used products). On occasion it is difficult to determine whether a given food component is an inherent aspect of a product or constitutes an adulteration of the product. Whether, for example, a fish bone in commercially

distributed fish chowder constitutes a manufacturing defect within the meaning of 2(a) is best determined by focusing on reasonable consumer expectations. Regarding commercially distributed used products, the rules set forth in 2 are not adequate to the task of determining liability. Variations in the type and condition of used products are such that the stringent rules for imposition of liability for new products are inappropriate. On occasion the seller of a used product may market the product in a manner that would cause a reasonable person in the position of the buyer to expect the used product to present no greater risk of defect than if it were new; or a used product may be remanufactured, justifying heightened seller's responsibility. In these limited settings it is appropriate to treat the sale under rules similar to those applicable to new products. See 8(b) and 8(c). i. Inadequate instructions or warnings. Commercial product sellers must provide reasonable instructions and warnings about risks of injury posed by products. Instructions inform persons how to use and consume products safely. Warnings alert users and consumers to the existence and nature of product risks so that they can prevent harm either by appropriate conduct during use or consumption or by choosing not to use or consume. In most instances the instructions and warnings will originate with the manufacturer, but sellers down the chain of distribution must warn when doing so is feasible and reasonably necessary. In any event, sellers down the chain are liable if the instructions and warnings provided by predecessors in the chain are inadequate. See Comment o. Under prevailing rules concerning allocation of burdens of proof, plaintiff must prove that adequate instructions or warnings were not provided. Subsection (c) adopts a reasonableness test for judging the adequacy of product instructions and warnings. It thus parallels Subsection (b), which adopts a similar standard for judging the safety of product designs. Although the liability standard is formulated in essentially identical terms in Subsections (b) and (c), the defectiveness concept is more difficult to apply in the warnings context. In evaluating the adequacy of product warnings and instructions, courts must be sensitive to many factors. It is impossible to identify anything approaching a perfect level of detail that should be communicated in product disclosures. For example, educated or experienced product users and consumers may benefit from inclusion of more information about the full spectrum of product risks, whereas less-educated or unskilled users may benefit from more concise warnings and instructions stressing only the most crucial risks and safe-handling practices. In some contexts, products intended for special categories of users, such as children, may require more vivid and unambiguous warnings. In some cases, excessive detail may detract from the ability of typical users and consumers to focus on the important aspects of the warnings, whereas in others reasonably full disclosure will be necessary to enable informed, efficient choices by product users. Product warnings and instructions can rarely communicate all potentially relevant information, and the ability of a plaintiff to imagine a hypothetical better warning in the aftermath of an accident does not establish that the warning actually accompanying the product was inadequate. No easy guideline exists for courts to adopt in assessing the adequacy of product warnings and instructions. In making their assessments, courts must focus on various factors, such as content and comprehensibility, intensity of expression, and the characteristics of expected user groups. Depending on the circumstances, Subsection (c) may require that instructions and warnings be given not only to purchasers, users, and consumers, but also to others who a reasonable seller should know will be in a position to reduce or avoid the risk of harm. There is no general rule as to whether one supplying a product for the use of others through an intermediary has a duty to warn the ultimate product user directly or may rely on the intermediary to relay warnings. The standard is one of reasonableness in the circumstances. Among the factors to be considered are the gravity of the risks posed by the product, the likelihood that the intermediary will convey the information to the ultimate user, and the feasibility and effectiveness of giving a warning directly to the user. Thus, when the purchaser of machinery is the owner of a workplace who provides the machinery to employees for their use, and there is reason to doubt that the employer will pass warnings on to employees, the seller is required to reach the employees directly with necessary instructions and warnings if doing so is reasonably feasible. In addition to alerting users and consumers to the existence and nature of product risks so that they can, by appropriate conduct during use or consumption, reduce the risk of harm, warnings also may be needed to inform users and consumers of nonobvious and not generally known risks that unavoidably inhere in using or consuming the product. Such warnings allow the user or consumer to avoid the risk warned against by making an informed decision not to purchase or use the product at all and hence not to encounter the risk. In this context, warnings

must be provided for inherent risks that reasonably foreseeable product users and consumers would reasonably deem material or significant in deciding whether to use or consume the product. Whether or not many persons would, when warned, nonetheless decide to use or consume the product, warnings are required to protect the interests of those reasonably foreseeable users or consumers who would, based on their own reasonable assessments of the risks and benefits, decline product use or consumption. When such warnings are necessary, their omission renders the product not reasonably safe at time of sale. Notwithstanding the defective condition of the product in the absence of adequate warnings, if a particular user or consumer would have decided to use or consume even if warned, the lack of warnings is not a legal cause of that plaintiff's harm. Judicial decisions supporting the duty to provide warnings for informed decisionmaking have arisen almost exclusively with regard to those toxic agents and pharmaceutical products with respect to which courts have recognized a distinctive need to provide risk information so that recipients of the information can decide whether they wish to purchase or utilize the product. See 6, Comment d. 11. ABC Adhesives Inc. manufactures a chemical adhesive for home use. Sandra purchased a gallon for use in laying tile in her kitchen. The label on the container warned in large letters that fumes from the adhesive were flammable and toxic, that the product should be used with adequate ventilation, and that all sources of fire should be extinguished. Sandra opened the windows in her kitchen, but did not extinguish the pilot light in her gas stove. When she had partly completed laying the tile, the pilot light suddenly ignited the fumes from the adhesive, causing Sandra serious burns. In an action against ABC, Sandra contends that the warnings were inadequate in failing specifically to state that gas-stove pilot lights should be extinguished. Whether the warning actually given was reasonable in the circumstances is to be decided by the trier of fact. j. Warnings: obvious and generally known risks. In general, a product seller is not subject to liability for failing to warn or instruct regarding risks and risk-avoidance measures that should be obvious to, or generally known by, foreseeable product users. When a risk is obvious or generally known, the prospective addressee of a warning will or should already know of its existence. Warning of an obvious or generally known risk in most instances will not provide an effective additional measure of safety. Furthermore, warnings that deal with obvious or generally known risks may be ignored by users and consumers and may diminish the significance of warnings about non-obvious, not-generally-known risks. Thus, requiring warnings of obvious or generally known risks could reduce the efficacy of warnings generally. When reasonable minds may differ as to whether the risk was obvious or generally known, the issue is to be decided by the trier of fact. The obviousness of risk may bear on the issue of design defect rather than failure to warn. See Comments d and g. Illustration: 12. XYZ Ladder Co. manufactures kitchen step ladders for home use. Sid used an XYZ ladder to post a sign above the door of his home office, unaware that his five-year-old son was playing in the office. While Sid was standing on the ladder, his son suddenly opened the door, which struck the ladder. Sid fell off the ladder and suffered a fractured hip. There were no warnings on the ladder, nor in the instruction booklet that came with it, not to use the ladder in front of an unlocked door. The danger should be obvious to foreseeable product users. No reasonable trier of fact would find XYZ liable for failing to warn about it and the court should rule for XYZ as a matter of law. k. Warnings: adverse allergic or idiosyncratic reactions. Cases of adverse allergic or idiosyncratic reactions involve a special subset of products that may be defective because of inadequate warnings. Many of these cases involve nonprescription drugs and cosmetics. However, virtually any tangible product can contain an ingredient to which some persons may be allergic. Thus, food, nonprescription drugs, toiletries, paint, solvents, building materials, clothing, and furniture have all been involved in litigation to which this Comment is relevant. Prescription drugs and medical devices are also capable of causing allergic reactions, but they are governed by 6. The general rule in cases involving allergic reactions is that a warning is required when the harm-causing ingredient is one to which a substantial number of persons are allergic. The degree of substantiality is not precisely quantifiable. Clearly the plaintiff in most cases must show that the allergic predisposition is not unique to the plaintiff. In determining whether the plaintiff has carried the burden in this regard, however, the court may properly consider the severity of the plaintiff's harm. The more severe the harm, the more justified is a conclusion

that the number of persons at risk need not be large to be considered "substantial" so as to require a warning. Essentially, this reflects the same risk-utility balancing undertaken in warnings cases generally. But courts explicitly impose the requirement of substantiality in cases involving adverse allergic reactions. The ingredient that causes the allergic reaction must be one whose danger or whose presence in the product is not generally known to consumers. When both the presence of an allergenic ingredient in the product and the risks presented by such ingredient are widely known, instructions and warnings about that danger are unnecessary. When the presence of the allergenic ingredient would not be anticipated by a reasonable user or consumer, warnings concerning its presence are required. Similarly, when the presence of the ingredient is generally known to consumers, but its dangers are not, a warning of the dangers must be given. Finally, as required in Subsection (c), warnings concerning risks of allergic reactions that are not reasonably foreseeable at the time of sale need not be provided. See Comment m. Illustration: 13. XYZ produces an over-the-counter nonprescription medicine containing aspirin, a well-known allergen to which a substantial minority of persons are sensitive. XYZ may reasonably assume that those who are allergic to aspirin are aware of their allergy or that, if they are not aware, warnings of possible allergic reactions would not be heeded. Thus, it is necessary to warn only of the fact that the medicine contains aspirin. l. Relationship between design and instruction or warning. Reasonable designs and instructions or warnings both play important roles in the production and distribution of reasonably safe products. In general, when a safer design can reasonably be implemented and risks can reasonably be designed out of a product, adoption of the safer design is required over a warning that leaves a significant residuum of such risks. For example, instructions and warnings may be ineffective because users of the product may not be adequately reached, may be likely to be inattentive, or may be insufficiently motivated to follow the instructions or heed the warnings. However, when an alternative design to avoid risks cannot reasonably be implemented, adequate instructions and warnings will normally be sufficient to render the product reasonably safe. Compare Comment e. Warnings are not, however, a substitute for the provision of a reasonably safe design. The fact that a risk is obvious or generally known often serves the same function as a warning. See Comment j. However, obviousness of risk does not necessarily obviate a duty to provide a safer design. Just as warnings may be ignored, so may obvious or generally known risks be ignored, leaving a residuum of risk great enough to require adopting a safer design. See Comment d. Illustration: 14. Jeremy's foot was severed when caught between the blade and compaction chamber of a garbage truck on which he was working. The injury occurred when he lost his balance while jumping on the back step of the garbage truck as it was moving from one stop to the next. The garbage truck, manufactured by XYZ Motor Co., has a warning in large red letters on both the left and right rear panels that reads "DANGER -- DO NOT INSERT ANY OBJECT WHILE COMPACTION CHAMBER IS WORKING -- KEEP HANDS AND FEET AWAY." The fact that adequate warning was given does not preclude Jeremy from seeking to establish a design defect under Subsection (b). The possibility that an employee might lose his balance and thus encounter the shear point was a risk that a warning could not eliminate and that might require a safety guard. Whether a design defect can be established is governed by Subsection (b). m. Reasonably foreseeable uses and risks in design and warning claims. Subsections (b) and (c) impose liability only when the product is put to uses that it is reasonable to expect a seller or distributor to foresee. Product sellers and distributors are not required to foresee and take precautions against every conceivable mode of use and abuse to which their products might be put. Increasing the costs of designing and marketing products in order to avoid the consequences of unreasonable modes of use is not required. In cases involving a claim of design defect in a mechanical product, foreseeability of risk is rarely an issue as a practical matter. Once the plaintiff establishes that the product was put to a reasonably foreseeable use, physical risks of injury are generally known or reasonably knowable by experts in the field. It is not unfair to charge a manufacturer with knowledge of such generally known or knowable risks. The issue of foreseeability of risk of harm is more complex in the case of products such as prescription drugs, medical devices, and toxic chemicals. Risks attendant to use and consumption of these products may, indeed, be

unforeseeable at the time of sale. Unforeseeable risks arising from foreseeable product use or consumption by definition cannot specifically be warned against. Thus, in connection with a claim of inadequate design, instruction, or warning, plaintiff should bear the burden of establishing that the risk in question was known or should have been known to the relevant manufacturing community. The harms that result from unforeseeable risks -- for example, in the human body's reaction to a new drug, medical device, or chemical -- are not a basis of liability. Of course, a seller bears responsibility to perform reasonable testing prior to marketing a product and to discover risks and risk-avoidance measures that such testing would reveal. A seller is charged with knowledge of what reasonable testing would reveal. If testing is not undertaken, or is performed in an inadequate manner, and this failure results in a defect that causes harm, the seller is subject to liability for harm caused by such defect. Illustration: 15. ABC Adhesives Inc. manufactures a chemical adhesive for use in laying ceramic tile. Recently it has become known that prolonged use of its ceramic adhesive over many years by diabetics can cause severe aggravation of the diabetic condition. Diabetics who have been using the ABC adhesive and have suffered serious aggravation of their condition bring an action against ABC for failing to warn about the risks of prolonged product use. However, it cannot be established that, at the time ABC's product was distributed, special risks to diabetics were reasonably foreseeable or that reasonable testing of the product would have led to the discovery of the risks. ABC is not liable since the risks attendant to such product use were not reasonably foreseeable. n. Relationship of definitions of defect to traditional doctrinal categories. The rules in this Section and in other provisions of this Chapter define the bases of tort liability for harm caused by product defects existing at time of sale or other distribution. The rules are stated functionally rather than in terms of traditional doctrinal categories. Claims based on product defect at time of sale or other distribution must meet the requisites set forth in Subsection (a), (b), or (c), or the other provisions in this Chapter. As long as these requisites are met, doctrinal tort categories such as negligence or strict liability may be utilized in bringing the claim. Similarly, a product defect claim satisfying the requisites of Subsection (a), (b), or (c), or other provisions in this Chapter, may be brought under the implied warranty of merchantability provisions of the Uniform Commercial Code. It is recognized that some courts have adopted a consumer expectations definition for design and failure-to-warn defects in implied warranty cases involving harm to persons or property. This Restatement contemplates that a well-coordinated body of law governing liability for harm to persons or property arising out of the sale of defective products requires a consistent definition of defect, and that the definition properly should come from tort law, whether the claim carries a tort label or one of implied warranty of merchantability. In connection with a claim under 1 and 2 and related provisions of this Restatement, the evidence that the defendant did or did not conduct adequately reasonable research or testing before marketing the product may be admissible (but is not necessarily required) regardless of whether the claim is based on negligence, strict liability, or implied warranty of merchantability. Although a defendant is held objectively responsible for having knowledge that a reasonable seller would have had, the fact that the defendant engaged in substantial research and testing may help to support the contention that a risk was not reasonably foreseeable. Conversely, the fact that the defendant engaged in little or no research or testing may, depending on the circumstances, help to support the contention that, had reasonable research or testing been performed, the risk could have been foreseen. Moreover, as long as the requisites in Subsection (a), (b), or (c), or other provisions in this Chapter, are met, the plaintiff may in appropriate instances -- for example, in connection with comparative fault or punitive damage claims -- show that the defect resulted from reckless, willfully indifferent, or intentionally wrongful conduct of the defendant. A separate and more difficult question arises as to whether a case should be submitted to a jury on multiple theories of recovery. Design and failure-to-warn claims may be combined in the same case because they rest on different factual allegations and distinct legal concepts. However, two or more factually identical defective-design claims or two or more factually identical failure-to-warn claims should not be submitted to the trier of fact in the same case under different doctrinal labels. Regardless of the doctrinal label attached to a particular claim, design and warning claims rest on a risk-utility assessment. To allow two or more factually identical risk-utility claims to go to a jury under different labels, whether "strict liability," "negligence," or "implied warranty of merchantability," would generate confusion and may well result in inconsistent verdicts.

In proceedings in which multiple theories are alleged, the Restatement leaves to local law the question of the procedural stage in a tort action at which plaintiff must decide under which theory to pursue the case. A different approach may be appropriate for claims based on manufacturing defects, since the rule set forth in Subsection (a) does not require risk-utility assessment while a negligence claim does. That is, the two types of manufacturing defect claims are based on different factual predicates. Negligence rests on a showing of fault leading to product defect. Strict liability rests merely on a showing of product defect. When a plaintiff believes a good claim for the negligent creation of (or failure to discover) a manufacturing defect may be established, the plaintiff may assert such a claim in addition to a claim in strict liability under Subsection (a). The plaintiff in such a case should have the opportunity to prove fault and also to assert the right to recover based on strict liability. However, clearly it would be inconsistent for a trier of fact to find no manufacturing defect on a 2(a) claim and yet return a verdict of liability because the defendant was negligent in having poor quality control. What must be shown under either theory is that the product in question did, in fact, have a manufacturing defect at time of sale that contributed to causing the plaintiff's harm. In connection with manufacturing defects, a 2(a) tort claim and an implied warranty of merchantability claim rest on the same factual predicate -- the sale by the defendant of a product that departs from the manufacturer's specifications irrespective of anyone's fault. Thus, these two claims are duplicative and may not be pursued together in the same case. The same analysis applies to claims against a nonmanufacturing supplier. The supplier can be held liable as the seller of a defective product under 2(a) or can be held liable under a negligence theory for failing reasonably to inspect a product or for negligently introducing a defect into the product. Since these claims are based on different factual predicates, the plaintiff may bring actions in both strict liability and negligence. Again, of course, recovery under either theory requires a finding of defect. The plaintiff in the nonmanufacturing-supplier case should, once again, not be free to submit a case to a jury based on both the implied warranty of merchantability and strict liability theories since they rest on the same factual base -- the sale by the supplier of a defective product regardless of fault. The theories are thus duplicative and do not constitute valid separate claims that may be given to the trier of fact in the same case. In all instances set forth above in which claims are duplicative, if one or the other theory presents an advantage to the plaintiff -- in connection with the statute of limitations, for example -- the plaintiff may pursue the more advantageous theory. But the trier of fact may not consider both theories on the same facts. Plaintiffs may, consistent with the foregoing principles, join claims based on product defect existing at time of sale or other distribution and claims based on theories of recovery that do not rest on a premise of product defect at time of sale. Claims based on misrepresentation, express warranty, and implied warranty of fitness for particular purpose, in particular, are not within the scope of this Chapter and thus are unaffected by it. Finally, negligence retains its vitality as an independent theory of recovery for a wide range of product-related, harm-causing behavior not involving defects at time of sale. This Restatement includes several such topics in later Chapters, including post-sale failure to warn (see 10); post-sale failure to recall (see 11); and a successor's liability for its own failure to warn (see 13). Other topics are covered in the Restatement, Second, of Torts. Thus, for example, negligent entrustment is treated in 390. Liability for negligent service, maintenance, or repair, or negligent overpromotion of a product, is governed by the rules set forth in 291 et seq. Illustrations: 16. A bottle of soda exploded in Bob's hand, causing harm. The soda was manufactured by LMN Bottlers, against whom Bob brings an action in tort. Bob's expert testifies that the bottle contained a manufacturing defect that caused the explosion. The expert also testifies that LMN's quality control efforts are below levels that reasonably prudent bottlers should maintain. Bob may rely on both strict liability and negligence theories. 17. Same facts as Illustration 6, but plaintiff presents no evidence that a reasonable alternative vaporizer design was available at the time of sale that would secure the top heating unit to the glass jar so that water would not escape when the vaporizer tipped over. Plaintiff nonetheless claims that the defendant is strictly liable because the product design caused her harm. Plaintiff has failed to establish an alternative design as required by Subsection (b). Her claim for recovery under a theory of strict liability thus fails.

18. Ray's automobile overturned while turning a corner, causing Ray to suffer harm. Ray brings an action against the manufacturer, alleging that the automobile was defectively unstable in design and also that the manufacturer should have warned of the automobile's instability in turning. The design and warning claims are independent. Ray may plead and prove either or both as grounds for recovery. 19. Same facts as Illustration 18, except that Ray asserts a claim for defectively unstable design alleged to be the result of negligence and a factually identical claim of defectively unstable design based on strict liability. A court should not allow Ray to submit both claims to the trier of fact. o. Liability of nonmanufacturing sellers for defective design and defects due to inadequate instructions or warnings. Nonmanufacturing sellers such as wholesalers and retailers often are not in a good position feasibly to adopt safer product designs or better instructions or warnings. Nevertheless, once it is determined that a reasonable alternative design or reasonable instructions or warnings could have been provided at or before the time of sale by a predecessor in the chain of distribution and would have reduced plaintiff's harm, it is no defense that a nonmanufacturing seller of such a product exercised due care. Thus, strict liability is imposed on a wholesale or retail seller who neither knew nor should have known of the relevant risks, nor was in a position to have taken action to avoid them, so long as a predecessor in the chain of distribution could have acted reasonably to avoid the risks. See Comment a. For exceptions to the general rule regarding the liability of a nonmanufacturer seller, see 1, Comment e. p. Misuse, modification, and alteration. Product misuse, modification, and alteration are forms of post-sale conduct by product users or others that can be relevant to the determination of the issues of defect, causation, or comparative responsibility. Whether such conduct affects one or more of the issues depends on the nature of the conduct and whether the manufacturer should have adopted a reasonable alternative design or provided a reasonable warning to protect against such conduct. Under the rule in Subsection (b), liability for defective design attaches only if the risks of harm related to foreseeable product use could have been reduced by the adoption of a reasonable alternative design. Similarly, under the rule in Subsection (c), liability for failure to instruct or warn attaches only if the risks presented by the product could have been reduced by the adoption of reasonable instructions or warnings. Foreseeable product misuse, alteration, and modification must also be considered in deciding whether an alternative design should have been adopted. The post-sale conduct of the user may be so unreasonable, unusual, and costly to avoid that a seller has no duty to design or warn against them. When a court so concludes, the product is not defective within the meaning of Subsection (b) or (c). A product may, however, be defective as defined in Subsection (b) or (c) due to the omission of a reasonable alternative design or the omission of an adequate warning, yet the risk that eventuates due to misuse, modification, or alteration raises questions whether the extent or scope of liability under the prevailing rules governing legal causation allow for the imposition of liability. See 15. Moreover, a product may be found to be defective and causally responsible for plaintiff's harm but the plaintiff may have misused, altered, or modified the product in a manner that calls for the reduction of plaintiff's recovery under the rules of comparative responsibility. Thus, an automobile may be defectively designed so as to provide inadequate protection against harm in the event of a collision, and the plaintiff's negligent modification of the automobile may have caused the collision eventuating in plaintiff's harm. See 17. It follows that misuse, modification, and alteration are not discrete legal issues. Rather, when relevant, they are aspects of the concepts of defect, causation, and plaintiff's fault. Jurisdictions differ on the question of who bears the burden of raising and introducing proof regarding conduct that constitutes misuse, modification, and alteration. The allocation of burdens in this regard is not addressed in this Restatement and is left to local law. Illustrations: 20. The ABC Chair Co. manufactures and sells oak chairs. The backs of the chairs have five horizontal wooden bars shaped to the contour of the human back. John, a college student, climbed up to the top bar of an ABC chair to reach the top shelf of a bookcase. The chair tipped and John fell, suffering serious harm. John brings an action against ABC, alleging that the chair should either have had the stability to support him when standing on the top bar or have had a differently designed back so that he could not use the bars for that purpose. The ABC

chair is not defectively designed. John's misuse of the product is so unreasonable that the risks it entails need not be designed against. q. Causation. Under 1, the product defect must have caused harm to the plaintiff. See 17 and 18. r. Warranty. Liability for harm caused by product defects imposed by the rules stated in this Chapter is tort liability, not liability for breach of warranty under the Uniform Commercial Code (U.C.C.). Courts may characterize claims under this Chapter as claims for breaches of the implied warranty of merchantability. But in cases involving defect-caused harm to persons or property, a well-coordinated body of law dealing with liability for such harm arising out of the sale of defective products would adopt the tort definition of product defect. See Comment n. 1. MANUFACTURING DEFECTS When a product departs from its intended design is defective it failed to perform in the way in which an ordinary consumer would expect WELGE V. PLANTERS Plaintiff consumer cut his hand and was left with permanent impairment when a jar of peanuts shattered as he was replacing its cap. He filed a products liability action against defendant corporations, the seller, the manufacturer of the product, and the manufacturer of the glass jar. Defendants filed a motion for summary judgment, which the trial court granted on the ground that the consumer had failed to exclude possible causes of the accident other than a defect introduced during the manufacturing process. The plaintiff in a products liability suit is not required to exclude every possibility, however fantastic or remote, that the defect which led to the accident was caused by someone other than one of the defendants. The doctrine of res ipsa loquitur teaches that an accident that is unlikely to occur unless the defendant was negligent is itself circumstantial evidence that the defendant was negligent. The doctrine is not strictly applicable to a products liability case because unlike an ordinary accident case the defendant in a products case has parted with possession and control of the harmful object before the accident occurs. But the doctrine merely instantiates the broader principle, which is as applicable to a products case as to any other tort case, that an accident can itself be evidence of liability. If it is the kind of accident that would not have occurred but for a defect in the product, and if it is reasonably plain that the defect was not introduced after the product was sold, the accident is evidence of the defect. RESTATEMENT 3D OF TORTS: PRODUCTS LIABILITY, 3: CIRCUMSTANTIAL EVIDENCE SUPPORTING INFERENCE OF PRODUCT DEFECT: It may be inferred that the harm sustained by the plaintiff was caused by a product defect existing at the time of sale or distribution, without proof of a specific defect, when the incident that harmed the plaintiff: (a) was of a kind that ordinarily occurs as a result of product defect; and (b) was not, in the particular case, solely the result of causes other than product defect existing at the time of sale or distribution. COMMENTS & ILLUSTRATIONS: Comment: a. History. This Section traces its historical antecedents to the law of negligence, which has long recognized that an inference of negligence may be drawn in cases where the defendant's negligence is the best explanation for the cause of an accident, even if the plaintiff cannot explain the exact nature of the defendant's conduct. See Restatement, Second, Torts 328D. As products liability law developed, cases arose in which an inference of product defect could be drawn from the incident in which a product caused plaintiff's harm, without proof of the specific nature of the defect. This Section sets forth the formal requisites for drawing such an inference. b. Requirement that the harm be of a kind that ordinarily occurs as a result of product defect. The most frequent application of this Section is to cases involving manufacturing defects. When a product unit contains such a

defect, and the defect affects product performance so as to cause a harmful incident, in most instances it will cause the product to malfunction in such a way that the inference of product defect is clear. From this perspective, manufacturing defects cause products to fail to perform their manifestly intended functions. Frequently, the plaintiff is able to establish specifically the nature and identity of the defect and may proceed directly under 2(a). But when the product unit involved in the harm-causing incident is lost or destroyed in the accident, direct evidence of specific defect may not be available. Under that circumstance, this Section may offer the plaintiff the only fair opportunity to recover. When examination of the product unit is impossible because the unit is lost or destroyed after the harm-causing incident, a somewhat different issue may be presented. Responsibility for spoliation of evidence may be relevant to the fairness of allowing the inference set forth in this Section. In any event, the issues of evidence spoliation and any sanctions that might be imposed for such conduct are beyond the scope of this Restatement Third, Torts: Products Liability. Although the rules in this Section, for the reasons just stated, most often apply to manufacturing defects, occasionally a product design causes the product to malfunction in a manner identical to that which would ordinarily be caused by a manufacturing defect. Thus, an aircraft may inadvertently be designed in such a way that, in new condition and while flying within its intended performance parameters, the wings suddenly and unexpectedly fall off, causing harm. In theory, of course, the plaintiff in such a case would be able to show how other units in the same production line were designed, leading to a showing of a reasonable alternative design under 2(b). As a practical matter, however, when the incident involving the aircraft is one that ordinarily occurs as a result of product defect, and evidence in the particular case establishes that the harm was not solely the result of causes other than product defect existing at time of sale, it should not be necessary for the plaintiff to incur the cost of proving whether the failure resulted from a manufacturing defect or from a defect in the design of the product. Section 3 allows the trier of fact to draw the inference that the product was defective whether due to a manufacturing defect or a design defect. Under those circumstances, the plaintiff need not specify the type of defect responsible for the product malfunction. It is important to emphasize the difference between a general inference of defect under 3 and claims of defect brought directly under 1 and 2. Section 3 claims are limited to situations in which a product fails to perform its manifestly intended function, thus supporting the conclusion that a defect of some kind is the most probable explanation. If that is not the case, and if no other provision of Chapter 1 allows the plaintiff to establish defect independently of the requirements in 2 (see 4 and Comment e to 2), a plaintiff is required to establish a cause of action for defect based on proof satisfying the requirements set forth in 2. See 2, Comment b. Illustrations: 1. John purchased a new electric blender. John used the blender approximately 10 times exclusively for making milkshakes. While he was making a milkshake, the blender suddenly shattered. A piece of glass struck John's eye, causing harm. The incident resulting in harm is of a kind that ordinarily occurs as a result of product defect. 2. Same facts as Illustration 1, except that John accidentally dropped the blender, causing the glass to shatter. The product did not fail to function in a manner supporting an inference of defect. Whether liability can be established depends on whether the plaintiff can prove a cause of action under 1 and 2. 3. Mary purchased a new automobile. She drove the car 1,000 miles without incident. One day she stopped the car at a red light and leaned back to rest until the light changed. Suddenly the seat collapsed backward, causing Mary to hit the accelerator and the car to shoot out into oncoming traffic and collide with another car. Mary suffered harm in the ensuing collision. As a result of the collision, Mary's car was set afire, destroying the seat assembly. The incident resulting in the harm is of a kind that ordinarily occurs as a result of product defect. Mary need not

establish whether the seat assembly contained a manufacturing defect or a design defect. 4. Same facts as in Illustration 3, except that the seat-back assembly failed when Mary, while stopped at the red light, was rear-ended by another automobile at 40 m.p.h. Mary cannot make out liability under this Section. The product did not fail to function in a manner supporting an inference of defect since the collapse of the seat is not the kind of incident that ordinarily occurs as a result of product defect. Liability must be established under the rules set forth in 1 and 2. 5. While carefully driving a new automobile at legal speed on a well-maintained road, Driver felt something crack below where the steering column connects with the dashboard. The steering wheel spun to the right and the automobile turned sharply. Before Driver could stop, the automobile crashed into a wall and Driver suffered harm. Driver has brought an action against the manufacturer of the automobile. The automobile had been driven on short trips before the accident and had 300 miles on its odometer. Driver's qualified expert witness testifies that in her opinion the accident was caused by a defect in the steering mechanism. The expert identifies four specific manufacturing and design defects that could have caused the accident, but was unable to say, on a balance of the probabilities, which of the four defects was the cause. Under this Section it is not necessary to identify the specific defect in order to draw the inference that a product defect caused the plaintiff's harm. c. No requirement that plaintiff prove what aspect of the product was defective. The inference of defect may be drawn under this Section without proof of the specific defect. Furthermore, quite apart from the question of what type of defect was involved, the plaintiff need not explain specifically what constituent part of the product failed. For example, if an inference of defect can be appropriately drawn in connection with the catastrophic failure of an airplane, the plaintiff need not establish whether the failure is attributable to fueltank explosion or engine malfunction. d. Requirement that the incident that harmed the plaintiff was not, in the particular case, solely the result of causes other than product defect existing at the time of sale. To allow the trier of fact to conclude that a product defect caused the plaintiff's harm under this Section, the plaintiff must establish by a preponderance of the evidence that the incident was not solely the result of causal factors other than defect at time of sale. The defect need not be the only cause of the incident; if the plaintiff can prove that the most likely explanation of the harm involves the causal contribution of a product defect, the fact that there may be other concurrent causes of the harm does not preclude liability under this Section. But when the harmful incident can be attributed solely to causes other than original defect, including the conduct of others, an inference of defect under this Section cannot be drawn. Evidence may permit the inference that a defect in the product at the time of the harm-causing incident caused the product to malfunction, but not the inference that the defect existed at the time of sale or distribution. Such factors as the age of the product, possible alteration by repairers or others, and misuse by the plaintiff or third parties may have introduced the defect that causes harm. Illustrations: 6. While driving a new automobile at high speed one night, Driver drove off the highway and crashed into a tree. Driver suffered harm. Driver cannot remember the circumstances surrounding the accident. Driver has brought an action against ABC Company, the manufacturer of the automobile. Driver presents no evidence of a specific defect. However, Driver's qualified expert presents credible testimony that a defect in the automobile must have caused the accident. ABC's qualified expert presents credible testimony that it is equally likely that, independent of any defect, Driver lost control while speeding on the highway. If the trier of fact believes the testimony of Driver's expert, then an inference of defect may be established under this Section. If, however, ABC's expert is believed, an inference of product defect may not be drawn under this Section because Driver has failed to establish by a preponderance of the evidence that the harm did not result solely from Driver's independent loss of

control at high speed. 7. Jack purchased a new ABC Electric Power Screwdriver. He inserted the bit for the appropriate screw size and turned the power button on. The bit shot out of the tool and lodged itself in Jack's arm, causing serious injury. Two weeks after purchasing the electric screwdriver, Jack believed the tool was making too much noise and brought it to the Acme Tool Repair Shop to check it out. Acme removed the mechanism that held the bit, examined it, and then reassembled it. Finding no problem, Acme returned the tool to Jack. The accident occurred the next day. On direct examination Jack's expert testifies that the accident was caused by a defect existing at time of sale. On cross-examination, however, Jack's expert admits it is equally probable that the problem with the tool was introduced by Acme. An inference that the power tool was defective at the time of sale cannot be drawn under this Section. Comment b. Requirement that the harm be of a kind that ordinarily occurs as a result of product defect, and Comment c. No requirement that plaintiff prove what aspect of the product was defective. 2. DESIGN DEFECT What do Prentis/Vautour and Potter tell us about Design Defect? - Prentis - Risk-utility and consumer expectation tests are used in design defect cases. - Vautour - Evidence of a reasonable alternative design is not required in strict liability cases applying a riskutility balancing test but can be used as a factor to be considered. - Potter This applies the consumer expectation test from manufacturing defect to a design defect case in conjunction with risk-utility balancing test in complex designs. PRENTIS V. YALE MANUFACTURING Appellees brought suit alleging liability of the manufacturer based on the defective design of a forklift. The competing factors to be weighed under a risk-utility balancing test invite the trier of fact to consider the alternatives and risks faced by the manufacturer and to determine whether in light of these the manufacturer exercised reasonable care in making the design choices it made, including a RAD. VAUTOUR V. BODY MASTERS The injured person was hurt using a leg press weight lifting exercise machine and claimed that the machine's safety stops were improperly designed or improperly placed on the machine. To prevail on a defective design products liability claim, a plaintiff must prove the following four elements: (1) the design of the product created a defective condition unreasonably dangerous to the user, (2) the condition existed when the product was sold by a seller in the business of selling such products, (3) the use of the product was reasonably foreseeable by the manufacturer, and (4) the condition caused injury to the user or the user's property. Under a risk-utility approach, a product is defective as designed if the magnitude of the danger outweighs the utility of the product. The risk-utility test requires a multifaceted balancing process involving evaluation of many conflicting factors. In order to determine whether the risks outweigh the benefits of the product design, a jury must evaluate many possible factors including the usefulness and desirability of the product to the public as a whole, whether the risk of danger could have been reduced without significantly affecting either the product's effectiveness or manufacturing cost, and the presence and efficacy of a warning to avoid an unreasonable risk of harm from hidden dangers or from foreseeable uses. Reasonableness, foreseeability, utility, and similar factors are questions of fact for the jury. RUB no RAD! Potter v. Chicago Pneumatic The court decided this on a simple design case basis the court is struggling to stay within strict liability and trying to avoid a negligence analysis so it creates a hybrid test to maintain the consumer expectations test hence the modified consumer expectations test

POTTER CAN USE BOTH 1. Consumer Expectation Test simple; OR - where a lay person can make the judgment as an ordinary consumer, but can have expert testimony 2. Modified Consumer Expectation Test complex - Risk-Utility and Consumer Expectation Test SOULE V. GENERAL MOTORS In a tort action for enhanced collision injuries allegedly caused by an uncrashworthy vehicle, plaintiff automobile owner contended that she was seriously injured in an accidental collision as a result of defects in her automobile caused by the negligence of defendant automobile manufacturer. A product's design is defective if it either violates the minimum safety expectations of an ordinary consumer or contains dangers which outweigh its benefits. Can use consumer expectation test when consumers know by their own experience about how safely a product should perform. ** see footnote on page 590 for types of cases that court says consumer expectation test is appropriate. Court intended to restrict consumer expectations test to the most obvious of scenarios. *** on exam - Fact pattern that lends itself to ex Toyota case where consumer expectation test OR risk utility balancing test. Know case names and tests. In case which follows Soule and judge says consumer expectation test is the test to be used and if so then def cannot defend with risk utility balancing. How do we know the difference between a complicated design and a simple design so you know whether to use risk utility balancing or consumer expectation test. SHOULD PRODUCT CATEGORIES BE DECLARED DEFECTIVE? OBRIEN V. MUSKIN CORP. Plaintiff was seriously injured when he slipped on the liner in a swimming pool manufactured by defendant. Plaintiff sued defendant in strict liability alleging defendant's product was defectively designed. Some factors relevant in risk-utility analysis are: (1) The usefulness and desirability of the product -- its utility to the user and to the public as a whole. (2) The safety aspects of the product -- the likelihood that it will cause injury, and the probable seriousness of the injury. (3) The availability of a substitute product which would meet the same need and not be as unsafe. (4) The manufacturer's ability to eliminate the unsafe character of the product without impairing its usefulness or making it too expensive to maintain its utility. (5) The user's ability to avoid danger by the exercise of care in the use of the product. (6) The user's anticipated awareness of the dangers inherent in the product and their avoidability, because of general public knowledge of the obvious condition of the product, or of the existence of suitable warnings or instructions. (7) The feasibility, on the part of the manufacturer, of spreading the loss by setting the price of the product or carrying liability insurance. ** effect of achieving the state of the art (best there is) = could still be liable. Even if you make it the best it can be and under Muskin you can still have a defective product. Because court will allow the jury to decide whether or not it should be made at all. 3. FAILURE TO WARN : Sometimes products are defective because they lack appropriate warnings. State of the art evidence is different in failure to warn than defective product. ANDERSON V. OWENS-CORNING FIBERGLASS CORP. Anderson is state of the art case in failure to warn. State of the art evidence in a failure to warn case goes to what was known at the time of the making of the product regarding potential danger. Plaintiff has to prove that def knew or should have known about the potential for danger (or that leading edge of science knew). In strict liability- failure to warn plaintiff has to show def had knowledge or should have known of danger and failed to warn.

In negligence failure to warn def knew or should have know and unreasonably failed to warn. (page 603 (middle paragraph) court discusses the difference between the two E. PROXIMATE CAUSE: UNION PUMP CO. V. ALLBRITTON Court in Texas says not proximate cause foreseeability not a part of it. (page 610) proximate cause v. producing cause. Tripp says ignore it is a disaster. The court uses the foreseeability test but pretends that they are not. Tripp says mark this opinion out it is not the law (maybe in Texas). Proximate cause do normal test - either foreseeability or scope of the risk test. Utilize normal proximate cause principles. F. COMPARATIVE FAULT : MURRAY V. FAIRBANKS MORSE Court called it comparative causation. Causation is a concept that is not amenable to such a division. Court applied this new rule in order to avoid negligence. Tripp said not to use this use normal comparative fault. **Tripp says we are not covering express warranty and misrepresentation. Hypo new car and drive it off the lot T gets drunk, is speeding in the rain and starts skidding toward the Waffle House, but hits the breaks before she actually hits and the brakes are defective and she hits the Waffle House they were defective, but would have stopped her before she hit if she had not been speeding (you can apportion fault, but you cannot apportion cause) Negligence did not give the tools for plaintiffs to figure out what went wrong when the product failed. It was virtually impossible to figure that out at such a remote place and time. Transfer the costs from the injury from the plaintiff to the person making profit from the item that caused the injury. Restatement 3rd: the plaintiff must show reasonable alternative design. CHAPTER 12. TRESPASS A. Interest Protected Traditional Trespass Nuisance Exclusive Right of Possession Use and Enjoyment of Property Unreasonable Invasion Sufficient Invasion Any Interference (tangible) Substantial and injunctive relief

Private inverse condemnation where the use of ones property essentially condemns a property if the use of the original property is beneficial see Boomer Cement B. TRESPASS IN SHADES OF GRAY What happens when the trespass is something that is intangible? BRADLEY V. AMERICAN SMELTING AND REFINING

Even though the manufacturer was in compliance with Federal standards and there is no federal statute that precludes a private nuisance claim.
The court holds that the elements to establish trespass for intangible matter: 1. an invasion affecting an interest in the exclusive possession of his property 2. an intentional doing of the act which results in the invasion 3. reasonable foreseeability that the act done could result in an invasion of plaintiffs possessory interest 4. actual and substantial damages to the res (thing) (none in this case) Most courts follow Bradley and recognize two types of trespass: 1) direct trespasses (tangible invasions) that are subject to the traditional rule that any interference with exclusive possession constitutes a trespass and 2) indirect trespasses (intangible invasions) arising from some form of pollution which require proof of actual harm.

C. PRIVATE NUISANCE To make a case for interference with the use and enjoyment of property, a plaintiff must establish 1) a basis for liability (either negligence, intentional tort or strict liability), 2) significant harm, and 3) an unreasonable invasion of the plaintiffs land. HUGHES V. EMERALD MINES CORP Appellee landowners brought suit against appellant mining corporation when both of appellees' water wells failed allegedly due to appellant's mining operations. Upheld verdict for P. Reduced amount. Based on: RESTATEMENT 2D OF TORTS, 822: PRIVATE NUISANCE: One is subject to liability for a private nuisance if, but only if, his conduct is a legal cause of an invasion of another's interest in the private use and enjoyment of land, and the invasion is either (a) intentional and unreasonable, or (b) unintentional and otherwise actionable under the rules controlling liability for negligent or reckless conduct, or for abnormally dangerous conditions or activities. COMMENTS & ILLUSTRATIONS: Comment: a. Types of conduct. Nuisance, either public or private, is a field, or rather two distinct fields, of tort liability. It is not a single type of tortious conduct. The feature that gives unity to either public or private nuisance is the interest invaded, namely either the public right or the private interest in the use and enjoyment of land. (See 821B and 821D). These interests may be invaded by any one of the types of conduct that serve in general as bases for all tort liability. The conduct that creates liability for public nuisance was determined in the first instance by the criminal law. In general, because of the traditional use of the one word "nuisance" to include both the public and the private invasion, the criminal law has tended to follow the rule stated in this Section, although numerous special statutes have led to departures from it. Subject to the exceptions created by these special statutes, which may be regarded as a legislative declaration of unreasonableness, the tort law of public nuisance is consistent with this Section. (See 821B). Private nuisance is solely a matter of tort liability. The interest in the private use and enjoyment of land may be invaded by more than one type of conduct. The invasion may be intentional and unreasonable. It may be unintentional but caused by negligent or reckless conduct; or it may result from an abnormally dangerous activity for which there is strict liability. On any of these bases the defendant may be liable. On the other hand, the invasion may be intentional but reasonable; or it may be entirely accidental and not fall within any of the categories mentioned above. In these cases there is no liability. b. Failure to recognize that private nuisance has reference to the interest invaded and not to the type of conduct that subjects the actor to liability has led to confusion. Thus, in respect to an interference with the use and

enjoyment of land, attempts are made to distinguish between private nuisance and negligence, overlooking the fact that private nuisance has reference to the interest invaded and negligence to the conduct that subjects the actor to liability for the invasion. Similar distinctions are attempted between private nuisance and abnormally dangerous activities for the same reason. This confusion is mainly due to a failure to notice in respect to private nuisance the change that has occurred in the basis of tort liability. In early tort law the rule of strict liability prevailed. An actor was liable for the harm caused by his acts whether that harm was done intentionally, negligently or accidentally. In course of time the law came to take into consideration not only the harm inflicted but also the type of conduct that caused it, in determining liability. This change came later in the law of private nuisance than in other fields. Private nuisance was remediable by an action on the case irrespective of the type of conduct involved. Thus the form of action did not call attention to the change from strict liability to liability based on conduct. But the change has occurred, and an actor is no longer liable for accidental interferences with the use and enjoyment of land but only for such interferences as are intentional and unreasonable or result from negligent, reckless or abnormally dangerous conduct. c. Liability for an invasion of interests in the use and enjoyment of land now depends upon the presence of some type of tortious conduct. Most invasions are intentional, and, with regard to the interest invaded, they are calledprivate nuisances. These invasions have no name derived from the type of conduct involved. This circumstance naturally led to thinking of private nuisance as itself a type of liability-forming conduct and to contrasting it with negligence. But as has been said, a negligent interference with the use and enjoyment of land is private nuisance in respect to the interest invaded and negligence in respect to the type of conduct that causes the invasion. Many interests other than those in the use and enjoyment of land may be invaded by negligent, reckless or abnormally dangerous conduct, and it is only when an interest in the use and enjoyment of land is invaded that an action for private nuisance and an action based on the type of conduct involved are actions for the same cause, and are not to be distinguished but identified. An invasion of a person's interest in the private use and enjoyment of land by any type of liability-forming conduct is private nuisance. The invasion that subjects a person to liability may be either intentional or unintentional. A person is subject to liability for an intentional invasion when his conduct is unreasonable under the circumstances of the particular case, and he is subject to liability for an unintentional invasion when his conduct is negligent, reckless or abnormally dangerous. These are the types of conduct that are stated in this Chapter as subjecting a person to liability for invasions of interests in the private use and enjoyment of land. d. Action for damages distinguished from suit for injunction. A potent cause of confusion as to the meaning and scope of private nuisance lies in the failure to distinguish the action at law from the suit for injunction in equity. Cases in equity are cited as precedents in actions at law without regard to their differences. In respect to the terms used, equity has not nicely observed the distinctions between the types of torts that were observed in cases at law under common law pleading. Equity looks at the fact situation, and if the case appears to be one for an injunction, that relief is given without much regard to the legal categories of tort liability. Thus it is unimportant for the purpose of injunction whether the conduct involved is a continuing trespass or a nuisance, and in granting the injunction it may be indifferently classified. Courts of equity have enjoined continuing intrusions on plaintiff's land as nuisances when the intrusions were clearly trespasses at law. This disregard of the legal classification adds to the confusion as to the meaning and scope of private nuisance, when the cases are used as precedents in an action at law without regard to the situation in the particular case. Furthermore, considerations enter into the determination of the right to an injunction that are inapplicable or have less weight in determining the right to damages. An injunction may be obtained in a proper case against a threatened private nuisance, but an action cannot be maintained at law unless harm already has been suffered. (See

821C for a similar distinction regarding a public nuisance). Even when there is present harm, it is one thing to say that a defendant should pay damages for the harm his factory is causing but it is a different thing to say that he must close his factory if the harm cannot be stopped. For the purpose of determining liability for damages for private nuisance, an invasion may be regarded as unreasonable even though the utility of the conduct is great and the amount of harm is relatively small. (See 826, Comment f). But for the purpose of determining whether the conduct producing the invasion should be enjoined, additional factors must be considered. It may be reasonable to continue an important activity if payment is made for the harm it is causing but unreasonable to initiate or continue it without paying. Thus denial of relief by way of injunction is not always a precedent for denial of relief by way of damages. Consequently, liability for damages should be treated independently, and this Chapter is primarily concerned with damages. Injunctions are the subject of Chapter 48, but some aspects of injunctions against nuisance are given treatment in this Chapter. e. Legal cause. "Legal cause" is that cause-effect relation between act or omission and consequence that is necessary in law before liability for the consequence can be imposed on the actor. The principles and problems involved in determining when conduct is the legal cause of harm are dealt with at length in the division of this Restatement dealing with liability for negligent conduct ( 430-453), and in the division dealing with liability for intentional harms ( 279-280) and are not restated in this Chapter. f. Conduct. On the type of conduct necessary to liability under the rule stated in this Section, see 824. Comment on Clause (a): g. Intentional invasions -- unreasonableness. Intentional invasions are defined in 825. The rules for determining when an intentional invasion is unreasonable are stated in 826-831. The special application of these rules to the case in which the defendant's conduct was solely for the purpose of causing the invasion is found in 829. Not every intentional and significant invasion of a person's interest in the use and enjoyment of land is actionable, even when he is the owner of the land in fee simple absolute and the conduct of the defendant is the sole and direct cause of the invasion. Life in organized society and especially in populous communities involves an unavoidable clash of individual interests. Practically all human activities unless carried on in a wilderness interfere to some extent with others or involve some risk of interference, and these interferences range from mere trifling annoyances to serious harms. It is an obvious truth that each individual in a community must put up with a certain amount of annoyance, inconvenience and interference and must take a certain amount of risk in order that all may get on together. The very existence of organized society depends upon the principle of "give and take, live and let live," and therefore the law of torts does not attempt to impose liability or shift the loss in every case in which one person's conduct has some detrimental effect on another. Liability for damages is imposed in those cases in which the harm or risk to one is greater than he ought to be required to bear under the circumstances, at least without compensation. (See 829A). In respect to unintentional invasions of another's interests, certain broad general principles of liability have been developed. These principles are embodied in the rules governing liability for negligent, reckless and abnormally dangerous conduct and apply to unintentional invasions of interests in the use and enjoyment of land as well as to other interests. In respect to intentional invasions of interests, however, there are no broad general principles of liability applicable to different types of interests. In respect to certain types of interests, such as those in bodily security and in the exclusive possession of land, the law has developed strict rules of liability for intentional invasions, qualified by specific privileges. In respect to interests in the use and enjoyment of land, however, the law has developed a broader, more indefinite and more comprehensive, rule of liability for intentional invasions. This rule is expressed in terms of unreasonableness and, as stated in this Section, requires that an intentional invasion be unreasonable before one is liable for causing it.

Comment on Clause (b): h. Otherwise actionable. The term "otherwise actionable" is used to indicate that in addition to the requirements listed earlier in this Section, the other elements of a cause of action for harm caused by reckless, negligent or abnormally dangerous conduct, stated in other parts of this Restatement, must be present in order to establish liability when the invasion of a person's interest in the use and enjoyment of land is unintentional. i. Negligence and recklessness. Interests in the use and enjoyment of land, like interests in bodily security and in the exclusive possession of land, are accorded legal protection against unintentional invasions. But like those interests they are not protected against all invasions. There is no general rule of law that one acts at his peril in respect to interferences with another's use or enjoyment of his land, and therefore when the interferences are purely accidental the actor is not liable for causing them. It is only when unintentional invasions are caused by another's conduct that is negligent, reckless or abnormally dangerous that the law subjects the actor to liability. The rules determining when conduct is negligent are stated in 281-328D. The rules determining when conduct is reckless are stated in 500. The rules for determining when conduct is abnormally dangerous are stated in 519, 520. Although the rules for determining negligence and recklessness are the same in respect to invasions of interests in the use and enjoyment of land as they are in respect to invasions of other interests, the fact that the actor's conduct involves a risk of harm through invasion of the interest in the use and enjoyment of land rather than through invasion of some other legally protected interest is often of importance. In order that the actor's conduct may be negligent or reckless, it must involve an unreasonable risk of harm. (See 282, 500). In determining the unreasonable character of a realizable risk, the value that the law attaches to the interest imperiled is an important factor. (See 293). Thus the actor's conduct may have sufficient utility ( 292) to outweigh a certain quantum of risk to another's use and enjoyment of land although it might not have sufficient utility to outweigh a similar quantum of risk to another's bodily security. j. Abnormally dangerous conditions and activities. The last basis for liability for a private nuisance is the defendant's abnormally dangerous activity, enterprise or maintained condition, under the rules stated in Chapters 20 and 21. Thus a dog known by the owner to be vicious may create a private nuisance when it interferes with the use or enjoyment of the land next door, and the owner may be subject to strict liability because of his knowledge of the dog's propensities. So likewise, blasting activities or the storage of a large quantity of explosives in an inappropriate place may create a private nuisance because of the resulting interference with the use and enjoyment of land in the vicinity. Even in jurisdictions that refuse to accept by name the principle in the case of Rylands v. Fletcher, (1868) L.R. 3 H.L. 330, and have rejected any general rule of strict liability for abnormally dangerous conditions or activities, the strict liability is commonly recognized and applied under the name of nuisance when it is found that there is an interference with the use and enjoyment of land or even with the rights of the general public. k. Unintentional invasions -- unreasonableness. As set forth in Clause (a), in order to give rise to a cause of action for private nuisance, an intentional invasion of the interest in the private use and enjoyment of land must be unreasonable. This requirement has an analogue in Clause (b), regarding unintentional invasions. The standards for negligent or reckless conduct, as stated in 291-293 and 500, involve a balancing process very similar to the test stated in 826 to 830 for determining whether an intentional invasion is unreasonable -- that of balancing the gravity of the harm against the utility of the conduct. One difference in language should be noted. The Sections on intentional harm refer to the magnitude of the risk, rather than only the gravity of the harm. In unintentional invasions, it is the risk of harm that makes the conduct unreasonable. The risk is a product of the likelihood of injury multiplied by the prospective extent of the harm. When the harm is intended, on the other hand, it is

necessary to look only at the gravity, or extent, of the harm actually suffered. Thus the test for negligence includes the concept of the gravity of the harm in the broader concept of the magnitude of the risk (see 293, especially Clauses (c) and (d)), and for conduct to be found negligent and therefore to constitute a private nuisance it must involve an unreasonable risk of an unreasonable invasion of another's interest in the use and enjoyment of land. Reckless conduct is subject to a similar analysis. It is also important to remember that for negligent or reckless conduct to be actionable as a private nuisance, the risk involved must be risk of the invasion of an interest in the use and enjoyment of land. Conduct that is negligent or reckless because it creates an unreasonable risk of some other kind of harm may not be determinative of the liability involved in this Section. (Compare 281, Comments c to f). An abnormally dangerous activity is not treated as unreasonable conduct in the same sense that negligent or reckless conduct is; but the process of balancing the conflicting factors to determine liability as indicated in 519 and 520, is similar. An abnormally dangerous enterprise is required to pay its way by compensating for the harm it causes, because of its special, abnormal and dangerous character. No particular purpose is served by determining whether the conduct should be characterized as unreasonable, even though a decision to that effect could be made by defining the conduct to include both the activity and the failure to pay for the harm it causes. Compare the similarity to intentional invasions, where it is sometimes held to be reasonable to carry on a socially useful enterprise if a payment is made for the harm it causes but unreasonable to continue it without paying. (See 826). In any event, for the abnormally dangerous activity to give rise to an action for private nuisance the harm must have come within the scope of the risk that made the activity abnormally dangerous. (See 519(2)). Restatement
826

RESTATEMENT 829(A)
The harm is unreasonable if it is severe and greater than that which the plaintiff should be required to bear without compensation. (does not take the financial stability of the defendant into account)

CARPENTER V. DOUBLE R CATTLE COMPANY The communitys interest comes into play under 826(a) and that is the state of the law in Idaho. The community is one of farming & cattle. The expansion of the farms is reasonable. D. PUBLIC NUISANCE Public policy considerations is the focus this is the hot, new legal theory that attorneys are bringing against lead paint manufacturers cutting edge use of this doctrine Benefits of this theory over negligence for lead paint 1. Defendants conduct causes 2. unreasonable interference 3. with a right common to the public, health, safety and to some extent convenience RESTATEMENT 2D OF TORTS, 821B: PUBLIC NUISANCE: (1) A public nuisance is an unreasonable interference with a right common to the general public. (2) Circumstances that may sustain a holding that an interference with a public right is unreasonable include the following: (a) Whether the conduct involves a significant interference with the public health, the public safety, the public peace, the public comfort or the public convenience, or (b) whether the conduct is proscribed by a statute, ordinance or administrative regulation, or (c) whether the conduct is of a continuing nature or has produced a permanent or long-lasting effect, and, as

the actor knows or has reason to know, has a significant effect upon the public right. COMMENTS & ILLUSTRATIONS: Comment: a. History. In its inception a public, or common, nuisance was an infringement of the rights of the Crown. The earliest cases appear to have involved purprestures, which were encroachments upon the royal domain or the public highway and could be redressed by a suit brought by the King. By the time of Edward III the principle had been extended to the invasion of the rights of the public, represented by the Crown, by such things as interference with the operation of a public market or smoke from a lime-pit that inconvenienced a whole town. The term "nuisance," which had acquired no very definite meaning other than that of something causing harm or inconvenience, was applied rather loosely to these interferences. By degrees the class of offenses was greatly enlarged until it came to include, in the words of a leading writer on criminal law, any "act not warranted by law, or omission to discharge a legal duty, which obstructs or causes inconvenience or damage to the public in the exercise of rights common to all Her Majesty's subjects." Stephen, General View of the Criminal Law of England (1890) 105. The remedy remained exclusively a criminal one in the hands of the Crown until the sixteenth century, when it was first held that a private individual who had suffered particular damage differing from that sustained by the public at large might have a tort action to recover damages for the invasion of the public right. (See 821C). b. Common law public nuisances. At common law public nuisance came to cover a large, miscellaneous and diversified group of minor criminal offenses, all of which involved some interference with the interests of the community at large -- interests that were recognized as rights of the general public entitled to protection. Thus public nuisances included interference with the public health, as in the case of keeping diseased animals or the maintenance of a pond breeding malarial mosquitoes; with the public safety, as in the case of the storage of explosives in the midst of a city or the shooting of fireworks in the public streets; with the public morals, as in the case of houses of prostitution or indecent exhibitions; with the public peace, as by loud and disturbing noises; with the public comfort, as in the case of widely disseminated bad odors, dust and smoke; with the public convenience, as by the obstruction of a public highway or a navigable stream; and with a wide variety of other miscellaneous public rights of a similar kind. In each of these instances the interference with the public right was so unreasonable that it was held to constitute a criminal offense. For the same reason it also constituted a tort. Many states no longer recognize common law crimes, treating the criminal law as entirely statutory. But the common law tort of public nuisance still exists, and the traditional basis for determining what is a public nuisance may still be applicable. c. Statutes. With the elimination of common law crimes, general statutes have been adopted in most of the states to provide criminal penalties for public nuisances, often without defining the term at all, or with only a very broad and sometimes rather vague definition. These statutes uniformly have been construed to include the interferences with the rights of the public that were public nuisances at common law. In other states the crime of public nuisance in this broad, vague and general sense has become anachronistic. It is regarded as inconsistent with the position that criminal conduct must be defined specifically and clearly and statutes are drafted to meet this requirement. In addition, all of the states have numerous special statutes declaring certain conduct or conditions to be public nuisances because they interfere with the rights of the general public. For example, a common type of statute declares black currant bushes or barberry bushes or other plants that harbor parasites such as rust that are destructive to grain or timber to be public nuisances. These statutes amount to a legislative declaration that the conduct proscribed is an unreasonable interference with a public right. Municipal ordinances and administrative orders and regulations may have a similar effect. In these cases there may be no need for a court finding of unreasonableness. d. Criminal character. It has been stated with some frequency that a public nuisance is always a criminal offense.

This statement is susceptible of two interpretations. The first is that in order to be treated as a public nuisance, conduct must have been already proscribed by the state as criminal. This is too restrictive. The second is that any conduct that is found to be a public nuisance is for that reason a criminal offense, either at common law or under statute. While this has been true in most of the cases in which the statement has appeared and is still so in many states, it affords little assistance in determining what conduct amounts to a public nuisance. It no longer has significance in states where the general crime of public nuisance has ceased to exist. In any event, there is clear recognition that a defendant need not be subject to criminal responsibility. Thus a municipal corporation, which cannot be prosecuted for a crime, may still be liable in tort for the creation or maintenance of a public nuisance if the conduct is such that a private individual would be liable. In addition, other remedies, such as the recovery of damages in tort by one who has suffered particular damages (see 821C) or abatement of the nuisance or injunction may lie in favor of the state or even a private individual who suffers particular harm. e. Unreasonable interference. The common law criminal offense of public nuisance involved an interference with a right common to the general public. Little more than this in the way of a standard for determining what kinds of interferences constitute the crime of public nuisance was to be found in the cases. But as the tort action came into the picture, the use of the single word "nuisance" to describe both the public and the private nuisance, led to the application in public nuisance cases, both criminal and civil, of an analysis substantially similar to that employed for the tort action for private nuisance. This analysis is set forth below in 826-831. Thus, by analogy to the rules stated in 822, the defendant is held liable for a public nuisance if his interference with the public right was intentional or was unintentional and otherwise actionable under the principles controlling liability for negligent or reckless conduct or for abnormally dangerous activities. Liability was not normally imposed for a pure accident that did not fall into one of the three traditional categories of tort liability. In each of these categories, some aspect of the concept of unreasonableness is to be found. If the interference with the public right is intentional, it must also be unreasonable. (See 822, and 826-831, involving the weighing of the gravity of the harm against the utility of the conduct). If the interference was unintentional, the principles governing negligent or reckless conduct, or abnormally dangerous activities all embody in some degree the concept of unreasonableness. (See 282, 500, 520). If the common law crimes for public nuisance have been supplanted or supplemented by a broad general statute, the situation has not been changed in any material respect, and the common law rules are generally still applicable to both criminal and civil liability. On the other hand, a specific statute may expressly provide, or be construed by the court to mean, that the only remedy for violation is the criminal prosecution, or at least that a tort action for damages is not intended as an available remedy. If, however, particular conduct is declared to be a public nuisance by a specific statute, an ordinance, or an administrative regulation, the act may provide, or be construed to mean, that the defendant is guilty of the crime even though his interference with the public right was purely accidental and unintentional. Within the constitutional or other limitations upon the power of the legislative body, these acts are conclusive as to the existence of the crime and the unreasonableness of the interference. This strict criminal responsibility is carried over to the tort action. There is a clear analogy to the doctrine of negligence as a matter of law, under which a legislative act is taken as laying down a specific rule of conduct that substitutes for the general standard of what a reasonable prudent man would do in like circumstances. (See 286). Depending upon the construction of the statute, the liability may be strict. (Cf. 288A, Comment c). If a defendant's conduct in interfering with a public right does not come within one of the traditional categories of the common law crime of public nuisance or is not prohibited by a legislative act, the court is acting without an established and recognized standard. The analysis set forth in 826-831 then becomes more significant. In view of the potentially widespread damage liability for a public nuisance, the court will give attention to the possibility that the defendant was not aware of the injurious character of his conduct and will usually confine the tort liability for damages to cases in which the defendant's conduct was of a continuing nature or produced a permanent or

long-lasting effect or its detrimental effect on the public right was significant and he knew or had reason to know (see 12) of that effect. In the awarding of an injunction less weight may be placed on the aspect of knowledge. Subsection (2) has listed three sets of circumstances for determining whether an interference with a public right is unreasonable. They are not conclusive tests controlling the determination of whether an interference with a public right is unreasonable. They are listed in the disjunctive; any one may warrant a holding of unreasonableness. They also do not purport to be exclusive. Some courts have shown a tendency, for example, to treat significant interferences with recognized aesthetic values or established principles of conservation of natural resources as amounting to a public nuisance. The language of Subsection (2) is not intended to set restrictions against developments of this nature. f. Effect of compliance. Although it would be a nuisance at common law, conduct that is fully authorized by statute, ordinance or administrative regulation does not subject the actor to tort liability. Aside from the question of the validity of the legislative enactment, there is the question of its interpretation. Legislation prohibiting some but not other conduct is not ordinarily construed as authorizing the latter. In the case of negligence as a matter of law, the standard defined by a legislative enactment is normally a minimum standard, applicable to the ordinary situations contemplated by the legislation. Thus traveling at less than the speed limit may still be negligence if traffic conditions indicate that a lesser speed is required. (See 288C). The same general principle applies to public nuisance. Consideration may appropriately be given, however, to the fact that acts were taken in reliance upon legislation, as when expensive screening processes are installed to reduce the level of pollution to a legislative maximum. In addition, if there has been established a comprehensive set of legislative acts or administrative regulations governing the details of a particular kind of conduct, the courts are slow to declare an activity to be a public nuisance if it complies with the regulations. Thus, at one time courts frequently engaged in "judicial zoning," or the determination of whether a particular land use was unsuitable to a locality and therefore unreasonable. Now that most cities have complete sets of zoning regulations and agencies to plan and administer them, the courts have shown an inclination to leave the problem of the appropriate location of certain types of activities, as distinguished from the way in which they are carried on, to the administrative agencies. The variety and complexity of a problem and of the interests involved and the feeling that the particular decision should be a part of an overall plan prepared with a knowledge of matters not presented to the court and of interests not represented before it, may also promote judicial restraint and a readiness to leave the question to an administrative agency if there is one capable of handling it appropriately. g. Interference with public right. Conduct does not become a public nuisance merely because it interferes with the use and enjoyment of land by a large number of persons. There must be some interference with a public right. A public right is one common to all members of the general public. It is collective in nature and not like the individual right that everyone has not to be assaulted or defamed or defrauded or negligently injured. Thus the pollution of a stream that merely deprives fifty or a hundred lower riparian owners of the use of the water for purposes connected with their land does not for that reason alone become a public nuisance. If, however, the pollution prevents the use of a public bathing beach or kills the fish in a navigable stream and so deprives all members of the community of the right to fish, it becomes a public nuisance. It should be noted that in some states there are statutes defining a public nuisance to include interference with "any considerable number of persons;" and under these statutes no public right as such need be involved. It is not, however, necessary that the entire community be affected by a public nuisance, so long as the nuisance will interfere with those who come in contact with it in the exercise of a public right or it otherwise affects the interests of the community at large. The obstruction of a public highway is a public nuisance, although no one is travelling upon the highway or wishes to travel on it at the time. In many cases the interests of the entire

community may be affected by a danger to even one individual. Thus the threat of communication of smallpox to a single person may be enough to constitute a public nuisance because of the possibility of an epidemic; and a fire hazard to one adjoining landowner may be a public nuisance because of the danger of a conflagration. In any case in which a private nuisance affects a large number of persons in their use and enjoyment of land it will normally be accompanied by some interference with the rights of the public as well. Thus the spread of smoke, dust or fumes over a considerable area filled with private residences may interfere also with the use of the public streets or affect the health of so many persons as to involve the interests of the public at large. h. Relation to private nuisance. Unlike a private nuisance, a public nuisance does not necessarily involve interference with use and enjoyment of land. A public nuisance as such does not afford a basis for recovery of damages in tort unless there is particular harm to the plaintiff, as stated in 821C. When the particular harm consists of interference with the use and enjoyment of land, the public nuisance may also be a private nuisance, as when a bawdy house that interferes with the public morals and constitutes a crime also interferes with the use and enjoyment of land next door. In this case the landowner may recover either on the basis of the particular harm to him resulting from the public nuisance or on the basis of the private nuisance. i. Action for damages distinguished from one for injunction. There are numerous differences between an action for tort damages and an action for an injunction or abatement, and precedents for the two are by no means interchangeable. In determining whether to award damages, the court's task is to decide whether it is unreasonable to engage in the conduct without paying for the harm done. Although a general activity may have great utility it may still be unreasonable to inflict the harm without compensating for it. In an action for injunction the question is whether the activity itself is so unreasonable that it must be stopped. It may be reasonable to continue an important activity if payment is made for the harm it is causing, but unreasonable to continue it without paying. On the other hand an award of damages is retroactive, applying to past conduct, while an injunction applies only to the future. In addition, for damages to be awarded significant harm must have been actually incurred, while for an injunction harm need only be threatened and need not actually have been sustained at all. (See 821F, Comment b). To maintain a damage action for a public nuisance, one must have suffered damage different in kind from that suffered by the general public; this is not necessarily true in a suit for abatement or injunction. (See 821C). This Chapter is concerned primarily with rules governing actions for damages. Rules governing suits for injunctions are treated in a general fashion in Chapter 48. Sometimes statements are made in this Chapter about rules governing suits for injunction or for abatement; on these occasions this will be specifically indicated. (Cf. 822, Comment d). RESTATEMENT 2D OF TORTS, 821C: WHO CAN RECOVER FOR PUBLIC NUISANCE: (1) In order to recover damages in an individual action for a public nuisance, one must have suffered harm of a kind different from that suffered by other members of the public exercising the right common to the general public that was the subject of interference. (2) In order to maintain a proceeding to enjoin to abate a public nuisance, one must (a) have the right to recover damages, as indicated in Subsection (1), or (b) have authority as a public official or public agency to represent the state or a political subdivision in the matter, or (c) have standing to sue as a representative of the general public, as a citizen in a citizen's action or as a member of a class in a class action. COMMENTS & ILLUSTRATIONS: Comment: a. Remedies for public nuisance. The original remedies for a public nuisance were a prosecution for a criminal

offense or a suit to abate or enjoin the nuisance brought by or on behalf of the state or an appropriate subdivision by the proper public authority. The first recorded case permitting a private action in tort was decided in 1536. Anonymous, Y.B.Mich., 27 Hen. 8, f. 26, pl. 10. This case held that a tort action could be maintained by a person who could show that he had suffered particular harm, over and above that caused to the public at large or to other members of the public exercising the same public right. This is one of two instances since the old days of the action of trespass in which a tort remedy has been engrafted onto a crime, without other circumstances or proof than the existence of the crime itself. The other is that of libel. The cases allowing tort damages for public nuisance constitute an exception to the rule stated in 288(b), that the requirements of a criminal statute will not be adopted by the courts to impose liability in tort when the purpose of the statute is found to be exclusively to secure to individuals the enjoyment of rights or privileges to which they are entitled only as members of the public. The limitation imposed in 1536 has persisted, and it is uniformly agreed that a private individual has no tort action for the invasion of the purely public right, unless his damage is to be distinguished from that sustained by other members of the public. It is not enough that he suffers the same inconvenience or is deprived of the same enjoyment or is exposed to the same threat of injury as everyone else who may be exercising the same public right. Redress of the wrong to the entire community is left to its duly appointed representatives. The reasons usually given for the rule are that it is essential to relieve the defendant of the multiplicity of actions that might follow if everyone were free to sue for the common wrong; and that any harm or interference shared by the public at large will normally be, if not entirely theoretical or potential, at least minor, petty and trivial so far as the individual is concerned. b. Difference in kind and degree. The private individual can recover in tort for a public nuisance only if he has suffered harm of a different kind from that suffered by other persons exercising the same public right. It is not enough that he has suffered the same kind of harm or interference but to a greater extent or degree. Thus when a public highway is obstructed and all who make use of it are compelled to detour a mile, no distinction is to be made between those who travel the highway only once in the course of a month and the man who travels it twice a day over that entire period. For both there has been only interference with the public right of travel and resulting inconvenience, even though the interference and the inconvenience have been much greater in the one case than in the other. The explanation of the refusal of the courts to take into account these differences in extent undoubtedly lies in the difficulty or impossibility of drawing any satisfactory line for each public nuisance at some point in the varying gradations of degree, together with the belief that to avoid multiplicity of actions invasions of rights common to all of the public should be left to be remedied by action by public officials. Illustration: 1. A builds a bridge that unlawfully obstructs a navigable stream. B, who has a motorboat and for his own pleasure navigates the stream five times as frequently as any other person in the community, is prevented from passing up the stream by the bridge. B cannot recover in tort for the public nuisance. c. Difference in degree of interference cannot, however, be entirely disregarded in determining whether there has been difference in kind. Normally there may be no difference in the kind of interference with one who travels a road once a week and one who travels it every day. But if the plaintiff traverses the road a dozen times a day he nearly always has some special reason to do so, and that reason will almost invariably be based upon some special interest of his own, not common to the community. Significant interference with that interest may be particular damage, sufficient to support the action in tort. Deprivation of immediate access to land (see

Comment f below), which is clearly a special kind of harm, shades off by imperceptible degrees into the remote obstruction of a highway, which is just as clearly not. Thus in determining whether there is a difference in the kind of harm, the degree of interference may be a factor of importance that must be considered. d. Physical harm. When the public nuisance causes personal injury to the plaintiff or physical harm to his land or chattels, the harm is normally different in kind from that suffered by other members of the public and the tort action may be maintained. Illustration: 2. A digs a trench across the public highway and leaves it unguarded at night without any warning light. B, driving along the highway, drives into the trench and breaks his leg. B can recover for the public nuisance. e. Private nuisance. When the nuisance, in addition to interfering with the public right, also interferes with the use and enjoyment of the plaintiff's land, it is a private nuisance as well as a public one. In this case the harm suffered by the plaintiff is of a different kind and he can maintain an action not only on the basis of the private nuisance itself, but also, if he chooses to do so, on the basis of the particular harm from the public nuisance. One important advantage of the action grounded on the public nuisance is that prescriptive rights, the statute of limitations and laches do not run against the public right, even when the action is brought by a private person for particular harm. Illustration: 3. A operates a house of prostitution, which by statute is declared to be a nuisance. This interferes with the use and enjoyment of B's dwelling next door. B can recover on the basis of either the private or the public nuisance. f. Access to land. The right of access to land, that is, the right of reasonable and convenient ingress and egress, is itself a property right in the land. If the public nuisance interferes with immediate ingress and egress to the plaintiff's land, the nuisance is a private as well as a public one and the harm suffered by the plaintiff is particular harm differing in kind from that suffered by the general public, so that the plaintiff can recover for the public nuisance. Complete deprivation of access, so that the land of the plaintiff is completely cut off is obviously sufficient particular damage. But the deprivation need not be complete and it is enough that the ingress or egress is made unreasonably burdensome or inconvenient or unsafe. Access by a particular entry is still a valuable property right even though there may be another entry left open; and the fact that there is access from the north left open does not prevent the recovery when the plaintiff is deprived of access from the south. Illustrations: 4. A digs a trench across the public street, which not only prevents travel on the street but also blocks the entrance to B's private driveway, so that B cannot get his car out of his garage. B can recover for the public nuisance. 5. The same facts as in Illustration 4, except that the entry into the driveway is not completely blocked but is made extremely difficult, inconvenient or dangerous. The same conclusion. 6. The same facts as in Illustration 4, except there is another entry to the driveway left open through an alley in the rear of B's premises. The same conclusion. g. Distance. When a public highway is obstructed, the distance of the obstruction from the plaintiff's land may be important in determining whether there is interference with access to it. If access is completely cut off, as

when the only road to the plaintiff's land is blocked, the distance is not material. But when there are other routes left open it may be decisive. If the plaintiff's point of entry upon his land abuts on the highway at the point of obstruction, there is clearly interference with access; and an obstruction a few feet away necessitating a long detour in order to proceed in a particular direction may also be enough. But when the obstruction is at a considerable distance from the land, the detour is to be regarded merely as a part of the inconvenience suffered by all members of the public who travel the highway and not particular damage of a different kind. Illustrations: 7. A's land is at the end of a public highway twenty miles long that is the only means of access to it. At the far end of the highway B destroys a bridge, cutting off all travel over the highway. A can recover for the public nuisance. 8. A travels daily from his home to his office over a public highway that is the most convenient route. Ten miles from A's home B obstructs the highway, compelling all those travelling on it to detour two miles. A cannot recover for the public nuisance. h. Pecuniary loss. Pecuniary loss to the plaintiff resulting from the public nuisance is normally a different kind of harm from that suffered by the general public. A contractor who loses the benefits of a particular contract or is put to an additional expense in performing it because of the obstruction of a public highway preventing him from transporting materials to the place of performance, can recover for the public nuisance. The same is true when it can be shown with reasonable certainty that an established business has lost profits, as when the obstruction of the highway prevents a common carrier from operating buses over it or access to the plaintiff's place of business is made so inconvenient that customers do not come to it. If, however, the pecuniary loss is common to an entire community and the plaintiff suffers it only in a greater degree than others, it is not a different kind of harm and the plaintiff cannot recover for the invasion of the public right. Illustrations: 9. A has a contract with B to float logs for B down a navigable stream. C constructs a dam in the stream, making it impossible to float the logs. As a result A cannot perform the contract or is put to heavy additional expense in performing it and so suffers pecuniary loss. A can recover for the public nuisance. 10. A constructs a bridge over a navigable stream, blocking navigation. B, who operates a commercial boat line on the stream, is prevented from passing the bridge and as a result loses profits. B can recover for the public nuisance. 11. A pollutes public waters, killing all of the fish. B, who has been operating a commercial fishery in these waters, suffers pecuniary loss as a result. B can recover for the public nuisance. i. Delay and inconvenience. When a highway is obstructed, the delay or inconvenience of a detour is a kind of harm suffered by any member of the public who travels that way. The delay or inconvenience suffered by a particular plaintiff, even though it may be greater in degree, is not particular harm of a different kind and will not support an action for the public nuisance. It is only when the plaintiff is able to prove that by reason of the delay he has been put to special expense of a different kind that he can recover on the basis of the particular harm. Comment on Subsection (2): j. Action to enjoin or abate. A public official who is authorized to represent the state or an appropriate

subdivision in an action to abate or enjoin a public nuisance may of course maintain the action. An administrative agency may also be given this authority, whether it promulgated the administrative regulations it is seeking to enforce or not. So also a person who has suffered damages that are different from those suffered by other members of the public and who is thus able to bring an action in tort for his damage is able to seek an injunction against the public nuisance. It has been the traditional rule that if a member of the public has not suffered damages different in kind and cannot maintain a tort action for damages, he also has no standing to maintain an action for an injunction. The reasons for this rule in the damage action are that it is to prevent the bringing of a multiplicity of actions by many members of the public and the bringing of actions for trivial injury. These reasons are much less applicable to a suit to enjoin the public nuisance and there are indications of a possible change. Statutes allowing citizens' actions or authorizing an individual to represent the public, and extensive general developments regarding class actions and standing to sue are all pertinent. Since standing to sue is primarily a procedural matter, not fully appropriate for a Restatement of the substantive law of Torts, it has been regarded as outside this scope of this Section to set forth the rules for determining when there is standing to sue for abatement or injunction. The purpose of the Subsection is to point out that there may be a distinction between an individual suit for damages and a suit in behalf of the public or a class action. The Subsection is worded so as to leave the courts free to proceed with developments regarding standing to sue without the restrictive effect that would be imposed by a categoric statement of the traditional rule, which is found in a limited number of cases. Private cause of action under public nuisance- if you can show you have been harmed in a different manner than everyone else must be unique to you ex. Blocking your driveway Lead-based paint litigation one view is that the attorneys must take up the mantle for those who are disenfranchised and a government that has refused to do anything This case was based on fault and the attorneys forwarded a case using negligence principles, about being right or wrong nuisance theory is attractive because there is no statute of limitations this is a strange theory because the paint was brought into the homes voluntarily and was legal at the time dont have to show fault under nuisance also dont have to show causation; there is a products liability claim but is outside of the statute of limitations; negligence claim but issues of causation F. REMEDIES What is private inverse condemnation? The airport cases, but think about this in relation to the Boomer case. Inverse condemnation is a term used in the law to describe a situation in which the government takes private property but fails to pay the just compensation required by the Constitution. In order to be compensated, the owner must then sue the government. In such cases the owner is the plaintiff and that is why the action is called inverse -- the order of parties is reversed, as compared to direct condemnation where the government is the plaintiff who sues a defendant-owner to take his property. Private inverse condemnation transfer of property to a private institution and not by the government BOOMER V. ATLANTIC CEMENT CO Plaintiff landowners neighbored defendant's cement factory. Plaintiffs sought an injunction for property damages from the factory's vibration, smoke, and dirt. The lower court found the factory a nuisance and ordered temporary damages, but denied an injunction. The court found it should not try to lay down a policy for the difficult problem of pollution elimination as the byproduct of private litigation. The court determined permanent damages were allowed where the loss recoverable is small in comparison with the cost of removal of the nuisance. The court further indicated permanent damages were appropriate when there was a continuing and recurrent nuisance, as in this case. The court found it equitable to award plaintiffs permanent damages based on the theory of compensation

for servitude on the land which precluded future recovery by plaintiffs or their grantees. The court granted an injunction which was vacatable upon defendant's payment of permanent damages. CHAPTER 13. DAMAGES Tort reform is aimed at punitive damages and non-pecuniary awards. Questions from Tripp What is remittitur? This is a procedural process when a jury award for damages is excessive the court may lower the amount, or essentially have the plaintiff remit a portion of the damages back to the defendant if the plaintiff does not agree then the case is remanded for a new trial. What criticisms of the "maximum recovery rule" do you have? Hmm, seems rather arbitrary that the judge can put a dollar amount on an award and then claim that since his award adds up to more than what the jury awarded it is not excessive or it can work they other way to lessen someones award if the judge finds that his damages amount to less than the jury came up with this seems to second guess the jury.
Maximum recovery rule the trial judge determines whether the verdict of the jury exceeds the maximum amount which the jury could reasonably find and if it does, the trial judge may then reduce the verdict to the highest amount that the jury could properly have awarded. The five cardinal elements of damages: past physical and mental pain, future physical and mental pain, future medical expenses (economic), loss of earning capacity (economic), and permanent disability and disfigurement.

A. COMPENSATORY DAMAGES To restore the plaintiff to her pre-injury condition by paying an amount equal to the value of the interests that the defendant has diminished or destroyed. ANDERSON V. SEARS, ROEBUCK & CO.
Infant plaintiff and her mother suffered burn injuries in a house fire caused by a defective heater, and her parents, individually and on her behalf, sued defendant manufacturers of the heater and its components, defendant installer and repairer, and defendant insurers. A jury returned a verdict for plaintiff and awarded damages. Defendants sought remitittur of the damages awarded infant plaintiff. The court applied the "maximum recovery rule," which directs a trial judge to determine whether the verdict of the jury exceeded the maximum amount it could reasonably have found, based on an analysis of all of the evidence as to each element of damages. It held that the award was well within the amount the jury could have awarded, and denied remitittur. Defendants' challenge of the verdict because it exceeded the amount requested in plaintiffs' complaint was rejected because the complaint was properly amended to conform to the verdict. The legal standard on which to gauge a jury verdict for remittitur purposes is the "maximum recovery rule." This rule directs the trial judge to determine whether the verdict of the jury exceeds the maximum amount which the jury could reasonably find and if it does, the trial judge may then reduce the verdict to the highest amount that the jury could properly have awarded. Thus, the court's task is to ascertain, by scrutinizing all of the evidence as to each element of damages, what amount would be the maximum the jury could have reasonably awarded.

RICHARDSON V. CHAPMAN
The court held that the car driver's economist was not required to use "neutral" figures-amounts having no relation to the damages alleged by the parties-in explaining the concept of present cash value to the jury. The economist's methodology was reasonable because he did not make a prediction of future growth and inflation rates, he was consistent in his treatment of inflation, and he did not adopt a method that would under or over compensate the car driver. The court reduced the differential between the jury's award for the car driver's future medical expenses and the higher figure presented in the testimony because only a portion of the difference was allowed for expenses that the car driver was likely to incur, but that were not included in the calculations. The court found that the car passenger's award for pain and

suffering was excessive and reduced it. The plain language of the lease agreement granted the truck lessor the contractual right to seek indemnity from the truck driver's employer for the amount in excess of the insurance policy provided under the agreement. The truck lessor was entitled to indemnity from the truck driver under an implied indemnity theory. A reward is excessive if it falls outside the range of fair and reasonable compensation (lacks evidentiary support), results from passion or prejudice or is so large that it shocks the judicial conscience.

Can money compensate for non-economic harm? No, not really - nothing can give you back what you have lost this is just to make the courts feel better that justice has been achieved. Which opinion do you think is more persuasive, the majority or dissent in McDougald v. Garber? Hmm, I agree with the majority I think that there should have to be some awareness for there to be damages awarded for subjective loss. This does not mean that the victim cannot recover any damages, just the non-pecuniary ones that seem more punitive than compensatory because the victim will not get to enjoy the benefits of these kind of damages anyway. I also agree with the majority that when a plaintiff is aware and can get damages for pain and suffering, a jury will take all of the victims loss into account when calculating damages so that loss of enjoyment will be compensated for under pain and suffering. MCDOUGALD V. GARBER Plaintiff underwent surgery by defendant doctor and suffered severe brain damage as a result of oxygen deprivation, leaving plaintiff comatose. On appeal, the court modified the award and granted a new trial on nonpecuniary damages on the basis that the trial court had erred in its jury instructions. The court found that a comatose patient, without some degree of cognitive awareness, could not recover for loss of enjoyment of life, and that in determining damages, loss of enjoyment should not have been considered separately from pain and suffering. What is the doctrine of mitigation of damages? The mitigation of damages is intended to prevent plaintiffs from recovery if they do not act in a manner that will lessen the damage done by the injury. If they refuse to try and abate the damage then the defendant should not have to be responsible for the subsequent worsening of the injury or a permanent injury that could have been mitigated by actions that the plaintiff has refused to undertake, such as surgery or therapy regimen. MEASURING ECONOMIC/PECUNIARY LOSSES Medical expenses, lost wages (more certain) and reduced earning capacity (more speculative) are the largest components Jurisdictions have the plaintiff choose either lost wages or reduced earning capacity reduced earning capacity is often allowed even if the plaintiff will continue to receive a wage equal to what was being received before the injury the theory is that the plaintiff may wish to pursue another career but be unable to do so due to the injury for example, losing an arm and then cannot be a farmer Problems also arise when the plaintiff is not a wage earner at the time of the accident if someone is working toward a career then wages can be based on that, unless they are only in high school and have not selected a career path for example in cases involving homemakers, the jury can be compensated based on the amount that is required to replace the activities she is no longer able to do Awards are reduced to present value and will be adjusted downward to reflect the extra value received by being paid a lump sum now so that the plaintiff is not given an unfair windfall Courts make an educated guess as to what a safe investment will be in the future Courts will also adjust an award upward to compensate for future inflation in the instance of medical bills because they know medical costs will rise.

PERMANENT DISABILITY IN CHILDREN Some courts will look at the childs aptitude to determine what kind of career path they may have chosen and compensate them based on that Other courts will look to see how far they are in their education program and allow recovery based on that career choice TAXES Some courts allow a jury to know that the award will not be taxed and can adjust the award downward in that respect Other courts will not allow a jury to know the award is not taxable Federal courts generally hold that juries are to be informed of the tax-free nature of the award NON-ECONOMIC/NON-PECUNIARY LOSSES Courts have difficulty determining non-pecuniary damages because they are not things that are bought and sold in order to determine what the value should be. It is very difficult to put a value on something that is subject and no market exists for comparison. Courts realize that money cannot fully compensate people but awarding damages will give something. Some courts hold that the plaintiff must be cognitively aware in order to recover nonpecuniary damages. DISABILITY AND DISFIGUREMENT There can be minimal and total disability as well as permanent disfigurement. MEASURING NON-ECONOMIC/NON-PECUNIARY Collateral Source Rule plaintiffs can recover damages for expenses paid by collateral sources such as health and auto insurance and the jury cannot hear evidence regarding these payments one justification is deterrence making people responsible supports the deterrence of bad behavior What is the collateral source rule and what are the criticisms of the doctrine? The collateral source rule allows a plaintiff to recover for expenses that others have paid on his behalf, such as health insurance or life insurance. The criticisms are that this allows the plaintiff to receive a windfall because they were not responsible for paying out of pocket but are still allowed to recover. COYNE V. CAMPBELL Plaintiff accident victim sought review of an order of the Appellate Division of the Supreme Court in the Fourth Judicial Department (New York) affirming the supreme court's judgment in his favor, in the accident's victim's negligence action against defendant tortfeasor, following the supreme court's determination that the accident victim could not recover the value of medical expenses that were gratuitously rendered to him. With respect to medical expenses, a plaintiff must show what he paid the doctor, and can recover only so much as he paid or was bound to pay. Minority Rule (which the majority followed) no collateral source rule and can only recover pecuniary losses when gratuitous services or payments have been given on behalf of the plaintiff Majority Rule Collateral Source Rule third party payments are not allowed into evidence and are recoverable SUBROGATION
The third party wanting to be reimbursed through the plaintiffs settlement for the things they paid for as a result of the accident.

MONTGOMERY WARD V. ANDERSON Four situations exist in which the collateral source rule does not apply. They are cases in which a collateral source of recovery may be introduced (1) to rebut the plaintiff's testimony that he or she was compelled by financial

necessity to return to work prematurely or to forego additional medical care; (2) to show that the plaintiff had attributed his condition to some other cause, such as sickness; (3) to impeach the plaintiff's testimony that he or she had paid his medical expenses himself; (4) to show that the plaintiff had actually continued to work instead of being out of work, as claimed. The court has also allowed evidence of collateral sources when the plaintiff opens the door to his or her financial condition. Plaintiffs recovery is not reduced by contributions from collateral sources. DUTY TO MITIGATE Occurs after the plaintiff is injured and does not do anything to prevent any further injury or act in such a way that would minimized the harm suffered; the defendant bears the burden to show that the plaintiff did not mitigate damages
TEST: In determining whether a plaintiff has unreasonably failed or refused to mitigate his damages by submitting to a surgical operation is whether, under the circumstances of the case, an ordinarily prudent person would do so; i.e. the duty to exercise reasonable care under the circumstances. FACTORS: 1) The Risk involved (Hazard of the operation); 2) the probability of success; 3) the expenditure of money or effort required (Learned Hand Risk Utility Balancing) tells us when someone has acted reasonably

ZIMMERMAN V. AUSLAND Defendant admitted liability in plaintiff's suit for damages for personal injuries sustained in an automobile accident. The issue of damages was submitted to a jury and defendant appealed the verdict. On appeal, the defendant contended that the trial court erred in submitting to the jury the issue whether plaintiff sustained a permanent injury. The court affirmed the verdict, and held that defendant failed to meet its burden in showing that plaintiff failed to mitigate her injuries. Defendant did not offer any evidence that plaintiff unreasonably failed or refused to submit to a surgical operation that may have alleviated the injury. Thus, the court determined that, as a matter of law, it could not support defendant's contention that plaintiff was required to submit to surgery and the related contention that, for failure to do so, she was barred from claiming damages for a permanent injury. The plaintiff in a personal injury case cannot claim damages for what would otherwise be a permanent injury if the permanency of the injury could have been avoided by submitting to treatment by a physician, including possible surgery, when a reasonable person would do so under the same circumstances. 2. HARM
TO

PROPERTY :

(A) destroyed or permanently taken - market value at the time of destruction or taking (B)- damaged property 1) 2)diminished value or cost of repairs if repair is reasonable - sentimental value require a repair even if it makes no economic sense (C) temporary deprivation of use fair rental value of the property while you have been deprived of the property and also substitute costs if incurred

GASPERINI V. CHRISTIAN SCIENCE MONITOR


Loss of slides he loaned to a friend uniqueness and earning level for each of the slides was taken into account no evidence that he was going to profit off of the pictures must ensure that there is some value before there is a need for market value

C. WRONGFUL DEATH AND SURVIVAL Who may recover? What damages are recoverable? WRONGFUL DEATH Close relative, determined by state statute Pecuniary harm & emotional harm, a portion of earnings limited to economic damages suffers as a result of the death of decedent, No Pain & Suffering SURVIVAL STATUTE Estate medical expenses, lost wages, decedents pain & suffering (all from time of injury to death

Effect of instantaneous death from tortious injury? Effect of death from natural causes not related to tortious injury?

Limited by what actually lost Does NOT bar claim

only) BARS claim

BARS action

Does NOT bar action

SURVIVAL STATUTE Concerned with what happens between the time of the injury and the time of death WRONGFUL DEATH Statutory beneficiaries - deals with the harm caused to those who are left behind - get economic damages from loss of support - no pain & suffering damages (bare majority allow for loss of consortium claim-moving toward) MURPHY V. MARTIN OIL CO. Plaintiff's husband was injured through defendants' negligence in a fire on defendants' premises. After he died nine days later, plaintiff filed a complaint claiming damages for wrongful death under the Illinois Wrongful Death Act (count I) and sought damages for conscious pain and suffering, loss of wages and property damage (count II). The circuit court allowed defendants' motion to strike the second count holding it failed to state a cause of action. The appellate court affirmed the dismissal of count II of the complaint as to its allegations of pain and suffering and reversed the judgment as to its allegations of loss of wages and property damage. Plaintiff appealed. The court reversed in part, holding a cause of action could be maintained for decedent's pain and suffering because that action would survive decedent's death allowing the estate to collect damages he no longer was able to pursue himself. B. PUNITIVE DAMAGES At early common law, emotional harm was not considered compensatory, but in our system they are considered compensatory therefore punitive damages came to be for deterrence and retribution to punish the defendant. Primary goals of punitive damages: BENEFITS OF PUNITIVE DAMAGES Punishment Deterrence defendants and others similarly situated Attorney fees are not compensable so punitive damages can be used to make the plaintiff truly whole by compensating for these fees and costs of the lawsuit Legislatures are capping awards and some are also allocating some of the damages to the State CRITICISMS Double-jeopardy can be punished in the criminal justice system as well as in a civil system Standard of evidence in the civil system is much less than in the criminal justice system there are significant procedural safeguards that allow for a safe punishment of wrongdoers and these safeguards do not exist in the civil system Runaway juries - statutes proscribe the punishment and it is not the function of the jury in a criminal setting the jury proscribes the punishment in civil systems and there is no guidance Windfall to the plaintiff the few win off the backs of the many who are not compensated

Again, there is a lot of difficulty in determining punitive damage awards, an even greater concern about the appropriateness of dealing with damages that intended to punish a defendant. How much is appropriate is enough to punish, and what type of conduct should we punish? OWENS-ILLINOIS V. ZENOBIA Defendants, a manufacturer and two suppliers/installers of asbestos containing products, challenged judgments entered in the circuit court awarding compensatory and punitive damages to plaintiffs in plaintiffs' strict liability action alleging that they were injured by exposure to asbestos. The court vacated the judgment of the Court of Special Appeals, which affirmed the compensatory damage awards as to all defendants and the punitive damage award against defendant manufacturer, and remanded for further proceedings. The court ruled, among other things, that the jury was not properly instructed with respect to the standard for awarding punitive damages against defendant manufacturer. For purposes of awarding punitive damages in a products liability action, the plaintiff is required to show that, armed with actual knowledge of the defective condition, the defendant consciously or deliberately disregarded the potential harm to consumers. Negligence alone, no matter how gross, wanton, or outrageous, will not satisfy this standard. Instead the test requires a bad faith decision by the defendant to market a product, knowing of the defect and danger, in conscious or deliberate disregard of the threat to the safety of the consumer. Implied Malice Standard criticism is that it covers all types of tortious conduct because there is no notice in order to deter really poor behavior essentially overly broad

*Reckless unreasonable conduct where the actor knows of a substantial and unjustifiable risk of harm and knowingly takes the risk of causing harm anyway Gross negligence unreasonable conduct that is extremely likely to cause harm even though the actor should know even though he doesnt know the risk of harm New Rule - In a non-intentional tort action, the trier of facts may not award punitive damages unless the plaintiff has
established that the defendants conduct was characterized by evil, motive, intent to injure, ill will, or fraud, ie., actual malice.

STATE FARM V. CAMPBELL The insureds contended that the substantial punitive damages award was justified in view of the insurer's national scheme to meet corporate fiscal goals by capping claim payments and engaging in fraudulent practices. The insurer argued that the ratio of punitive damages to compensatory damages clearly indicated that the punitive damages award was excessive and unrelated to the actual harm suffered by the insureds. The United States Supreme Court held that the punitive damages award was neither reasonable nor proportionate to the wrong committed, and it was thus an irrational, arbitrary, and unconstitutional deprivation of the property of the insurer. While the insurer's nationwide policies were clearly deficient, evidence of dissimilar and out-of-state misconduct of the insurer, and out-of-state conduct which was lawful where it occurred, was an improper basis for punishing the insurer for the limited harm to the insureds. Further, neither the wealth of the insurer nor the fact that its nationwide misconduct went largely unpunished justified punitive damages which were grossly disproportionate to the compensatory damages awarded for the actual harm to the insureds.
The guideposts for punitive damages are: (1) the degree of reprehensibility (court suggests that this is the most important aspect), (2) the disparity between the harm or potential harm suffered the ratio between the punitive damages and the compensatory damages; and (3) the difference between this remedy and the civil penalties authorized or imposed in comparable cases.

The Court does not specify what ratio is appropriate anywhere from saying single digit ratios would be appropriate, but that 145 to 1 is not we are somewhere in between and the degree of reprehensibility may have affect on the outcome

PHILIP MORRIS V. WILLIAMS The manufacturer contended that the jury was impermissibly permitted to calculate punitive damages based on harm to parties who were not parties to the litigation. The representative asserted that the jury was entitled to consider harm to nonparties in assessing the reprehensibility of the manufacturer's conduct for purposes of punitive damages. The U.S. Supreme Court held that, if the punitive damages award was based in part on the jury's desire to punish the manufacturer for harming nonparties, such an award amounted to a taking of property from the manufacturer without due process. While it was permissible to consider nonparty harm in determining reprehensibility, the punitive damages award to punish the manufacturer for injury inflicted on strangers to the litigation, without an opportunity to defend the charge, violated due process. Thus, procedures were required to inform the jury that, while harm to nonparties was relevant to reprehensibility, punitive damages could not be awarded to punish the manufacturer for such harm Substantive Due Process looks at the outcome Procedural Due Process looks at how the jury determined the award and what evidence a jury can use and in what manner it can use it to arrive at its determination 15. DEFAMATION Injury to reputation instead of physical harm or property damage your reputation has been harmed was a concept that was brought from England who placed such a high value on ones reputation early common law defamation was easily proven we traditionally regard free speech to be one of the most precious rights we have the power and significance of this tort has been continually diminished because or our deference to free speech under the First Amendment the interest in protection ones reputation v. the freedom to speak without government sanction Moving away from plaintiff friendly A. INTRODUCTION The elements of common law defamation are (1) the publication of (2) a statement of fact (3) that is false (4) defamatory (5) and that reasonably refers to the plaintiff. Requires the plaintiff to establish that the defendant made the defamatory statement with knowledge of its falsity in reckless disregard of the truth. B. WHAT IS DEFAMATORY ? GRANT V. READERS DIGEST Plaintiff filed a complaint alleging that defendant published an article that charged plaintiff with having represented the Communist Party, which was untrue and malicious. A man may value his reputation even among those who do not embrace the prevailing moral standards; and the jury should be allowed to appraise how far he should be indemnified for the disesteem of such persons. Restatement 559 A communication is defamatory if it tends so to harm the reputation of another as to lower him in the estimation of the community or to deter third persons from associating with him. C. THE FORM OF COMMUNICATION LIBEL AND SLANDER LIBEL Reduced to writing or is embodied in some permanent form such as a book or a painting do not have to prove pecuniary loss in order to recover damages

SLANDER Transmitted by the spoken word must prove pecuniary loss in order to recover damages - except if the defamatory statement consisted of: a. a major crime b. suffering from a loathsome disease c. conduct that would affect the plaintiff in her business, trade, profession, or office d. serious sexual misconduct - will then be treated as libel. D. FACT OR OPINION JANKLOW V. NEWSWEEK According to the governor, the article defamed him by implying that he began prosecuting the activist in revenge for the instigation of tribal charges by the activist against him for the alleged rape of an Indian girl, when in fact the governor, then serving as special prosecutor and later as attorney general, had initiated and continued proceedings against the activist for riot and assault with intent to kill prior to the initiation of the tribal charges against him. The court affirmed and utilizing the factors for distinguishing fact from opinion held that the disputed statement in the article was not nearly so precise as a direct accusation of improper motive; that the implication was not the only plausible one; that the magazine's generally freer style of personal expression and its pro-activist posture would alert the reader to expect a fair amount of opinion; and that criticism of the conduct of a state attorney general was a matter of national importance entitled to First Amendment protection. The court also stated that the implication of revenge here stemmed from semantic ambiguity, not from false statements. RULE:
First, is the precision and specificity of the disputed statement. Something that is vague or imprecise can hardly be considered factual. Second, is the concept of verifiability. If a statement cannot be verified than it cannot be considered a fact. Third, is the literary context in which the disputed statement was made. This puts the statement in a social context in so far as the category of the publication, its style and intended audience. Fourth, a court must then consider public context. It is important to consider the public or political arena in which the statement is made.

E. COMMUNICATING THE DEFAMATION TO OTHERS PUBLICATION Must be intentional or negligent; Third party must have heard and understood the communication and its perceived defamatory aspects; If it is foreseeable that someone would see or hear the alleged defamation than that is publication; If you disclose to one of the partys about an alleged defamatory statement regarding two people, you have still published as against the other party involved in the statement VAN-GO TRANSPORT V. NEW YORK CITY RULE: compelled publication is satisfied if it was foreseeable that the allegation would be reproduced by the plaintiff and there was a compulsion to publish the material this is the minority rule. This generally comes up in employee firing cases the employee is compelled to explain why they have been fired from their last job courts have held generally that employers are not liable for defamation F. DID THE ARROW HIT THE TARGET ? OF AND CONCERNING NEIMAN-MARCUS V. LAIT RULE:
THE

PLAINTIFF

1.Where the group is large, none can sue even though the language used is inclusive unless the plaintiff is individually named or is easily distinguished in the alleged libelous statement. 2. Where the group is small, and each and every member of the group or class is referred to, then any individual member can sue. 3. Where the group is small and only some of the class is referred to, where no attempt is made to exclude the innocent they may all sue. - Essentially the size of the group defamed is the important factor. G. THE CONSTITUTION AND THE LAW OF DEFAMATION NEW YORK TIMES V. SULLIVAN RULE: A public official must prove that the published defamatory statement was made with actual malice, which is with knowledge that it was false or with a reckless disregard as to the truth by clear and convincing evidence, in order recover damages. Courts have held that no evil intent is implicit in actual malice. This was a ground-breaking move away from the common law rule for libel and was a strict departure from the ease of proving libel this makes it extremely difficult for a public official to recover for libel. The right to criticize our government, protection of political speech is of the utmost importance, but this rule protects misinformation as well as factual information. The concern is that this rule goes too far and does not protect plaintiffs with any right to protect their reputations. This also encourages false speech. GERTZ V. ROBERT WELCH RULE Actual malice is not required if there is negligence in order for a private citizen plaintiff to recover for libel, but some fault is required. Courts may not award punitive damages for libel. RULE Public official is either an individual who may achieve such pervasive fame or notoriety that he becomes a public figure for all purposes and in all contexts (all-purpose), or an individual voluntarily injects himself or is drawn into a particular public controversy and thereby becomes a public figure for a limited range of issues (limitedpurpose). 1. Must first determine what type of figure the plaintiff is: (a) Public official or public figure NYT v Sullivan Rule applies (b) Private figure Gertz rule applies and you do not have to show actual malice 2. Plaintiff must show some fault no more recovery for strict liability 3. There can be no recovery for punitive damages unless plaintiff can show actual malice actual injury out of pocket expenses, damage to reputation, personal humiliation, and mental anguish and suffering. The court held that private individuals should not be brought under Sullivan in cases where there is an important public interest judges should not be responsible for deciding what an important public interest is (this will be addressed later). DUN AND BRADSTREET RULE

A private citizen can collect presumed and punitive damages without showing actual malice if the defamatory statement is not in regard to public matter. A private citizen with a private statement actual malice is not necessary to recover presumed and punitive damages must still show fault. Private person + private matter = presumed and punitive without showing actual malice Private person + public matter = presumed and punitive must show actual malice can still collect compensatory damages PHILADELPHIA NEWSPAPERS V. HEPPS RULE A private citizen bears the burden of proving the falsity of a defamatory statement when the matter is of public concern and the defendant is the media. A public figure must show the falsity of the statement. The Court in this case acknowledges that there will be cases of demonstrably false statements that will not be able to recover under this rule, but the Court comes down on the side of the First Amendment. This is the first case in which the Court brings in the identity of the defendant this only holds for the media they essentially contradict what they wrote in Dun and Bradstreet. Keep in mind that the core protections of the First Amendment will take precedence in matters of public concern this is the direction the Court has been taking us The Court refused to change the common law in regard to a private citizen and a private matter. FAULT Deals with the way in which the statement is published in Dun and Bradstreet fault deals with whether they were at fault in publishing the information (pay attention to the level of fault this leads to what damages can be collected) FALSITY Looks at the statements themselves to determine if they are true or false in Dun and Bradstreet whether the statements were true or not Defamatory Falsity Falsity Of & Published Statement? Presumed? Proved Concerning by ? by ? ? (just 1 ok) X X X X Fault? Presumed & Punitive Damages? X Actual Damages Only? NEVER bc they must show actual malice which limits damages to presumed & punitive Yes if negligence but no

Public Official / Public Figure (NY Times)

Actual Malice

Private Person / Public

Yes if is NOT media

Yes if IS media

Negligence or higher required

Yes if actual malice

Matter (Gertz) Private Person / Private Matter Dunn & Bradstreet

(NO strict liability) Strict liability or higher

actual malice X

Falsity asks Is statement true? Fault asks Did make decision to publish? Knowledge of falsity? Reckless disregard for truth? MILKOVICH V. LORAIN JOURNAL RULE
The First Amendment does not require a separate "opinion" privilege limiting the application of state defamation laws.

The Court holds that the Janklow test is still used throughout the country and the Court did not abolish this test they just said it was not required. This case has largely been disregarded. There are opinions that either state or imply facts in them. There are other opinions that do not either state or imply any facts. The Court makes this distinction to show that there are some opinions that should not be protected and therefore opinions should not be constitutionally protected. The Court takes us through an analysis based on Hepps a statement must be proven false The statement must be provably false they hold that I think Mayor Jones is a liar this statement would be actionable. In my opinion Mayor Jones shows his abysmal ignorance by accepting the teachings of Marx and Lenin - the Court holds that this would not be actionable because it cannot be provably false. Bresler-Letter Carriers Falwell imaginative expression or rhetorical hyperbole are protected because they are not reasonably interpreted as stating facts. New York Times Butts-Gertz culpability requirements protect uninhibited, robust and wide-open debate on public issues. EXAM will require more than the common law analysis must identify the type of plaintiff, type of matter and the constitutionality of the statements H. PRIVILEGES
AFTER THE

CONSTITUTIONAL TAKEOVER

1. QUALIFIED PRIVILEGES AT COMMON LAW A qualified privilege can be defeated if: 1. defending oneself against an accuser, 2. to protect the interest of the recipient, 3. to protect the interest of a family member, 4. share a common interest with others in a particular subject matter. ERICKSON V. MARSH & MCLENNAN

RULE The appropriateness of the occasion on which the defamatory information is published, the legitimacy of the interest thereby sought to be protected or promoted, and the pertinence of the receipt of that information by the recipient. This test goes to whether a qualified privilege exists at all. Once a court has determined that there is a qualified privilege it must be determined whether the privilege has been lost by proving actual malice with clear and convincing evidence. The cases show that there are certain situations that are so important that the court will give them a qualified immunity so that speech is not hampered so much that important information is still published. 2. ABSOLUTE PRIVILEGES AT COMMON LAW An absolute privilege cannot be defeated Congress, Judicial Process, state legislatures CHAPTER 16: PRIVACY Started with a Law Review article by Brandeis RIGHT OF PRIVACY 1. Intrusion upon ones seclusion or solitude, 2. Public disclosure of embarrassing private facts, 3. Publicity which places one in a false light, 4. Appropriation of ones name or likeness for the defendants advantage. These are essentially different causes of action must be analyzed separately. false light is similar to defamation and will usually come together? A. APPROPRIATION OF THE PLAINTIFF S LIKENESS FOR THE DEFENDANT S ADVANTAGE protects property rights RESTATEMENT 652C: One who appropriates to his own use or benefit the name or likeness of another is subject to liability to the other for invasion of his privacy. CARSON V. HERES JOHNNY PORTABLE TOILETS RULE
A celebrity has a protected pecuniary interest in the commercial exploitation of his identity and that if the celebrity's identity was commercially exploited, there has been an invasion of his right whether or not his name or likeness was used.

Now covers an individuals name, likeness, achievements, identifying characteristics or actual performances, and phrases that are associated with someone Policy considerations for the right of publicity 1. the right of publicity vindicates the economic interests of celebrity, enabling those whose achievements have imbued their identities with pecuniary value to profit from their fame 2. the right of publicity fosters the production of intellectual and creative works by providing the financial incentive for individuals to expend the time and resources necessary to produce them 3. the right of publicity serves both individual and societal interests by preventing what our legal tradition regards as wrongful conduct; unjust enrichment and deceptive trade practices ZACCHINI V. SCRIPPS-HOWARD RULE The media does not have a constitutional privilege to infringe upon the exclusive control over the publicity given to his performances in entirety property right B. PUBLIC DISCLOSURE
OF

EMBARRASSING PRIVATE FACTS ABOUT PLAINTIFF

1. Publicizing or communicating to a significant amount of people of 2. private facts 3. highly offensive to a reasonable person which are 4. not of a legitimate public interest. COX BROADCASTING CORP. V. COHN Appellant broadcasting company's reporter broadcasted the name of a rape victim over the air. He obtained the name of the victim through court papers. Ga. law made it a misdemeanor to publish or broadcast the name or identity of a rape victim. Appellee parent brought an action for money damages against appellants pursuant to this statute claiming that his right to privacy had been invaded by the television broadcasts giving the name of his deceased daughter. The issue was whether consistent with U.S. Const. amend. I and U.S. Const. amend. XIV, a state might extend a cause of action for damages for invasion of privacy caused by the publication of the name of a deceased rape victim that was publicly revealed in connection with the prosecution of the crime. The Georgia Supreme Court held that the trial court had erred in construing 26-9901 to extend a civil cause of action for invasion of privacy. On appeal, the court reversed the judgment of the Georgia Supreme Court. The court held that the cause of action for invasion of privacy through public disclosure of the name of a rape victim imposed sanctions on pure expression, the content of a publication. GATES V. DISCOVERY COMMUNICATIONS, INC. The issue to be decided was whether defendants could be held liable in tort for publishing information they gathered from public official court records concerning plaintiff, a person who many years previously served a prison term for a felony conviction but who had since lived an obscure, lawful life and become a respected member of the community. The court of appeal concluded defendants could not be held liable under such circumstances. The court affirmed the judgment of the court of appeal, holding that an invasion of privacy claim based on allegations of harm caused by a media defendant's publication of facts obtained from public official records of a criminal proceeding was barred by the First Amendment to the United States Constitution. The court reversed its previous holding that actionable invasion of privacy could occur through the reckless, offensive, injurious publication of true, but not newsworthy, information concerning the criminal past of a rehabilitated convict. Because plaintiff's invasion of privacy cause of action was barred, defendants' motion brought pursuant to Cal. Code Civ. Proc. 425.16, the anti-SLAPP statute, should have been granted. C. PLACING THE PLAINTIFF IN A FALSE LIGHT Plaintiff must establish that the defendant:1. publicized to a significant amount of people 2. false facts 3. that a reasonably prudent person would find highly offensive. TIME, INC. V. HILL Appellant publisher of a national magazine sought review of judgment, contending that it was denied constitutional protections of speech and press by the application of N.Y. Civ. Rights Law 50-51 to award appellee damages on allegations that appellant falsely reported that a new play portrayed an experience suffered by appellee and his family. On appeal, judgment was reversed and remanded. In support of its ruling, the Supreme Court held that the constitutional protections for speech and press precluded the application of the statute to redress false reports of matters of public interest in the absence of proof that appellant published the report with knowledge of its falsity or in reckless disregard of the truth. The Court noted that the evidence could have supported either a jury finding of innocent or merely negligent misstatement by appellant, or a finding of appellant's reckless disregard of the truth or actual knowledge of falsity. The jury instruction given by the lower court, however, failed to confine the jury to a verdict of liability based upon a finding that the statements in the article were made with knowledge of their falsity or in reckless disregard of the truth. RESTATEMENT 2D OF TORTS, 652E: PUBLICITY PLACING PERSON IN FALSE LIGHT: One who gives publicity to a matter concerning another that places the other before the public in a false light is

subject to liability to the other for invasion of his privacy, if (a) the false light in which the other was placed would be highly offensive to a reasonable person, and (b) the actor had knowledge of or acted in reckless disregard as to the falsity of the publicized matter and the false light in which the other would be placed. D. INTRUSION UPON THE PLAINTIFF S SOLITUDE Plaintiff must establish that the defendant engaged in 1. intentional improper intrusive conduct 2. to elicit truly private facts not contained in any public record or through normal inquiry or observation 3. and the intrusion would be highly offensive to the normal person NADER V. GENERAL MOTORS CORP. Appellants sought review of appellate court's order denying appellants' motion to dismiss in respondent's action for invasion of privacy. Respondent alleged appellants engaged in harassing conduct with intent of preventing respondent from publishing his book, which criticized appellants' safety and design of automobiles. Court affirmed and determined that under law of District of Columbia, respondent had set out a claim for invasion of privacy. Court found District of Columbia intended to protect individuals from others who would unreasonably intrude into the personal affairs of others and disclose confidential information about the individual. There was no invasion of privacy claim set out where respondent alleged appellants asked friends of respondent personal information about respondent. Claim for invasion of privacy was set out where appellants wiretapped respondent's conversations.

S-ar putea să vă placă și